Exam Set 1

Pataasin ang iyong marka sa homework at exams ngayon gamit ang Quizwiz!

A. instability of thoughts and feelings about the therapist and themselves (the *best* answer) A characteristic of Borderline Personality Disorder is a pervasive pattern of instability in interpersonal relationships. While the other answers describe characteristics of individuals with BPD, this one offers the most general and complete description, and is, therefore, the best answer

In psychotherapy, patients with Borderline Personality Disorder are most likely to exhibit: A. instability of thoughts and feelings about the therapist and themselves B. idealization of the therapist C. feelings of inadequacy D. hostility towards the therapist

D. In all research with animals, the animals must be treated humanely (b.). If surgical procedures are performed, they are done under appropriate anesthesia (c.) with techniques to avoid infection and minimize pain. According to Ethical Standard 6.20 (Care and Use of Animals in Research), "when it is appropriate that the animal's life be terminated, it is done rapidly, with an effort to minimize pain, and in accordance with accepted procedures.

In research, a psychologist can terminate the life of an animal: A. under no circumstances B. as long as the animal is treated humanely C. as long as an anesthetic is used, with an effort to minimize pain D. when it is appropriate and if it is done rapidly, with an effort to minimize pain

A. Research on the validity of selection measures generally finds the cognitive ability tests and biodata are 1-2 in terms of predictive validity. Although validity coefficients will vary from study to study, the most widely accepted conclusion is that general ability tests are the best overall predictors

Of the following, the best predictor of job success would likely be a: A. general cognitive ability test. B. personality test. C. structured interview. D. vocational interest inventory.

D. Marriage, especially when reinforced with children, appears to lessen the risk of suicide. Among married people, the rate of suicide is about 11 per 100,000. This rate is higher for single, never-married persons (about 22 per 100,000), even higher for widows (24 per 100,000), and higher still for divorced individuals (40 per 100,000

Of the following, the highest rate of suicide occurs among: A. married persons. B. never-married persons. C. widowed persons. D. divorced persons.

Internalized racism (def)

refers to acceptance of negative messages about the abilities and intrinsic worth by members of the stigmatized races and manifests in embracing "whiteness", self-devaluation, and resignation, helplessness, and hopelessness

ultimate attribution error (def)

refers to attributional biases that occur at the group level. The tendency to assume that whole groups of people have similar dispositions may be based on stereotypes or limited evidence and occur to protect a group in which one belongs

Personally mediated racism

refers to prejudice and discrimination at the individual level, whether intentional or unintentional, and manifests as lack of respect, suspicion, devaluation, scapegoating and dehumanization

Subtle racism

refers to the beliefs, attitudes, and actions of individuals (rather than institutions) and is a less obvious form of racism, which has been proposed to have replaced overt or "old-fashioned" prejudice and discrimination

C. Rosenhan's famous "pseudopatient study" involved 8 confederates who presented themselves to mental hospitals complaining of hearing voices. All were admitted to the hospital and 7 of the 8 were given the diagnosis "Schizophrenia." Once admitted, they all stopped feigning any symptoms and behaved normally. Nevertheless, their hospital stays ranged from 7 to 52 days with an average stay of 19 days and although nearly one-third of the other patients recognized the pseudopatients as "normal," none of the hospital staff recognized them as such.

The "pseudopatients" who were admitted to mental hospitals in Rosenhan's (1973) study were: A. all discharged from the hospitals within72 hours due to their "remission" B. identified by hospital staff as being "normal" during the second week of their stay C. identified by other patients but not staff members as being "normal" D. identified by staff members as being "normal" only after they stopped feigning symptoms

C. Despite the continued popularity of policies for deinstitutionalization, research indicates that up to 80 percent of those released are readmitted within two years of discharge. Readmission rates have been attributed to lack of support in the community, inadequate coordination between the community programs and the hospitals, poor psychiatric follow-up, and a lack of governmental support and funding. Aggressive community outreach programs have shown to be effective when they are well-staffed and well-financed.

The "revolving door" phenomenon of deinstitutionalizing psychiatric patients refers to: A. placing patients into different facilities, such as halfway houses, as symptomology changes B. releasing patients to aftercare clinics for psychiatric services and community programs for rehabilitation services. C. required readmission to psychiatric hospitals. D. patients seeking services and support through community outreach programs only when symptoms increase.

A. The American Psychological Association's position on partner notification is consistent with its emphasis on maintaining client confidentiality. The APA's (1991) position is that "a legal duty to protect third parties from HIV infection should not be imposed." If legislation were to be passed, the APA recommends it should only permit disclosure when: the psychologist knows of an identifiable third party at significant risk for infection; knows that the third party is unaware of the risk; has urged the client to notify the third party but he/she has refused to do so; and the psychologist is legally protected from civil or criminal liability for disclosing

The APA's position on partner notification in cases of HIV/AIDS is that psychologists: A. not be required legally to protect the partners of individuals with HIV/AIDS. B. be required legally to protect the partners of individuals with HIV/AIDS. C. be required legally to protect the partners of individuals with HIV/AIDS, provided the psychologist is protected from civil and criminal liability for disclosing. D. be required legally to protect the partners of individuals with HIV/AIDS after obtaining the client's permission to disclose.

A. Although coping with sexual assault is influenced by an individual's personality, support system, the responding professionals' reactions after the assault and the assault itself, research has shown that there is a set of characteristic reactions, referred to as the Rape Trauma Syndrome, that many sexually assaulted individuals experience. The Acute Distress stage immediately follows the assault and is associated with an acute reaction taking the form of shock, disbelief, and dismay. Two styles of responding to the assault include an Expressive style (crying, sobbing, shaking, restlessness, tenseness), and a Controlled style (appearing calm, subdued, numb). The Outward Adjustment stage is characterized by denial (c.), suppression or rationalization; and the Integration stage, which can overlap the others, is the long-term response of striving to come to terms with the assault.

The Acute Phase of rape trauma syndrome is characterized by: A. disbelief B. fear C. denial D. depression

B. According to the Americans with Disabilities Act and the Federal Uniform Guidelines on Employee Selection Procedures, business necessity and job relatedness are conditions that may permit the use of a selection or other employment procedure that results in an adverse impact. If an employer can demonstrate that it is job related and a business necessity, despite having adverse impact, the employer may be able to continue using the procedure

"Business necessity" and "job relatedness" are related to: A. personnel training B. adverse impact C. comparable worth D. truth in testing

C. Like other episodic memories, flashbulb memories are for events that involve specific people, places, or things and that occurred at a specific time.

"Flashbulb memories" are vivid memories of emotionally-arousing events. Flashbulb memories are considered part of: A. procedural memory. B. implicit memory. C. episodic memory. D. prospective memory.

D. As the name implies, test-retest reliability involves administering a test to the same group of examinees at two different times and then correlating the two sets of scores. This would be most appropriate when evaluating a test that purports to measure a stable trait, since it should not be significantly affected by the passage of time between test administrations.

208. reliability would be most appropriate for which of the following types of tests? A. brief B. speed C. state D. trait

D. A mixed standard scale tries to overcome halo, leniency, and similar rater biases by arranging, in a non-hierarchal manner, items that describe performance as either good, average, or poor and then the rater rates whether the individual performs better, equal to, or worse (+, 0, -) than the behavior described in the item. Items are arranged in a way, which supposedly helps reduce rater biases

A "mixed standard scale" helps reduce rater biases by: A. using a forced-choice format that matches items with a comparable level of social desirability. B. using a specific description of work-related behavior to anchor each point on the rating scale. C. comparing two or more employees on each job behavior. D. obscuring order-of-merit when describing work-related behaviors.

D. Declarative or explicit memories are long-term memories that one can consciously recall. They include semantic (factual information) and episodic (personally experienced) memories. Riding a skateboard (A) requires procedural memory. All of the other choices require explicit memory; however, memory deficits due to head trauma or disease usually affect recent long-term memory before affecting remote long-term memory.

A 17-year-old male involved in a skateboarding accident suffered a head trauma resulting in deficits to his declarative memory. He would likely have the most difficulty: A. riding a skateboard B. recalling when he rode his first skateboard at 10-years-old C. recalling his own name D. recalling what he ate for lunch yesterday

B. A distinction is made between formative and summative evaluation in the literature on evaluation. Formative evaluations are conducted during the delivery of the intervention to identify ways to improve it. Summative evaluations (a.) are conducted at the end of the intervention to evaluate its effectiveness and participants' reactions to it. A needs analysis (d.) is the first step in developing a training program. It is conducted prior to developing an intervention to determine what the intervention should include. Whereas the utility analysis (c.) summarizes and identifies key variables that describe the consequences of HR programs

A __________________ is performed during the development or delivery of an intervention to determine if it needs to be modified in order to meet its goals. A. summative evaluation B. formative evaluation C. utility analysis D. needs analysis

A. In higher-order conditioning, a CS, after being paired with a US, becomes reinforcing on its own and can be paired with a second CS. Stimulus generalization (b.) occurs when one conditioned stimulus is generalized to other stimuli. Response generalization (c.) occurs when a CS elicits not only the CR but other responses similar to it. Aversive counterconditioning (d.) uses classical conditioning to reduce or eliminate an undesirable behavior.

A bell is paired with food resulting in salivation, the bell is then paired with a light until the light alone elicits salivation. This is an example of: A. higher-order conditioning B. stimulus generalization C. response generalization D. aversive counterconditioning

B. When a parent has Posttraumatic Stress Disorder, the entire family is affected, and children are particularly vulnerable to the disruption it causes in the family system. The disorder often impairs one's ability to parent, and it may render parents more likely to become impatient, angry, or neglectful with their children. Young children especially are not cognitively equipped to make sense of this behavior, and research has identified a number of behavioral problems they are likely to display. Depression, anxiety, self-blame, aggression, hyperactivity, and social withdrawal are common, and so are symptoms of PTSD itself, such as low frustration tolerance and outbursts of anger--some describe PTSD as a disorder that may be transmitted intergenerationally. Even though such children may experience a variety of behavioral problems, hyperactivity is the best answer because, of the choices listed, it is the one most commonly identified as occurring in children of individuals with PTSD

A child of a military veteran with Posttraumatic Stress Disorder is most likely to display which of the following behavioral problems? A. kleptomania B. hyperactivity C. Oppositional Defiant Disorder D. eating disturbances

A. The key issue in these prevention questions is not the age of the target population, but rather, whether or not they already have the identified problem, and if so, whether it's in an early stage (secondary prevention) or full-blown (tertiary prevention). Since the question did not indicate that these senior citizens have already been victims of financial scams, this class would be considered primary prevention. Primary prevention is aimed at preventing problems before they occur in the first place by targeting high risk groups.

A class designed to teach senior citizens how to avoid financial scams, led by former con artists, would be considered: A. primary prevention B. secondary prevention C. tertiary prevention D. rehabilitative consultation

C. This question is about multiple relationships. Ethical Standard 3.05(a) (Multiple Relationships) indicates that, "A psychologist refrains from entering into a multiple relationship if the multiple relationship could reasonably be expected to impair the psychologist's objectivity, competence, or effectiveness in performing his or her functions as a psychologist, or otherwise risks exploitation or harm to the person with whom the professional relationship exists." Thus, if there are just two of you in the class, this relationship might impact the therapeutic one, while in a large class, the effects will probably be negligible. The Standard doesn't tell you how to decide-it just instructs you to determine what is in the best interest of your client

A client of yours has signed up for the same exercise class you attend. You should: A. Terminate the client B. Change your health club membership C. Make a decision depending on whether the class has two or 100 participants D. Ask the client if he can tell that you are losing weight

D. This question seemingly presents two ethical issues: multiple professional relationships and provision of therapy to a client who is already in psychotherapy. However, the first issue takes precedence here because teacher/psychologist is an example of a multiple relationship that should be avoided. Standard 3.05 of APA's Ethical Standards states that "A psychologist refrains from entering into a multiple relationship if the multiple relationship could reasonably be expected to impair the psychologist's objectivity, competence, or effectiveness in performing his or her functions as a psychologist, or otherwise risks exploitation or harm to the person with whom the professional relationship exists." If a teacher accepts the student as a psychotherapy client, there would be clear risk of potential harm to the client and impairment of the psychologist's objectivity. The boundaries between the two roles could easily become blurred and overlap. Therefore, regardless of the circumstances, the psychologist should not accept the student as a psychotherapy client, and of the choices available, only choice four involves a refusal to accept the case. Because the psychologist should not even consider accepting the case due to the multiple relationship issue, the issue of providing services to a person already undergoing psychotherapy becomes moot

A clinical psychologist who teaches courses at a college is approached by a student who wishes to begin therapy with the teacher. The student is already in therapy with another psychologist. In this situation, the psychologist should: A. only accept the case if the psychologist believes that doing so would be beneficial to the patient's welfare. B. take the case if the student signs a release of information so that both psychologists can work together. C. seek consultation from an objective third party before deciding on a course of action. D. advise the student to discuss the situation with his current therapist.

A. In a forced distribution, or forced ranking, system of personnel evaluation, employees are basically ranked against each other. The method requires managers to distribute ratings into pre-specified performance distributions or rankings (e.g., 1 out of 100, top 10%), etc. Regarding the other choices, the forced choice methodology basically involves using a multiple choice scale to evaluate employees on a variety of dimensions; the rater must select one statement from a set (two or more) of statements that best describes the employee. BARS stands for Behaviorally Anchored Rating Scales. These are constructed by doing a job analysis to identify critical incidents, or specific behaviors that are associated with good or poor performance. Using such as scale, a rater would choose the behavior most characteristic of the employee being evaluated. Finally, the method the question describes is not illegal as long it is applied fairly, and in fact describes the method of employee evaluation that Jack Welch implemented at GE in the 1980s and has been adopted by many other firms.

A company that creates an overall ranking of its employees based on a variety of performance outcome measures and fires the employees in the bottom 10 percent of these rankings is using a(n) ______________ method of employee evaluation. A. forced distribution B. forced choice C. BARS D. illegal

A. Perceptual speed tests are highly speeded and are comprised of very easy items that every examinee, it is assumed, could answer correctly with unlimited time. The best way to estimate the reliability of speed tests is to administer separately timed forms and correlate these, therefore using a test-retest or alternate forms coefficient would be the best way to assess the reliability of the test in this question. The other response choices are all methods for assessing internal consistency reliability. These are useful when a test is designed to measure a single characteristic, when the characteristic measured by the test fluctuates over time, or when scores are likely to be affected by repeated exposure to the test. However, they are not appropriate for assessing the reliability of speed tests because they tend to produce spuriously high coefficients.

A company wants its clerical employees to be very efficient, accurate and fast. Examinees are given a perceptual speed test on which they indicate whether two names are exactly identical or slightly different. The reliability of the test would be best assessed by: A. test-retest B. Cronbach's coefficient alpha C. split-half D. Kuder-Richardson Formula 20

B. Pooled variance is the weighted average variance for each group. They are "weighted" based on the number of subjects in each group. Use of a pooled variance assumes that the population variances are approximately the same, even though the sample variances differ. When the population variances were known to be equal or could be assumed to be equal, they might be labeled equal variances assumed, common variance or pooled variance. Equal variances not assumed or separate variances is appropriate for normally distributed individual values when the population variances are known to be unequal or cannot be assumed to be equal

A condition necessary for pooled variance is: A. unequal sample sizes B. equal sample sizes C. unequal covariances D. equal covariances

A. If you are not familiar with the term "tangentiality," thinking about the phrase "going off on a tangent" might have helped you. Tangentiality involves responding to a question in an oblique or irrelevant way and thereby failing to make the point. Circumstantiality is a pattern of speech that is delayed in reaching the point due to irrelevant detail or parenthetical remarks. Eventually, however, the point is reached

A difference between tangentiality and circumstantiality is that: A. in circumstantiality, the person is delayed in reaching the point, whereas in tangentiality, the person never reaches the point. B. circumstantiality is speech that is not understandable due to the lack of a meaningful connection between words or sentences; while tangential speech is understandable. C. in tangentiality, the person is delayed in reaching the point, whereas in circumstantiality, the person never reaches the point. D. tangentiality is speech that is not understandable due to the lack of a meaningful connection between words or sentences; while circumstantial speech is understandable.

D. According to the Forensic Specialty Guidelines, "Forensic psychologists avoid giving written or oral evidence about the psychological characteristics of particular individuals when they have not had an opportunity to conduct an examination of the individual..." And, according to Ethical Standard 9.08, psychologists also do not base their decisions on outdated results. Thus, the psychologist in this case should make a reasonable effort to reevaluate the person. However, it would probably be inappropriate to completely disregard the previous findings. The best approach would be to reevaluate the person and interpret the previous findings with caution

A forensic psychologist is asked to review and make conclusions regarding a defendant's psychological evaluation that was conducted 4 years ago following a similar crime that the defendant is now accused of. The psychologist should: A. refuse the request B. comply with the request but interpret the previous findings with caution C. reevaluate the person and disregard the previous findings D. reevaluate the person and interpret the previous findings with caution

D. The "telling style" according to Hersey and Blanchard's (1974) description, is more effective when an employee's ability and willingness to accept responsibility are low. This leadership style involves high task orientation and low relationship orientation. With A, the leader should adopt the "selling" style-high task orientation and high relationship orientation. With B, the leader should adopt a "participatory" style-low task orientation and high relationship orientation. With C the leader can use a "delegating style"; both low task and low relationship orientation.

A leader should adopt a "telling" style if: A. The employee has low ability but high willingness to accept responsibility B. The employee has high ability but low willingness to accept responsibility C. The employee's ability to and willingness to accept responsibility are both high D. The employee's ability and willingness to accept responsibility are both low

C. You would never PURSUE a romantic relationship with a former client if you are an ethical psychologist. Standard 10.08 (Sexual Intimacies With Former Therapy Clients/Patients) states, "(a) Psychologists do not engage in sexual intimacies with former clients/patients for at least two years after cessation or termination of therapy. (b) Psychologists do not engage in sexual intimacies with former clients/patients even after a two-year interval except in the most unusual circumstances. Psychologists who engage in such activity after the two years following cessation or termination of therapy and of having no sexual contact with the former client/patient bear the burden of demonstrating that there has been no exploitation, in light of all relevant factors, including (1) the amount of time that has passed since therapy terminated; (2) the nature, duration, and intensity of the therapy; (3) the circumstances of termination; (4) the client's/patient's personal history; (5) the client's/patient's current mental status; (6) the likelihood of adverse impact on the client/patient; and (7) any statements or actions made by the therapist during the course of therapy suggesting or inviting the possibility of a posttermination sexual or romantic relationship with the client/patient." Choices A and D also involve multiple relationships and are not good choices. However, APA has not dedicated a specific standard to either of these situations.

A former patient and yourself meet at the shopping mall 26 months after she has completed therapy with you. Which is the most unethical choice? A. You help her choose her fall wardrobe. B. She has started smoking again, so you refer her to a smoking cessation group. C. You pursue a romantic relationship with her. D. She is a physician; describe your current physical symptoms and ask her opinion.

B. Understanding what retroactive and proactive interference (or inhibition) are would help you answer this question. Retroactive interference occurs when a person's ability to recall X is difficult because of interference by something learned after X. In other words, new learning interferes with the recall of prior learning. Additionally, the longer the period of time between learning X and being tested on it, the greater the opportunity for retroactive interference. Proactive interference occurs when the ability to recall X is impaired by previously learned material. Proactive interference can occur regardless of how long the interval is between learning X and recalling it. Finally, retroactive and proactive interference are most likely to be a problem for information that is not inherently meaningful, which would be the case for a set of unrelated words

A group of undergraduate students learn a list of new material. Half of the students then spend the next 8 hours awake while the other half sleep before being tested on the material. Which of the following results would most likely be expected? A. the students that stay awake will remember more because they will have more opportunity for post event rehearsal B. the students that sleep will remember more because they will experience less retroactive interference. C. the students that sleep will remember more because they have less opportunity for memory decay D. the groups will perform similarly because the retention interval is the same for both.

A. Fiedler's Contingency Theory proposed that in terms of a leader's style and the favorableness of a situation, the latter was determined by the degree to which the leader could control and influence their subordinate. Fiedler described a leader's style by his or her scores on his Least Preferred Coworker Scale. A high LPC leader describes their least preferred coworker in positive terms and these leaders are primarily relationship oriented. Note that the question talks about how a leader "treats" their worker rather than how they "describe" the worker.. While these are not exactly the same concepts, the EPPP will take these type of liberties so this is an example of choosing an answer that is in the "ballpark"

A high LPC leader: A. Treats his least favorite worker well B. Treats his least favorite worker poorly C. Treats his favorite worker like his least favorite worker D. Has an ambivalent style toward his workers .

D. A hypomanic episode is characterized by a period of persistently elevated, expansive, or irritable mood that lasts at least 4 days and is accompanied by three or more symptoms including: inflated self-esteem or grandiosity; decreased need for sleep; talkativeness; racing thoughts; distractibility; excessive involvement in high-risk pleasurable activities; and increase in goal-directed activity. The symptoms are not severe enough to require hospitalization or cause marked impairment in functioning and there are no psychotic features. Many individuals exhibit increased productivity, efficiency, and creativity during hypomanic episodes

A hypomanic episode would most likely result in: A. increased productivity, decreased efficiency, and decreased creativity B. decreased productivity, decreased efficiency, and decreased creativity C. decreased productivity, decreased efficiency, and increased creativity D. increased productivity, increased efficiency, and increased creativity

B. The actor-observer effect is the tendency to overestimate situational factors and underestimate dispositional factors regarding one's own behaviors and to underestimate situational factors and overestimate dispositional factors in others. Self-serving bias (A) is the tendency to take credit for our successes and to blame situational factors for our failures. Choice C is not a type of attribution bias. Projective identification (D) is a defense mechanism in which one or more parts of the self are falsely attributed to another, which are then unconsciously accepted by the recipient, and the projector then identifies with the projected part in the other.

A job applicant tells his friend that he's nervous because of his upcoming job interview. When the applicant sees other nervous-looking job applicants in the waiting room before his interview, he believes that they are generally nervous people. This is an example of: A. self-serving bias B. actor-observer effect C. social comparison effect D. projective identification

C. The subject is most likely to rush to aid someone in distress when all alone in the lab; less likely when there is a stranger or a friend in the room; and least likely when there is a passive confederate in the same room who has been instructed to ignore the whole thing. There is no "diffusion of responsibility" or "pluralistic ignorance" when the subject is alone

A lab subject heard a woman in the next lab fall and cry for help. In this "lady in distress" experiment, a person was least likely to help when tested with: A. no others present. B. a stranger. C. a passive confederate. D. a friend.

B. According to Herzberg's two-factor theory, need satisfiers can be divided into two types: hygiene factors and motivators. The absence of hygiene factors (e.g., money, security) results in dissatisfaction; however, their presence does not lead to satisfaction. By contrast, the presence of motivators (e.g., responsibility, power, self-esteem) results in satisfaction; however, their absence does not lead to dissatisfaction. Therefore, to increase a worker's motivation, one would have to add factors such as responsibility to his or her job. Increased money or security will reduce any dissatisfaction, but will not increase motivation or satisfaction

A man reports that he is dissatisfied with his low-level job on an assembly line. He says that money is the only reason he is doing his work. According to Herzberg, what would be the best way to increase this person's job motivation? A. give him a greater number of jobs to do B. give him more responsibility C. give him more money D. give him a longer lunch break

D. Ethical Standard 5.01 states, "Psychologists do not knowingly make public statements that are false, deceptive, or fraudulent, concerning their research, practice, or other work activities or those of persons or organizations with which they are affiliated." Although in this case the psychologist is asked not to publish data, this would still be deceptive. It also is inconsistent with General Principle A of the Ethics Code which states that, "psychologists seek to safeguard the welfare and rights of those with whom they interact professionally and other affected persons.... Because psychologists' scientific and professional judgments and actions may affect the lives of others, they are alert to and guard against personal, financial, social, organizational, or political factors that might lead to misuse of their influence."

A managed care company asks a psychologist to conduct a study on the relative effectiveness of a 10-week therapy program for depression vs. a 20-week therapy program. The company plans to publish the results of the research if no significant difference is found but will not publish the results if the longer therapy program is more effective. The psychologist should: A. agree to conduct the study because it is within the company's rights to publish or not publish their research findings B. agree to conduct the study if the psychologist is able to approve any article submitted for publication C. agree to conduct the study only if all research participants are appropriately debriefed at the conclusion of the study D. refuse to conduct the study

B. McGregor's (1960) Theory X and Theory Y management theories reflect the key differences between the scientific management and human relations administrative models. Managers adopting a scientific management model assume that workers are motivated primarily by financial self-interest (i.e., pay is the most effective motivator); that job demands must match workers' skills; and that workers need detailed guidelines and constant supervision because they are incapable of regulating themselves or assuming responsibility on their own. Theory X is similar to scientific management: Theory X managers believe that workers dislike work and avoid it whenever possible; therefore, workers must be directed and controlled

A manager assumes that his employees need constant supervision and detailed guidelines in order to perform their jobs well. What management style does this reflect? A. Theory Y B. Theory X C. laissez-faire D. management by objectives

A. Ipsative scores report an examinee's scores using the examinee him or herself as a frame of reference. They indicate the relative strength of a score within an individual but, unlike normative measures, do not provide the absolute strength of a domain relative to a normative group. Examples of ipsative scores are the results of a forced choice measure.

A measure of relative strength of a score within an individual is referred to as a(n): A. ipsative score B. normative score C. standard score D. independent variable

A. Of the groups listed, never married men have the highest rates of admission to inpatient mental health facilities regardless of the type of facility (state/county versus private). The other groups listed have similar rates; but, if you have to choose the group with the lowest rate, the best answer is never married women (answer D).

A member of which of the following groups is most likely to be admitted to an inpatient mental health facility? A. never married men B. divorced/separated men C. married men D. never married women

D. Overcorrection is a behavioral technique used to reduce a behavior targeted for elimination and replace it with a more adaptive behavior. It involves two components: restitution and positive practice. Restitution, seen as a punitive phase, involves having the individual correct the consequences of the behavior. In positive practice, the person practices an alternative, appropriate behavior. In both phases, the person often is physically guided and performs the actions in an exaggerated manner. Overcorrection has typically been used to treat disruptive behaviors in developmentally disabled children.©

A mother disciplines a child who had a tantrum and threw the pieces of a board game all over the floor first by making the child pick up the all components of the game and carefully placing them back into the box into the exact place they belong. Then while physically guiding him, she has him take out another board game and carefully set up the game in the correct manner. The behavioral technique the mother has used is: A. negative reinforcement. B. differential reinforcement for other behaviors. C. shaping. D. overcorrection.

D. A needs assessment involves identifying the needs of the organization. A needs assessment is often conducted to determine which employees need training and what should be included in that training. Choice B describes the purpose of a job analysis, which is to describe the requirements of a job. And choice C describes the purpose of a job evaluation, which is to determine the salary levels of jobs.

A needs assessment is usually conducted to: A. acertain a job applicant's skills B. describe the requirements of a job C. determine the relative worth of a job D. identify the needs of an organization

B. You are obligated to protect your client's confidentiality. However you want to educate her about the Ethical Standards regarding this situation and also explain to her avenues of recourse she could take.

A new client tells you that she recently had sex with her previous therapist. She wants you to "help her put the experience behind her". You should: A. Report this situation immediately B. Not report, but counsel her about the process C. Call the offending party but not give the client's name D. Encourage her to recall the event so she doesn't suffer from repression later

D. A diagnosis of ADHD requires the presence of characteristic symptoms in at least two settings. Therefore, it is necessary to investigate the child's behavior in other settings, such as school. However, the clinician needs to be aware that the disorder's nature and severity can differ across different situations. For instance, symptoms typically worsen in situations that require sustained attention or mental effort, and they may be minimal or absent when the child is frequently rewarded, is under very strict control, or is in a novel setting.

A parent brings his 7-year-old child in for evaluation, with the complaint that the child can't sit still, is constantly on the go, and never pays attention. You suspect that the diagnosis might be Attention Deficit/Hyperactivity Disorder. Of the following, which would be the best way to confirm this diagnosis? A. administer Ritalin to the child for a limited period of time to determine if the symptoms respond to the medication B. investigate the parent's interaction with the child C. determine if the child displays the same exact behaviors in other settings D. investigate the child's behavior in different settings with the awareness that the disorder may manifest itself differently in different situations

D. A percentage score indicates the number of items answered correctly. A percentile rank compares one examinee's score with all other examinee's scores

A percentage score, as opposed to a percentile rank, is based on: A. Total number of items B. An examinee's score in comparison to other examinee's scores C. That there are one hundred test items D. The number of items answered correctly

A. feels happy

A peripheral message is more likely to alter a person's attitude if the receiver: A. feels happy B. feels bored C. feels angry D. devotes considerable thought to the content of the message

Functional fixedness

A person calls into a radio talk show psychic, hears the psychic's reading and says "that is exactly my situation." This is NOT an example of: A. subjective validation B. functional fixedness C. the Barnum effect D. the Forer effect

D. Although this question is referring to a former client, due to the potential danger for a suicidal client, the most appropriate action would be to meet with the client until the situation is stabilized and then refer.

A psychologist is planning to retire in the next few months and has been decreasing her caseload. One of the psychologist's former clients, who had terminated a few months earlier, phones the psychologist and reports having suicidal thoughts. The psychologist should: A. inform the client that she is retiring and that she cannot accept any clients B. refer the client to a colleague immediately C. meet with the client until the client no longer needs treatment D. meet with the client until the situation is stabilized and then refer

B. The type of memory needed to hold the directions in one's mind while working on it is called working memory. In the 1980s, Baddeley and Hitch coined the term "working memory" for the ability to hold several facts or thoughts in memory temporarily while solving a problem or performing a task. Baddeley defined the process of rapid verbal repetition of the to-be-remembered information to facilitate maintaining it in working memory, as an "articulatory loop." He found there is a neural system or central executive in the frontal portion of the brain responsible for processing information in the "working memory." Short-term memory (c.) holds information in mind for only a few seconds as it is processed. Long-term memory (d.) is where such processed information is permanently stored. Working memory is an intermediary and active memory system in the information processing area of the brain.

A person stops and asks a parking attendant for directions after getting lost while driving to an appointment. The attendant states, "Make a left at the first red light. Go four blocks until you reach a stop sign and turn right onto Main Street. About three blocks down Main, look for a large red sign at the entrance to the parking lot." What type of memory is needed to hold such information? A. procedural memory B. working memory C. short term memory D. long term memory

fundamental attribution error (answer)

A person who has seen a co-worker of his get annoyed when others interrupt his work is surprised to see him behaving in a friendly manner at an office holiday party. The person's surprise is best explained by: the actor-observer effect. the fundamental attribution error. confirmation bias. cognitive dissonance

C. A predictor that is highly sensitive will more likely identify the presence of a characteristic; that is, it will result in more positives (true and false). This may be desirable when the risk of not detecting a problem is high. For example, in the detection of cancer, a blood test that results in a high number of false positives is preferable to one that has many false negatives. A positive test result can then be verified by another method, for example, a biopsy. Measurement error (A) is the part of test scores which is due to random factors. Type II error (B) is an error made when an experimenter erroneously accepts the null hypothesis

A predictor that is highly sensitive for identifying the presence of a disorder would most likely result in: A. measurement error B. type II error C. a high number of false positives D. a high number of false negatives

B. This question looks like one on confidentiality, and, indeed, there are some aspects of confidentiality embedded here. But, more than that, it's about professional relationships. You're told in the item stem that all consents have been obtained, so the confidentiality issue was already addressed. The question then is, "What should the psychologist do?" Since we're all here to help the patient, the most appropriate thing to do is to let the pediatrician know about your findings. You would call (probably call rather than write since calling is quicker) the pediatrician and report your findings. It was the pediatrician who made the referral in the first place, and you have the parents' consent.

A psychologist is consulted by the parents of a child who was referred by a pediatrician for evaluation. During the interview with the parents, they report the girl suffered a severe head injury which they haven't told the pediatrician about. Assuming appropriate consents have been obtained, the psychologist: A. should have the parents report this to the pediatrician. B. should report this to the pediatrician. C. should not report this until the parents tell the pediatrician first. D. should keep this information to herself.

D. An expert witness by definition is supposed to clarify issues of fact in a trial and is not supposed to act as an advocate for one side or the other, even if he is hired by one side. For this reason, APA's Specialty Guidelines for Forensic Psychologist explicitly state that psychologists do not provide services to parties in a legal proceeding on a contingent fee basis when those services involve expert testimony. Although the Specialty Guidelines are aspirational rather than binding, APA's Ethical Principles and Code of Conduct, which is binding, contains standards which clearly apply to this situation, such as the prohibition on assuming roles that involve a conflict of interest

A psychologist is hired by the defendant's attorney to serve as an expert witness in a criminal case. The arrangement includes a bonus fee for the psychologist if the client is found not guilty. This arrangement is A. ethical. B. ethical as long as it is disclosed to the court. C. ethical as long as the fee is not excessive. D. unethical.

C. According to Standard 3.11 (Psychological Services Delivered To or Through Organizations), "Psychologists delivering services to or through organizations provide information beforehand to clients and when appropriate those directly affected by the services about (1) the nature and objectives of the services, (2) the intended recipients, (3) which of the individuals are clients, (4) the relationship the psychologist will have with each person and the organization, (5) the probable uses of services provided and information obtained, (6) who will have access to the information, and (7) limits of confidentiality. As soon as feasible, they provide information about the results and conclusions of such services to appropriate persons." If you choose answer A and leave abruptly, you are abandoning your client. You may also want to do answer B—outline your points of dissatisfaction—but this wouldn't be your first or most important consideration

A psychologist is working for a company that changes health care providers due to cost-effectiveness. The new provider has a policy that compromises the employee's future limits to confidentiality. The psychologist should: A. refuse to follow the new procedures and leave immediately. B. outline her points of dissatisfaction and send it to the new provider. C. make clear with each client the company's new limits to confidentiality. D. organize a strike and call CNN.

A. Certain behaviors associated with Bulimia, specifically frequent vomiting and laxative use, can lead to serious medical complications including electrolyte disturbances. Potassium is an electrolyte that is important for the function of the nerves and muscles. Hypokalemia is the name given to the condition that involves low levels of serum potassium that can lead to kidney failure or cardiac arrest. Hypoglycemia is characterized by less than normal amounts of glucose in the blood. Hyperorexia refers to excessive appetite and hypalgesia involves a relative insensitivity to pain

A psychologist refers a client with Bulimia Nervosa to a physician for a medical evaluation. The physician discovers that the client has a serious medical complication as the result of frequent binge eating and purging. This condition involves a low level of serum potassium and increases the client's risk for kidney failure and cardiac arrest. This condition is called: A. hypokalemia B. hyperorexia C. hypalgesia D. hypoglycemia

D. The term "capitation" refers to a fixed amount of money paid per person, not by the visit or procedure. Managed care companies usually express capitation in terms of cost per member per month. As stated in response "D," providers are paid a specific dollar amount, for a specific time period, to cover the service needs of a specific number of people. If a provider exceeds his or her capitated payment, he or she may not be able to cover his or her costs and, consequently, may limit his or her services.

A psychologist sees clients at a facility where fees are capitated. What does this mean? A. providers are paid a fee each time a service is performed B. clients are billed according to their ability to pay C. clients are required to meet a deductible before their insurance will pay D. providers receive a fixed dollar amount over a specific period of time to cover the service needs of a fixed number of clients

A. In many states, it is illegal to provide assessment or treatment to a child in this situation. Even in states where the non-custodial parent has the right to consent to his or her minor child's psychological treatment, it is not considered ethical to begin treatment (except of course in emergency situations) without the custodial parent's consent. This is because the child's best interests are threatened by such an arrangement.

A psychologist tests a child from a home in which the parents were divorced at the request of the noncustodial parent. The custodial parent complains that the psychologist acted without her permission. According to accepted professional standards of practice, the psychologist acted: A. unethically because she failed to obtain proper consent. B. ethically but with poor judgment. C. ethically because the noncustodial parent still retains this right. D. unethically because she first should have consulted a lawyer.

B. Psychologists and other health professionals have no legal or ethical obligation to enter into a professional relationship. Though they cannot discriminate on the basis of race, gender, sexual orientation, or any basis proscribed by law, they can refuse to enter into a relationship based on a client's diagnosis -- even if they are competent to treat individuals with that diagnosis and even if their reasons for doing so are not valid. However, if they do refuse to take a case, psychologists are required to make appropriate referrals.

A psychologist who is very fearful of getting sued decides for this reason that he will not treat any patients with a diagnosis of Borderline Personality Disorder. The psychologist's actions are: A. unethical, assuming that the psychologist is competent to treat such patients. B. ethical as long as appropriate referrals are provided. C. ethical regardless of whether appropriate referrals are made. D. unethical, because it is inappropriate to consider the possibility of getting sued in making any clinical or treatment decision.

D. This answer is most consistent with the requirements of the Ethics Code and the Specialty Guidelines for Forensic Psychologists. For example, Standard 9.06 of the Code requires that psychologists "indicate any significant limitations of their interpretations." Although recommendations should be based on valid assessment techniques, those techniques do not have to be standardized tests (answer C).

A psychologist working in the prison system is asked to evaluate a prisoner and make a recommendation about his eligibility for parole. The psychologist should: A. refuse to do so since this is outside the scope of a psychologist's work. B. do the evaluation and report its results but refuse to make a specific recommendation. C. make a recommendation as long as it is based on the results of valid standardized tests. D. make a recommendation but point out any possible limitations of the data on which it is based.

A. This question relates to a psychotherapist's "duty to protect" the victim when a client threatens violence against an identifiable victim. In two rulings in the case of Tarasoff v. Regents of the University of California (1974 and 1976), the California Supreme Court ruled that, when a psychotherapy patient makes a specific threat of violence against a reasonably identifiable victim, the therapist must make reasonable efforts to protect the victim. Though this requirement is commonly called the "duty to warn", warning the victim is one of a number of possible ways to discharge this duty. Others include calling the police and hospitalizing the client and of course these steps can be combined. Though Tarasoff was a California case, many other states have adopted laws with substantially similar language, and the duty to protect has evolved into a standard of care in mental health fields. The duty to protect only applies when the client makes a specific, foreseeable, and believable threat of violence; when the victim or victims are identifiable; and when therapist intervention is feasible. In this case, there is no specific threat and no identifiable victim. Therapists are not required to pry threats of violence out of clients. Therefore, of the choices listed, the best option would be to maintain the client's confidentiality

A psychologist works with a client who is a reputed organized crime boss. The psychologist suspects that that the client has been and will be involved with acts of violence, but the client does not mention the name of any people he intends to harm. In this situation, the psychologist should: A. maintain the client's confidentiality. B. warn the police of the danger to others posed by the client. C. attempt to obtain the name of future victims via indirect questioning. D. explore the childhood roots of the client's behavior.

C. The Ethic Committee's procedures are outlined in APA's Rules and Procedures. When the Committee decides that it is appropriate to pursue a formal case investigation, it sends a charge letter to the complainee. The complainee then has thirty days to file an initial response. A request for independent adjudication or formal hearing (answer D) is not made until the Committee has actually found the psychologist to be guilty of an ethical violation

A psychologist, who is a member of APA, receives a charge letter from the Ethics Committee. The charge against the psychologist is that he cheated on the psychology-licensing exam. This means that the psychologist: A. will be barred from further consideration for licensure. B. will have to retake the exam under supervision. C. must respond to the charge letter within thirty days. D. can request independent adjudication or a formal hearing.

D. APA's 2002 Ethics Standard 7.05 permits undergraduate and graduate programs to have individual or group therapy as a program or course requirement. However, the students must be allowed "the option of selecting such therapy from practitioners unaffiliated with the program." Contrary to C, programs are not prohibited from approving credentials of the outside therapist

A psychology program requires its students to attend group therapy. This is permissible if: A. the therapy is only provided by the program's faculty B. the students are given the option of obtaining individual therapy in lieu of group therapy C. the program does not require approval of the therapist's credentials D. the students are given the option of receiving therapy from therapists not affiliated with the program

B. The rational-economic model assumes that decision-making involves a rational process in which all alternatives are considered and the best possible decision is eventually made. The model requires that decision-makers obtain information about and consider all possible alternatives before making a decision. This requirement places an inordinate demand on the people involved. Thus, in the real world, decisions are usually made based on the information that is available given restraints of resources such as time, money, and personnel

A reason why the rational-economic model of decision-making usually does not prove viable in an organization is that: A. it allows for the consideration of a limited number of options. B. it places too many demands on the individual and the organization. C. it arouses resentment in organization members who are excluded from the decision-making process. D. humans are irrational and emotional by their nature.

C. Triangulation is the attempt to increase reliability by reducing systematic or method error through a strategy in which the researcher employs multiple methods of measurement (e.g., observation, survey, archival data). If the alternative methods do not share the same source of systematic error, examination of data from the alternative methods gives insight into how individual scores may be adjusted to come closer to reflecting true scores, thereby increasing reliability. Calibration (a.) is the attempt to increase reliability by increasing homogeneity of ratings through feedback to the raters, when multiple raters are used. For example, raters might meet during pretesting of the instrument to discuss items on which they have disagreed seeking to reach consensus on rules for rating items (e.g.., defining a "2" for an item dealing with job performance). Intraclass correlation (ICC) (b.) is used to measure inter-rater reliability for two or more raters and may also be used to assess test-retest reliability. ICC may be conceptualized as the ratio of between-groups variance to total variance. Correction for attenuation (d.) is a method used to adjust correlation coefficients upward because of errors of measurement when two measured variables are correlated; the errors always serve to lower the correlation coefficient as compared with what it would have been if the measurement of the two variables had been perfectly reliable.

A researcher employs multiple methods of measurement in an attempt to increase reliability by reducing systematic error. This strategy is referred to as: A. calibration B. intraclass correlation (ICC) C. triangulation D. correction for attenuation

C. One of the most stable findings in job satisfaction research is that age is positively correlated with satisfaction. The older the employee, the higher the level of satisfaction he or she tends to report.

A researcher studying the relationship between age and job satisfaction collects data for a group of young, middle-aged, and older workers. The researcher can expect to find that: A. the young people are most satisfied with their jobs. B. the middle-aged people are most satisfied with their jobs. C. the older people are most satisfied with their jobs. D. there is no difference between the three groups in terms of their job satisfaction.

D. Information on a test's incremental validity for differing combinations of base rates, selection ratios, and validity coefficients is provided by the Taylor-Russell tables. Decision-making accuracy can improve on a test with a low or moderate validity coefficient when the selection ratio is low (e.g., .05) and the base rate is moderate (near .50).

A selection test that has a validity coefficient of .50 will have the greatest incremental validity when: A. the selection ratio is .95 and the base rate is .20 B. the selection ratio is .95 and the base rate is .50 C. the selection ratio is .05 and the base rate is .20 D. the selection ratio is .05 and the base rate is .50

C. The Pygmalion Effect

A self-fulfilling prophecy where subordinates perform better when expected to do so by their superiors is known as A. the Barnum effect B. Unconscious inference C. Pygmalion effect D. Tend-and-befriend response

D. According to cognitive dissonance theory, simultaneously holding two contradictory ideas or cognitions causes an uncomfortable feeling known as cognitive dissonance, and people have a drive to reduce this dissonance by changing one of the cognitions. This theory has been used to explain results of research in which subjects were induced to behave in a manner contradictory to their stated attitudes. For instance, in an early study on this phenomenon, subjects were instructed to write a counter-attitudinal essay and were paid either $20 or $1 for doing so. Afterwards, subjects in the $1 group displayed significant attitude change in the direction of the opinions in their essays, whereas subjects in the $20 showed no attitude change. According to the researchers, the $1 group experienced dissonance between two cognitions--"I disagree with this point of view" and "I wrote an essay espousing this point of view." Therefore, to reduce dissonance, subjects had to change their attitude to support the viewpoints of the essay. The $20 group did not experience dissonance or attitude change because the external incentive of money could justify their behavior. The question describes a situation that appears similar to the above experiment. From the perspective of the theory, the individuals may have two conflicting cognitions "As a leftist, I don't like large corporations" and "I work for a large corporation." Therefore, they may be motivated to change their attitude in a positive direction towards corporations.

A study shows that individuals with left-leaning or socialist political outlooks adopt more positive viewpoints towards large corporations after accepting a low paying job in one. This study provides evidence for: A. Lewin's field theory. B. Freud's structural theory. C. the James-Lange theory of emotion. D. cognitive dissonance theory.

A. In the terminology of program evaluation, a common distinction is between summative and formative evaluation. Summative evaluations deal with whether a program has achieved its intended objectives. They are typically used to address "bottom line" issues, such as if a program should continue existing, or if more or fewer resources should be allocated to it. By contrast, a formative evaluation addresses issues related to the implementation of a program. The main focus of a formative evaluation is how a program can be improved. In practice, most program evaluations contain elements of both formative and summative evaluation

A summative evaluation of a mental health program primarily deals with which of the following questions? A. "Did the program meet its goals?" B. "How can the program be improved in the future?" C. "How can the strengths and weaknesses of the program be summarized?" D. "What is preventing the program from operating more effectively right now?"

B. Parallel process is a phenomenon in clinical supervision where the therapist in training behaves toward the supervising therapist in ways that mirror how the client is behaving toward the therapist in training

A supervisee begins to unconsciously act toward her supervisor in similar ways as her own client acts towards her. This is an example of: A. Projective identification B. Parallel process C. Transference D. Identification.

B. In behavioral terms, the term "positive" means that a stimulus is applied (not, as implied by choice C, that the stimulus is pleasurable or enjoyable). In this case, the stimulus being applied is the teacher's yelling. And punishment means that the stimulus has the effect of reducing a behavior. In this case, the yelling has the effect of temporarily reducing the boy's disruptiveness, so it is punishment. The question illustrates a disadvantage of punishment -- the recipient becomes habituated (which just means he or she gets used to it) and it is necessary to keep increasing the intensity of the punishment for it to be effective.

A teacher finds out that if she yells at a disruptive boy in class, he will calm down for a few minutes. Over time, her yelling becomes louder and more frequent. Which of the following statements best describes, in behavioral terms, what is happening in this situation? A. The teacher's yelling is serving as positive reinforcement for the boy, which is why he is becoming more and more disruptive over time. B. The teacher's yelling is serving as positive punishment, but the boy is becoming habituated over time. C. The teacher's yelling is probably serving as negative punishment for her, because she likely finds the situation to be aversive; however, it is serving as positive punishment for the boy, because he seems to enjoy being yelled at. D. The teacher's yelling is serving as positive reinforcement for herself, as she seems to enjoy yelling.

B. Differential reinforcement of other behaviors (or DRO) involves administering reinforcement if a person does not engage in behavior targeted for elimination during a designated time period. It is commonly used with mentally retarded, autistic, and ADHD children.

A teacher in a class that includes a 9-year-old boy with ADHD has a timer at her desk and sets it to go off every 15 minutes. If the boy has not interrupted the class during that 15-minute time period, he receives a reward. The technique being used in this instance is known as: A. operant extinction. B. differential reinforcement of other behaviors. C. the Premack Principle. D. negative reinforcement.

D. A test with limited ceiling has an inadequate number of difficult items resulting in few low scores. Therefore the distribution would be negatively skewed

A test with limited ceiling would have a ____________ distribution shape. A. normal B. flat C. positively skewed D. negatively skewed

C. According to social loafing theory, people "loaf" (don't do their share) when they are participating in a large group and when their contribution, or lack of contribution, will not be detected

A tour guide is likely to get the smallest tip from a tour group when the group is large and the tourists contribute to a group tip. This is predicted by: A. social inhibition theory B. social comparison theory C. social loafing theory D. social exchange theory

B. The International Headache Society (IHS) has developed a headache classification system that divides headaches into thirteen types -- four of which are listed by the choices here. This question best matches the description of a migraine headache. Migraines are felt on one side of the head by a majority of sufferers and are typically throbbing in nature. In addition, nausea (with or without vomiting) as well as sensitivity to light and sound often accompany migraines

A unilateral, severe, and throbbing headache that involves nausea and sensitivity to light is most likely a: A. tension headache. B. migraine headache. C. cranial neuralgia. D. cluster headache.

D. Social Anxiety Disorder is characterized by marked and persistent fear of one or more social or performance situations in which the person may be exposed to scrutiny by others and the person fears he or she will act in way or exhibit anxiety symptoms that will be negatively evaluated.

A woman is offered a substantial raise and promotion in her job, but the new position will require her to periodically speak at conferences. She would very much like to accept the new position but fears speaking in public due to her history of panic attacks in the past while doing so. She is concerned that people will think she is incompetent if she has an attack while speaking. The most likely diagnosis is: A. Generalized Anxiety Disorder B. Agoraphobia. C. Specific Phobia D. Social Anxiety Disorder.

B. Aaron Beck, the founder of cognitive therapy, identified a number of cognitive distortions, or errors in logic and interpretation that contribute to negative automatic thoughts and therefore to depression. One of these, personalization, involves relating external events to oneself even when there is no logical basis to do so. In this case, the woman is relating the husband's comments about someone else to herself when there appears to be no logical reason to do so. Therefore, personalization is the best choice here. Regarding the other choices, selective abstraction occurs when one detail is taken out of context and used to draw a conclusion while ignoring other more salient features of a situation. Overgeneralization refers to drawing a general rule or conclusion based on one or a few isolated cases and applying the concept to other unrelated situations. And dichotomous thinking involves categorizing all experiences into one of two opposite categories (e.g., good-bad, competent-incompetent).

A woman whose husband tells her that a woman he knows at work is not bad looking replies "you think I'm ugly." From the perspective of cognitive therapy, this statement can be labeled as: A. a selective abstraction. B. a personalization C. an overgeneralization D. dichotomous thinking

D. Autism Spectrum Disorder has been linked to abnormalities in the cerebellum (which is involved in motor activity and coordination), amygdala (which plays a role in emotions and social behaviors), and corpus callosum (which facilitates communication between the right and left hemispheres

Abnormalities in several areas of the brain have been identified as contributors to Autism Spectrum Disorder including a smaller-than-normal: A. medulla. B. thalamus. C. hypothalamus. D. cerebellum.

C. According to APA's Record Keeping Guidelines, client "records may be maintained in a variety of media, so long as their utility, confidentiality and durability are assured" (1993, 48 (9), 984-986).

According to APA's Record Keeping Guidelines, client records: A. may not be maintained in electronic media B. may not be exclusively maintained in electronic media C. may be maintained in a variety of media D. may be maintained in electronic media if write-protected software is used which prevents changes from being made after initial data storage.

D. To reason this one out, you don't really have to be familiar with the language of the applicable standard, but here it is anyway: "Those responsible for testing programs should provide appropriate interpretations when test score information is released to students, parents, legal representatives, teachers, or the media. The interpretations should describe in simple language what the test covers, what scores mean, common misinterpretations of test scores, and how scores will be used" (standard 15.10). Other standards require that the standard error of measurement be reported so confidence intervals or percentile bands (choice C) can be constructed; however, the word "only" makes C incorrect

According to APA's Standards for Educational and Psychological Testing, test results should be reported: A. only to professionals trained in the interpretation of psychological tests. B. using only raw scores. C. using only confidence intervals or percentile bands. D. in clear and simple language.

C. Under the Cleary model, a test is considered unfair if the slope and/or the y-intercept of the regression line is different for one subgroup than for another. The effect of these statistical phenomena is that differences between subgroups on predictor scores would not be reflective of differences between the groups on the criterion. For instance, low scorers in one subgroup might do just as well on the criterion as high scorers in the other subgroup

According to Anne Cleary's model of test fairness, a job selection test would be considered unfair if: A. Based on the use of the test, a higher proportion of Caucasians than African-Americans are chosen for the job. B. the test has a higher validity coefficient for Caucasians than for African-Americans. C. the slope of the test's regression line is different for African-Americans than for Caucasians. D. the content of the test is culturally biased.

D. Bass (1985) suggested that transformational leadership contains the interrelated components of idealized or charisma influence (attributed or behavioral), inspirational motivation, intellectual stimulation, and individualized consideration. In truly transformational leadership, high morals and ethical standards characterize charismatic or idealized influence. In research by Podsakoff, MacKenzie, Moorman and Fetter (1990), trust was found to be the single most important variable moderating the effects of transformational leadership on the performance, attitudes, and satisfaction of the followers. Inspirational motivation (b.) provides followers with meaning and challenges for engaging in undertakings and shared goals. Intellectual stimulation (a.) helps followers to question assumptions and to generate more creative solutions to problems. Individual consideration (c.) treats each follower as an individual and provides coaching, mentoring and growth opportunities.

According to Bass, transformational leadership contains four interrelated components. Which of the following is characterized by high moral and ethical standards? A. intellectual stimulation B. inspirational motivation C. individualized consideration D. idealized influence

B. A person's career anchor is his or her self-concept consisting of self-perceived talents and abilities, basic values, motives, and needs as they pertain to the career. Schein says that people are primarily motivated by one of eight anchors — priorities that define how they see themselves and how they see their work. The eight anchors include: technical/functional competence - a desire to excel in a chosen line of work; general managerial competence - closely allied with the traditional career path of the corporation; autonomy/independence - individuals just want to be alone; security/stability - value predictable environment in which tasks and policies are clearly codified and defined; entrepreneurial creativity - desire to create something of own and run it; service/dedication to a cause - the need to focus work around a specific set of values; pure challenge - seek to solve or master challenges; and lifestyle - organized around an individual's private life.

According to Edgar Schein, the concept of career anchor refers to: A. the tendency to stay in a financially secure position regardless of personal interest B. the motivation of priorities that define how people see themselves and their work C. the motivation of advancement opportunities to stay with an organization D. the tendency to stay in a position regardless of advancement opportunities

C. Although Holland is probably most associated with interests, his theory proposes that interests are actually determined by personality. In addition, it proposes that workers will be happy and most productive when their personality matches the characteristics of the work environment.

According to Holland, career choice is optimal when there is a good "fit" between: A. interests and job demands. B. self-concept and the job requirements. C. personality type and the job environment. D. values and the intrinsic meaning of the job.

A. Ellen Berscheid's Emotion-in-Relationships Model proposes that positive and negative emotions are most likely to occur in a relationship when the partner's behavior interrupts the individual's typical on-going behaviors. Thus, when things are running smoothly, there are fewer interruptions and less intense emotions. Although the partners are highly interdependent during this period, they are also more likely to underestimate their emotional investment. Choice C reflects the opposite of Berscheid's model because there are more surprises or interruptions to the status quo in the early stage of a relationship -- which results in the most positive (and negative) emotions. Choice D is incorrect because Berscheid suggests that partners (and researchers) tend to underestimate the importance of external situational factors in a relationship

According to Ellen Berscheid's Emotion-in-Relationships Model partners in long-term relationships are most likely to: A. underestimate their emotional investment in the relationship when things are running smoothly B. overestimate their emotional investment in the relationship when things are running smoothly C. experience the most intense positive emotions after several years into the relationship D. focus on attributions which are external to their partners and themselves to understand their relationship

B. Herbert Simon earned a Nobel Prize in economics, but was also known for his theories in psychology, computer science (artificial intelligence), and administration. The one thread through all of his work was his interest in decision-making and problem solving. His "administrative" model maintains that decision-makers cannot always afford to be rational, instead, they must choose the first solution that is minimally acceptable or "satisficing." This is in contrast to another decision making theory, the "rational-economic model" in which decision-makers attempt to find the optimal solution to a problem

According to Herbert Simon's administrative model, decision-makers in organizational settings typically chose the: A. least expensive choice B. most "satisficing" choice C. best choice after weighing all the alternatives D. worst choice after weighing all the alternatives

D. The different leadership styles defined by Hersey and Blanchard involve different combinations of task and relationship orientation. A high task, low relationship orientation defines the telling leadership style. Answer A is a delegating leader; B is a participating leader, and C is a selling leader

According to Hersey and Blanchard (1974), a "telling" leader is most effective when employees are low in both ability and willingness to assume responsibility. A telling leadership style is characterized by a: A. low task and low relationship orientation. B. low task and high relationship orientation. C. high task and high relationship orientation. D. high task and low relationship orientation.

D. Hershey and Blanchard's Situational Leadership Theory holds that there are four leadership styles, and leaders should be flexible enough to adopt the one that applies best to the situation, based on the developmental or maturity level of the followers. At the lowest maturity levels, where followers cannot do the job and are unwilling or afraid to try, the leader should adopt a highly directive style called Telling / Directing. At the next lowest level, followers have variable competence, and are willing to do the job; the appropriate leadership style in this case is Selling / Coaching. Workers at the second highest developmental level can do the job but are uncommitted or unmotivated; here, a Participating / Supporting style is recommended. Finally, the least directive style of leadership, Delegating / Observing, is recommended for workers at the highest maturity levels--those who are competent and motivated to perform.©

According to Hershey and Blanchard's Situational Leadership Theory, the element of the situation that is most relevant to the ideal leadership style is: A. task type B. task difficulty C. leader personality D. follower developmental level

B. Hershey and Blanchard's Situational Leadership Theory holds that there is no one correct leadership style across situations; depending on the situation, the leader must respond with the correct task and relationship behavior or orientation. Leadership behaviors are classified as either high or low in task orientation or relationship orientation, resulting in four possible leadership styles. The correct style depends on the maturity level of subordinates. At the lowest maturity levels, where followers cannot do the job and are unwilling or afraid to try, the leader should adopt a high task/low relationship style called Telling / Directing. At the next lowest level, followers have variable competence, and are willing to do the job; the appropriate leadership orientation in this case is high task/high relationship, or Selling / Coaching. Workers at the second highest developmental level can do the job but are uncommitted or unmotivated; here, a low task/high relationship style, Participating / Supporting, is recommended. Finally, the least directive style of leadership, the low task/low relationship oriented style called Delegating / Observing, is recommended for workers at the highest maturity levels - those who are competent and motivated to perform.

According to Hershey and Blanchard's theory of situational leadership, a leader must respond to a particular work situation with the appropriate A. task and subordinate orientation. B. task and relationship orientation. C. subordinate and relationship orientation. D. task and temporal orientation.

A. Herzberg's two-factor theory divides job elements into two types: hygiene factors and motivators. Hygiene factors result in job dissatisfaction when absent, but do not lead to job satisfaction when present. Motivators result in job satisfaction when present, but do not lead to dissatisfaction when absent. Of the choices listed, only job content is a motivator. According to the theory, motivators are intrinsic to the work (i.e., they have to do with the job itself), while hygiene factors are extrinsic to the work. The other three choices are not directly related to the job itself and are classified by Herzberg as hygiene factors

According to Herzberg's two-factor theory, an employee who is happy with which of the following job elements will most likely be satisfied with her job? A. job content B. relationships with coworkers C. pay D. job security

C. As its name implies, path-goal theory predicts that leaders will be most successful when they show followers the path for achieving goals. Path-goal theory is also a contingency theory, which means that it proposes that the best leadership style depends on certain characteristics of the situation.

According to House's path-goal theory, the optimal leader style: A. is a democratic one that allows workers to participate in setting goals and identifying ways for achieving them. B. is the one that emphasizes a task-oriented (versus person-oriented) approach that focuses on ways to achieve goals. C. varies depending on the situation but always involves helping workers achieve their goals. D. varies depending on the leader's experience and personality but always focuses on ensuring that goals are consistent with workers' skills and knowledge.

D. In Seligman's theory of learned optimism, attributions of optimistic people are believed to be the opposite of attributions of depressed people. Since depressed people make internal, stable, and global attributions to negative events, optimistic people would tend to make external, unstable, and specific attributions in response to negative events. Therefore, we can readily eliminate "B" ("I didn't study enough") since that's an internal attribution. Choice "C" ("the teacher is always a tough grader") is a stable attribution. That leaves Choices "A" ("I was unlucky") and Choice "D" ("the test was hard this time") - which are both external and unstable attributions. Of the two, however, Choice "D" is better since being unlucky would imply that success is a matter of luck

According to M. Seligman's theory of learned optimism, a student with an optimistic attribution style who fails an exam in a class which he usually does well in is most likely to say: A. "I was unlucky" B. "I didn't study enough" C. "the teacher is always a tough grader" D. "the test was hard this time"

C. Rollins and Feldman (1970) found marital satisfaction tends to be greatest at the beginning and at the end of the family life cycle, in other words, a U-shaped relationship between length of marriage and satisfaction. The arrival of children or adolescents in the family was associated with increased dissatisfaction, but children leaving home and being of retirement age were associated with increased satisfaction. However, more recent evidence suggests that, on average, marital satisfaction may not follow a U-shaped pattern but instead decreases markedly over the first ten years and then drops more gradually over time

According to Rollins and Feldman, marital satisfaction across the lifespan: A. follows a linear pattern of gradually increasing satisfaction over time B. follows a linear pattern of gradually decreasing satisfaction over time C. follows a U-shaped pattern of gradually decreasing satisfaction over the first decade then gradually increasing over time. D. does not follow a consistent pattern of satisfaction over time.

A. Self Verification theory proposes that people need and seek confirmation of their self-concept, regardless of whether their self-concept is positive or negative. Thus, people prefer to be right rather than happy. According to this theory, a person who dances poorly would prefer to be told so (assuming the other's evaluation matches the person's self evaluation).

According to Self Verification Theory, a person who believes that he dances poorly would prefer to be told by friends: A. "You are a pretty bad dancer" B. "You are not a bad dancer" C. "You should become a professional dancer" D. nothing about dancing

A. The suicide rates for whites and African-Americans have both increased in recent years. However, the highest rates for white males are among the elderly; for African-Americans, the highest rates are among those aged 25 to 44

Among white males, the highest suicide rates are among those aged 75 and older. Among African-American males, the rates are highest for those: A. 25 to 44. B. 45 to 64. C. 65 to 74. D. 75 and above.

B. According to Social Judgment Theory, a person categorizes new information or positions into one of three zones or latitudes: latitude of acceptance, latitude of non-commitment, and latitude of rejection. The latitude of acceptance contains all the positions on a particular topic that a person finds acceptable. Within this latitude is the "anchor" (c.) which is the single position that a person finds the most acceptable. The latitude of non-commitment contains the positions that are neutral for the person. The latitude of rejection contains all the positions on an issue that a person rejects. The theory claims persuasion is a difficult process and cannot occur in circumstances such as: new information is judged to fall within the latitude of rejection; a person is ego-involved (b.) in the issue causing the latitude of rejection to be larger than usual and making persuasion even more difficult; and, when people tend to distort new information through assimilation and contrast diluting the persuasive potential of new information. When distortions like this occur, no persuasion will result if the new information falls within the latitude of acceptance and is close to the anchor position as the person will "assimilate" or pull the new position closer and make it seem like it is already accepted or more acceptable than it really is. However, if the new position is out of the latitude of acceptance, then a person will "contrast" or push the information farther away to the latitude of rejection and make it seem worse than it really is

According to Social Judgment Theory, persuasion occurs when all of the following happen EXCEPT: A. the new information is judged to fall in the latitude of acceptance B. the new information is an ego-involved issue C. the new information is different from the anchor position D. the new information, while discrepant from the anchor, isn't assimilated or contrasted

B. Weiner's attribution theory has to do with reasons people use to explain causes of behavior, events, and outcomes. According to Weiner, these explanations or attributions have three dimensions: locus-of control (internal-external), stability (stable-unstable), and controllability (controllable-uncontrollable). This makes for eight possible attribution types: internal-stable-controllable, internal-stable-uncontrollable, and so on. The focus of this theory is on attributions for achievement, and Weiner identified ability, effort, task difficulty, and luck and the most important explanations of achievement. The theory has a number of applications in a variety of settings. For example, students who attribute success to ability, an internal, stable, and uncontrollable factor, are likely to have higher self-esteem. Students who attribute academic successes and failure to effort (an internal, unstable, controllable factor) are more likely to persist at tasks.

According to Weiner's attribution theory, a person with high-self esteem is most likely to attribute success on an achievement-related task to: A. effort. B. ability. C. luck. D. the easiness of the task.

A. According to Bandura's social learning theory (or theory of observational learning), it is possible to learn a given behavior merely by watching a model perform it. Behavioral practice and reinforcement, though they influence the probability that a behavior will be learned, are not absolutely necessary for the behavior to be imitated. Therefore, Choices C and D, which involve reinforcement procedures, can be eliminated. Vestibule training (B) involves a simulation of work conditions, although it does not necessarily involve modeling

According to social learning theory, job training would be most effective when using: A. behavioral modeling B. vestibule training C. reinforcement on a variable interval schedule D. reinforcers tailored to each employee's needs

D. In its description of the development and course of Schizophrenia, the DSM-5 states that "the peak onset for the first psychotic episode is in the early- to mid-20s for males and in the late-20s for females"

According to the DSM-5, the peak age of onset of Schizophrenia is __________ for males and __________ for females. A. mid- to late-20s; mid-20s B. late teens; mid- to late-20s C. early-20s; early-30s D. early- to mid-20s; late-20s

A. The DSM-5 reports prevalence rates of ADHD in most cultures of about 5% for children and 2.5% for adults, with a male to female gender ratio of about 2:1 for children and 1.6:1 for adults.

According to the DSM-5, the prevalence rate of ADHD for children is approximately ____ percent. A. 5 B. 2.5 C. 10 D. 18

A. According to the EEOC's Uniform Guidelines on Employee Selection Criteria, disparate impact (also known as adverse impact) occurs if members of a protected class are selected at a rate less than 4/5ths, or 80% of the selection rate of another group. For example, if 2 out of every 10, or 20%, of African American applicants are hired, and 3 out of 10, or 30%, whites are hired, the ratio of the selection rates for the two groups would be 20/30, or 66%. This would indicate adverse impact against African-Americans in the hiring practice. Note that since this guideline was published, legal standards such as the Civil Rights Act of 1991 have held that failure to adhere to the 80% rule is not, by itself, sufficient to provide the basis for an adverse impact claim. Usually, additional proof, such as statistically significant differences between the hiring rates of two groups is required.

According to the EEOC's 80% rule for determining disparate impact of hiring practices on a minority group, disparate impact occurs when: A. the hiring rate of a protected group is less that 80% of the hiring rate of another group. B. 80% or more of applicants in a protected group are not hired. C. less than 20% of applicants for the job are members of any protected group. D. the number of people who would have been hired based on a random selection process is more than 80% greater than the number of protected group applicants hired.

C. According to the Elaboration Likelihood Model, there are two routes of communication: a central route and a peripheral route. A listener is most susceptible to persuasion via the peripheral route when the communicator is appealing (e.g., is of high status), the listener is uninvolved with the message or is distracted, and/or the message appeals to fear

According to the Elaboration Likelihood Model, a persuasive message processed via the peripheral rather than the central route is most likely to be successful when: A. the message is not of an urgent nature B. the listener has time to process the message C. the communicator is of high status D. the communicator is demographically similar to the listener

A. Using a style approach, the Ohio State leadership studies identified two behavioral dimensions of leaders: initiating structure and consideration. These dimensions were treated as independent of each other in contrast to previous studies of leader behavior which placed related dimensions along a single continuum of leadership ranging from employee to production-centered (response "B"). In different models of leadership, the dimension of initiating structure is sometimes referred to as task-orientation and concern for production. The dimension of consideration has also been labeled employee orientation, relations-oriented and concern for people

According to the Ohio State University studies from the 1950s, what are the two dimensions of leadership? A. consideration and initiating structure B. employee-centered and production-centered C. autocratic and democratic D. authoritarian and laissez-faire

A. The tension-reduction hypothesis (Conger, 1956) proposes that use of alcohol or other substance reduces stress and, as a result, substance use is negatively reinforced

According to the tension-reduction hypothesis, substance addiction is maintained by: A. negative reinforcement. B. stimulus control. C. negative punishment. D. higher-order conditioning.

B. As you may have guessed, "floor" refers to the lowest scores on a test (ceiling refers to the highest scores). Adding more easy to moderately easy items would lower or decrease the floor allowing for better discrimination of people at the low end

Adding more easy to moderately easy items to a difficult test will: A. increase the test's floor. B. decrease the test's floor. C. alter the test's floor only if there is an equal number of difficult to moderately difficult items. D. have no effect on the test's floor.

B. This question requires you to have a clear understanding of the definitions of reinforcement and punishment. "Reinforcement" always increases the frequency or likelihood of the targeted response. "Punishment" always decreases the frequency or likelihood of the targeted response. "Positive" means that a stimulus is applied, and "negative" means that a stimulus is removed. Choice A is an example of negative punishment because time-out involves the removal of normal activities and because it previously decreased the misbehavior. Choice C involved the application of a stimulus (yelling "Stop!"); thus it is positive, and it resulted in a decreased frequency of the misbehavior, and therefore, it is also a form of punishment. Choice D involves the application of attention; thus it is positive and because it reduces the frequency of the misbehavior, it would also be a form of punishment. Ironically, only Choice B would be permitted by the mother's self-imposed rule because the spanking has resulted in further misbehavior; thus, it would be considered a reinforcement procedure. It would also be positive, because it involves the application of a stimulus

After reading a best-selling book on parenting, a mother decides to stop using all forms of punishment on her child. By following her self-imposed rule, when her child misbehaves the mother could only continue to: A. send the child to time-out, which previously resulted in an improvement in the child's behavior B. spank the child, which previously led to additional misbehavior C. yell "STOP!," which caused the child to discontinue misbehaving for a short period of time D. gently explain to the child why his behaviors are inappropriate, which previously resulted in an improvement in the child's behavior

A. This is an example of Bandura's (1986) concept of reciprocal determinism. From this perspective, the relationship between personal factors or cognitions, behavior and the environment take turns influencing or being influenced by each other. As in this case, not only does the environment influence thoughts and thoughts impact actions but also actions effect the environment. Covert modeling (b.) involves the learning of new behaviors or the altering of existing behaviors by imagining scenes of others interacting with the environment. Impulsive aggression (c.) describes emotion-driven aggression produced in reaction to situations in the "heat of the moment." Instinctual drift (d.) refers to the tendency for learned behavior to drift toward instinctual behavior over time.

Aggressive thoughts result in aggressive behavior, which in turn has the effect of causing others to have aggressive thoughts. This is an example of: A. reciprocal determinism B. covert modeling C. impulsive aggression D. instinctual drift

D. Subjective rating scale accuracy is influenced by the suspectibility to rater biases. A contrast effect (a.) occurs when a rater's ratings of a ratee are impacted by the ratings given to another. Severity error (b.), or strictness bias, refers to the tendency to use the low end of a rating scale to rate all ratees, whereas the central tendency bias (c.) is the tendency to use the middle of a scale in rating. A floor effect is not a rater bias. It refers to a measuring instrument's inability to distinguish between individuals who have low scores on whatever is being measured

All of following are rater biases EXCEPT: A. contrast effect B. severity error C. central tendency bias D. floor effect

B. Rater biases impacts the accuracy of subjective rating scales. When a rater rates all individuals using the middle of a rating scale, he/she is exhibiting the central tendency bias (a.). When ratings of one individual are affected by ratings given to another then the rater is exhibiting a contrast effect (c.). Severity error (d.), also known as strictness error or bias, refers to the rater tendency to rate all individuals using the low end of the rating scale. A ceiling effect is not a rater bias, it refers to the measurement instrument's inability to distinguish between individuals who have high levels of what is being assessed by the instrument.

All of the following are examples of a rater bias except: A. central tendency bias B. ceiling effect C. contrast effect D. severity error

D. The DSM-5 diagnosis of Substance Use Disorder integrates the DSM-IV-TR diagnoses of Substance Abuse and Substance Dependence but removed recurrent substance-related legal problems as a criterion and added craving as a criterion.©

All of the following are included as symptoms of Substance Use Disorder in the DSM-5 except: A. persistent desire or unsuccessful efforts to control substance use. B. a craving for the substance. C. recurrent substance use resulting in a failure to fulfill major role obligations. D. recurrent substance-related legal problems.

B. onset is commonly between 5 and 10 years of age According to the DSM-5, the onset of Childhood-Onset Fluency Disorder is usually from 2 to 7 years of age and occurs by age 6 for 80 to 90% of individuals. The other answers provide accurate information about this disorder

All of the following are true about Childhood-Onset Fluency Disorder (Stuttering) except: A. it is more common in males than females. B. onset is commonly between 5 and 10 years of age. C. severity of symptoms at age 8 is a good predictor of persistence or recovery into adolescence. D. it may be accompanied by motor movements such as lip tremors or eye blinks.

A. Item response theory is a highly technical mathematical approach to item analysis. Use of item analysis is based on a number of complex mathematical assumptions. One of these assumptions, known as invariance of item parameters, holds that the characteristics of items should be the same for all theoretically equivalent groups of subjects chosen from the same population. Thus, any culture-free test should demonstrate such invariance; i.e., a set of items shouldn't have a different set of characteristics for minority and non-minority subgroups. For this reason, item response theory has been applied to the development of culture-free tests, and choice A is not a true statement. The other choices are all true statements about item response theory, and therefore incorrect answers to this question. Consistent with choice B, item response theory is the theoretical basis of computer adaptive assessment, in which tests tailored to the examinee's ability level are computer generated. As stated by choice C, an assumption of item response theory is that items measure a latent trait, such as intelligence or general ability. And, finally, research supports the notion that the assumptions of item response theory only hold true for very large samples (choice D)

All of the following statements regarding item response theory are true, except: A. it cannot be applied in the attempt to develop culture-fair tests. B. it's a useful theory in the development of computer programs designed to create tests tailored to the individual's level of ability. C. one of its assumptions is that test items measure a "latent trait." D. it usually has little practical significance unless one is working with very large samples.

D. The first wave of behavior therapy refers to traditional behavior therapy, which employs the learning principle of conditioni ng to replace harmful behaviors with constructive ones. The second wave of behavior therapy, cognitive therapy, is used to change the thoughts that cause and perpetuate problem behaviors. While the first and second wave therapies are focused mainly on immediate problems, the third wave of behavior therapy, emphasizes the broad constructs of values, spirituality, relationships, and mindfulness. Traditionally non-clinical treatment techniques such as acceptance, mindfulness, cognitive defusion, dialectics, values, spirituality, and relationship development are being explored and the definition, causes and diagnosis of psychological problems and treatment goals of psychotherapy are re-examined. Acceptance and Commitment Therapy (ACT) (a.) is part of the third wave in behavior therapy. ACT therapists initially encourage a client to accept the circumstances of his/her life "as is" without judgment and then guides him/her in a progression of identifying a set of his/her core values. Once these values are identified, the focus of therapy is making short and long term commitments to act in ways that affirm and expand these values. In general, diagnosing and treating a specific mental illness is set aside as, in therapy, healing comes as a result of living a value-driven life rather than controlling or eradicating a particular set of symptoms

All of the following therapies are considered part of the "third wave" of behavior therapy except: A. acceptance and commitment therapy B. mindfulness training C. dialectical behavior therapy D. systematic desensitization

D. According to Allport, intergroup contact for reducing prejudice is only likely to be beneficial when intergroup contact is both frequent (a.) and of a duration that allows meaningful relationships to develop; the two groups have equal status (b.); the two groups are working towards common goals based on co-operation and social and institutional support is given. Allport notes that if these conditions are not met, contact may actually increase prejudice

Allport (1954) proposed the most effective way of reducing prejudice and discrimination is by intergroup contact presuming certain conditions are met. Which of the following is not one of the conditions? A. frequent intergroup contact B. equal status between groups C. social and institutional support D. empathy toward the other group

D. The combination of social avoidance, fear of humiliation, and loneliness are characteristic of Avoidant Personality Disorder.

Although Jasper has worked for the same company for nearly 15 years, he has no friends at work and never eats lunch or takes breaks with his coworkers. Jasper hasn't been to a party for eight years. He says he'd like to go to parties but he feels no one will want to talk with him and that people will make fun of him. He also says that he often feels lonely. The best diagnosis is: A. Paranoid Personality Disorder. B. Schizoid Personality Disorder. C. Schizotypal Personality Disorder. D. Avoidant Personality Disorder.

C. Discrimination and generalization refer to the differences in precision of stimulus control, or the ability of a stimulus to alter the probability of a response. Response probabilities vary with different contexts, discriminative stimuli, and reinforcement contingencies (e.g., what is likely to be rewarded, ignored, or punished). In this case, the adolescent behaves differently in one context (with friends) than in another context (with adults). Friends are a discriminative stimulus in which the response of swearing is more likely to occur. Overcorrection (d.) is a technique that is used to reduce an undesirable behavior and replace it with a more desirable one

An adolescent swearing with friends but not when around adults is example of: A. response generalization B. stimulus generalization C. stimulus discrimination D. overcorrection

B. Surgery increases the risk for delirium, especially for older adults.

An elderly man who is just out of surgery is at highest risk for: A. dissociation. B. delirium. C. amnesia. D. dementia.

D. Engineering psychology is concerned with fitting aspects of the job to the worker. The focus is on the total environmental system, including procedures, work environments, and the design and functioning of equipment. Thus, in this situation, an engineering psychologist would most likely focus on the safety of the work environment.

An engineering psychologist hired by a company to decrease accidents and improve safety in the organization would most likely focus on: A. working with employees to ensure that they understand and accept the company's safety policies B. ensuring that enough information about ways to increase safety is available to employees C. working with managers to increase their commitment to safety D. identifying aspects of the work environment that could be modified in order to improve safety

B. According to Ethical Standard 1.06 [Cooperating With Ethics Committees], "Psychologists cooperate in ethics investigations, proceedings, and resulting requirements of the APA or any affiliated state psychological association to which they belong. In doing so, they address any confidentiality issues. Failure to cooperate is itself an ethics violation. However, making a request for deferment of adjudication of an ethics complaint pending the outcome of litigation does not alone constitute noncooperation."

An ethics committee contacts a psychologist regarding the investigation of an alleged ethics violation by another psychologist. How should the psychologist respond? A. The psychologist has no ethical responsibility to cooperate with the ethics committee. B. The psychologist should cooperate with an ethics committee. C. The psychologist should wait until ordered by the court before cooperating with the ethics committee. D. The psychologist should obtain the other psychologist's consent before cooperating with the ethics committee.

B. Multiple hurdles approach requires applicants to score above a specific level or pass several measures one at a time. The usefulness of the multiple hurdles technique is maximized by administering the predictors in a logical order (e.g., from least to most difficult or least to most expensive).

An important decision when utilizing the multiple hurdles approach in personnel selection is: A. how to weigh each predictor. B. which order to administer the predictors. C. if the same person should administer the criterion and evaluate predictor performance. D. which statistical test to use to assess if the results are significant.

D. Covert modeling is based on its overt, simple modeling equivalent. Covert modeling involves the learning of new behaviors or the altering of existing behaviors by imagining scenes of others interacting with the environment. Overcorrection (a.) involves an individual consistently being reinforced for engaging in behaviors other than the target behavior during a predetermined period of time. Covert sensitization (c.) uses counterconditioning in imagination to reduce or eliminate a target behavior. An individual imagines he/she is engaging in the undesirable behavior and then imagines an aversive consequence for doing so.

An individual imagining others engaging in a variety of alternative or desirable behaviors is using the technique known as: A. overcorrection B. simple modeling C. covert sensitization D. covert modeling

D. D. Summarizing the predictive validity of interviews and other selection techniques in a meta-analysis of the research, Schmidt and Hunter (1998) report measures of general mental ability are the most valid predictors across different jobs (a.). For structured and unstructured interviews, they note corrected validity coefficients of .51 and .38, respectively (b.). When an interview is combined with another selection procedure, especially a general mental ability test, the predictive validity of interviews is increased

An organization is considering different selection techniques and comparing the respective predictive validity of each. The validity of structured interviews as predictors of performance: A. are the most valid across different jobs B. are no more valid than unstructured interviews when used alone C. is increased when the interview includes both structured and unstructured items D is increased when the interview is used in combination with a measure of general mental ability

A. A needs assessment, or analysis, is carried out to identify training needs and includes other analyses such as task analysis and person analysis. Determining training objectives (b.) is an outcome of a needs analysis and a personnel audit (c.) is part of an organizational assessment which is also a part of a needs analysis. Job evaluations (d.) are used to set wages and salaries

An organization is updating its telecommunication and information technology systems and its employees will need to be retrained on the changes. The most likely initial step for establishing training needs would be to: A. perform a needs analysis. B. pinpoint training objectives. C. complete a personnel audit. D. execute a job evaluation.

C. French and Raven identified six categories of power, which reflect the sources of influence upon which the power holder relies. Even if you were familiar with these categories, this question may have been difficult to answer, because all four of the choices could have been operating here. The way to choose the best answer is to work with the information you have at hand and not to add your own inferences to the question. Legitimate power refers to formal authority delegated to the individual in the form of rank, title, decree, etc. This question indicates that the requester is a high-ranking executive; i.e., he has a title and the formal authority and right to exercise power within the organization. Therefore, based on the available information, legitimate power is the best answer. Coercive power stems from the ability of the power-holder to punish others. Reward power arises from the ability to reward others. Referent power is based on personal qualities, and operates to the degree others identify with the power holder. As noted, all three of these power bases could have been at work here: the employee may have feared that the executive would fire him or otherwise punish him if he didn't work overtime; he may have thought the executive would give him a raise or reward him some other way if he did work overtime; or he may have accepted the request out of admiration for or loyalty to the executive. However, the question does not explicitly refer to any of these scenarios. The only source of power that is obvious--due to the wording that the request comes from a "high ranking executive"--is legitimate power. The other two sources of power are expert power, which stems from a belief that the power holder has specialized knowledge or expertise, and informational power, which stems from the ability to control the availability of information

An organization's employee agrees to a high-ranking executive's request for the employee to work overtime. The previous week, the same employee turned down a similar request from a peer. From the perspective of French and Raven's analysis of the bases of power, the employee accepted the executive's request due to the latter's _________ power. A. coercive B. reward C. legitimate D. referent

D. The changing complexion and feminization of the workforce has become a reality with racial/ethnic minorities and women compromising about 75% of the new entries into the labor force. The diversity index of the United States stands at 49, indicating that nearly half the people selected at random are racially or ethnically different. Racial/ethnic minority students constitute 45% of the population in U.S. public schools; some school systems in California reached 50% in the late 80s. It is estimated Visible Racial/Ethnic Minority Groups (VREG) will constitute a numerical majority sometime between 2030 and 2050

Approximately what percentage of those now entering the labor force are racial/ethnic minorities and women? A. 45% B. 55% C. 65% D. 75%

A. In the literature on the psychology of groups, a distinction between additive, conjunctive, disjunctive, and compensatory tasks is sometimes made. Additive tasks permit the addition of individual efforts so that the outcome is a combination of individual contributions. This is the type of task described by the question: The combined efforts of all group members will determine if the tasks are completed and team members receive a bonus. Let's briefly go through the other type of tasks: On conjunctive tasks, everyone must achieve a given goal in order for the task to be complete. As a result, task performance depends on the performance of the least competent group member. On disjunctive tasks, the group must choose one of many alternative ways to do the task. Thus, performance on a task depends on the performance of the most competent group member, because if one person can complete the task, the task gets completed. Finally, on compensatory tasks, the average performance of all group members represents the group's product.

As a management consultant for a corporation, a psychologist devises a way for employees working in groups to complete their work. The psychologist divides the employees into two teams. Each team has to complete a set of tasks, and all group members are free to work on any or all of the tasks. If all tasks are completed, each team member receives a bonus. If even one of the tasks is not completed, nobody receives a bonus. This is an example of which type of task? A. additive B. conjunctive C. compensatory D. disjunctive

A. The research has generally found neurofeedback to be an effective treatment for the core symptoms of ADHD. For example, based on a meta-analysis of the research, M. Arns et al. concluded that it has a large effect size for inattention and impulsivity and a medium effect size for hyperactivity.

As a treatment for ADHD, neurofeedback: A. has been found to be an effective treatment for the disorder's core symptoms. B. has been found to be an effective treatment for inattention only. C. has been found to be an effective treatment for hyperactivity only. D. has not been found to be an effective treatment for the disorder's core symptoms.

A. The DSM-5 diagnosis of Reactive Attachment Disorder requires a consistent pattern of inhibited and socially withdrawn behavior toward adult caregivers, a persistent social and emotional disturbance (e.g., limited positive affect), and evidence that the disturbance is related to exposure to extreme insufficient care

As described in the DSM-5, Reactive Attachment Disorder is characterized by: A. inhibited, emotionally withdrawn behavior toward adult caregivers. B. indiscriminate attachment to and overly familiar behavior with unknown adults. C. symptoms of Posttraumatic Stress Disorder in children. D. symptoms of Social Anxiety Disorder in children.

C. Both Malingering and Factitious Disorder involve the intentional production of physical or psychological symptoms. However, the motivation for symptom production is different. In Malingering, which is included in the DSM-5 with Other Conditions That May Be a Focus of Clinical Attention, the person intentionally fakes or exaggerates symptoms for a known external reason, such as to get out of work or avoid criminal responsibility. Individuals with Factitious Disorder produce symptoms without the presence of an external reward for doing so.

As described in the DSM-5, which of the following factors is most relevant for differentiating between Malingering and Factitious Disorder? A. compatibility of symptoms with a known medical condition B. demographic characteristics of the patient C. motivation for symptom production D. nature and duration of symptoms

A. The engineering psychologist tends to examine the factors making up the job and how these impact the worker. The job elements, from such things as the illumination in the plant to the work rate, are manipulated to maximize the productive work of the employee. Fitting the worker to the job (e.g., hiring better people) is a function of personnel psychology. By contrast, the engineering psychologist works with the job elements and designs the most productive environment for the worker hired

As opposed to industrial psychology, the approach of engineering psychology is to fit the: A. job to the worker. B. worker to the job. C. job to the organization. D. worker to the organization.

B. According to standard 1.04 of APA's ethical standards, "When psychologists believe that there may have been an ethical violation by another psychologist, they attempt to resolve the issue by bringing it to the attention of that individual, if an informal resolution appears appropriate and the intervention does not violate any confidentiality rights that may be involved." Here there is good reason to believe that the colleague is in violation of ethical standards involving confidentiality, so your obligation would be to bring the violation to his attention. The first choice also involves bringing the violation to the colleague's attention, but there is no ethical obligation (or common-sense need) to pull the client aside in a public place and address the issue in this setting.

At a supermarket, you overhear a colleague of yours discussing confidential information about a client with a mutual friend whom you know is not a mental health professional. Your best course of action would be to: A. pull the colleague aside and explain to him that he is committing an ethical violation. B. call the colleague later in the day and make an appointment to talk to him. C. report your colleague to the state ethics board. D. do nothing because the colleague has not necessarily violated ethical standards.

B. In his work on observational learning, Bandura found evidence for his theory that learning and performance are separate phenomena and that learning can occur without reinforcement. For example, in the study described in this question, children learned aggressive responses by watching an aggressive model (even when the model was punished) and, consequently, were able to imitate the aggressive behavior when told they would be rewarded for doing so. Interestingly, boys (but not girls) who were rewarded for imitating the aggressive model were more likely to do so when the model had been punished than when the model had been rewarded.

Bandura's research looking at the impact of punishing a model for acting aggressively on male and female observers showed that: A. when the model is punished, male and female children are more likely to act aggressively than when the model is rewarded. B. when the model is punished, performance (but not learning) of aggressive behaviors is reduced. C. when the model is punished, learning and performance of aggressive behaviors are both reduced. D. when the model is punished, male and female children are less likely to agree that aggressiveness is acceptable.

A. A co-founder of Gestalt psychology, which focuses on perception, Kohler's research revealed that animals, like humans, appear to experience insight or an "aha" experience while solving problems

Based on his animal learning and cognition research, Wolfgang Kohler concluded that learning is: A. insightful B. the result of reinforcement and punishment C. the result of trial-and-error D. biologically-based

A. In general, the greater the level of dependence on nicotine, the harder it is to stop smoking and the amount of smoking is typically used to determine dependence. In this question, however, dependence is not a factor of consideration. Research indicates there are other characteristics linked to successful smoking cessation. They include: being age 35 or older, married or living with a partner, later age when started smoking, and being male. Considering these factors, a young single male who started smoking at an early age is least likely to successfully quit smoking and most likely to relapse.

Based on the factors associated with successful smoking cessation, which of the following individuals is most likely to relapse? A. a single, 28 year old man who began smoking in high school B. a married, 50 year old woman who started smoking during her medical residency C. a co-habitating, 33 year old woman who began smoking in graduate school D. a divorced, 44 year old man who started smoking during his last year of college

D. According to the DSM-5, the symptoms of Antisocial Personality Disorder (APD), particularly criminal behaviors, often become less evident or remit by the time the person reaches the fourth decade of life

By the time an individual with __________________ reaches the fourth decade of life, symptoms often become less evident or remit and no longer meet all of the disorder's diagnostic criteria. A. Histrionic Personality Disorder B. Avoidant Personality Disorder C. Schizotypal Personality Disorder D. Antisocial Personality Disorder

C. dry mouth, headache, and hypotension. Since the question mentions that clonidine is safer for chronic use, you'd want to look for less serious side effects, which narrows it down to two responses ("dry mouth, headache, and hypotension" and "vomiting, dizziness, and irritability"). In addition to dry mouth, headache, and hypotension, clonidine may also cause sedation and dizziness

Clonidine, an alpha-noradrenergic drug, is often preferable to dopaminergic blocking agents for treating Tourette's Disorder because it is safer for chronic use. However, clonidine does produce some undesirable side effects including: A. Parkinsonism, akathesia, and dysphoria. B. anorexia, sexual dysfunction, and emotional blunting. C. dry mouth, headache, and hypotension. D. vomiting, dizziness, and irritability.

A. The purpose of cluster analysis is to place objects into categories. More technically, the technique is designed to help one develop a taxonomy, or classification system of variables. The results of a cluster analysis indicate which variables cluster together into categories. The technique is sometimes used to divide a population of individuals into subtypes.

Cluster analysis would most likely be used to: A. construct a "taxonomy" of criminal personality types. B. obtain descriptive information about a particular case. C. test the hypothesis that an independent variable has an effect on a dependent variable. D. test statistical hypotheses when the assumption of independence of observations is violated.

C. Cognitive dissonance theory predicts that, when we have two conflicting cognitions (e.g., I like the club but they don't want me"), we'll be motivated to reduce the tension that this causes by changing one of our cognitions. Since the student can't change the sorority's decision in this situation, she's likely to change her attitude toward the sorority

Cognitive dissonance theory predicts that, when a college student is not accepted by the sorority she most wanted to join, the student will: A. blame the sorority for not being accepted. B. blame herself for not being accepted. C. decide that she doesn't really want to be in the sorority after all. D. feel depressed until an alternative sorority accepts her.

B. According to the theory of cognitive dissonance, a person is motivated to reduce the negative, aversive state that results when his or her cognitions conflict with each other. From the perspective of this theory, the poor client paying a very high fee would experience a state of dissonance. Therefore, this client, more than those in the other responses, would be motivated to believe that he or she is benefitting from therapy.

Cognitive dissonance theory suggests that which of the following clients would be most likely to report benefitting from psychotherapy? A. a wealthy client who pays a very high fee B. a poor client who pays a very high fee C. a wealthy client who pays a very low fee D. a poor client who pays a very low fee

A. Rater biases include the strictness, leniency, central tendency, and halo biases. The other phenomenon listed in the responses are problems in other areas than performance ratings

Common biases that reduce rater accuracy when assessing job performance include: A. the leniency and strictness biases. B. the constant and contrast biases. C. the Rosenthal and Hawthorne biases. D. the reactivity and expectancy biases.

Neurocognitive Disorder Due to HIV Infection (common symptoms)

Common symptoms include cognitive slowing, impaired attention, and forgetfulness; apathy and social withdrawal; and clumsiness and leg weakness.

C. are more effective when the task is easy.

Compared to decentralized networks, centralized networks: A. are less effective but more accepted by employees. B. are more effective when the task is complex. C. are more effective when the task is easy. D. are more effective and more accepted by employees.

D. In computerized adaptive testing, the examinee's previous responses are used to tailor the test to his or her ability. As a result, inaccuracy of scores is reduced across ability levels

Computer-adaptive testing will yield: A. more accurate results for high scorers on a test. B. more accurate results for low scorers on a test. C. more accurate results for examinees who score in the middle range of a test. D. equally accurate results across all range of scores on a test.

C. The Levels of Processing Theory is an alternative to stage theories of memory. The levels of processing theory proposes that stimuli are processed at different levels, and the deeper the level of processing, the more likely it will be remembered. That is why techniques such as elaborative rehearsal, in which a person thinks about the meaning of new information and its relation to existing memories, is more effective than more superficial approaches such as "mindless" repetition. Of the choices offered in this question, rhyming would be the deepest, and therefore, most effective way to enhance recall.

Craik and Lockhart's Levels of Processing Theory would predict that in order to maximize recall of a list of words a person should: A. memorize the first letters of each of the words B. memorize the sequence of the words C. make up a rhyme using the words D. recreate the exact mental state that was experienced during the first exposure to the words

B. Crystallized Intelligence (Gc) is the breadth and depth of a person's acquired knowledge of a culture and the effective application of this knowledge. According to the APA Dictionary of Psychology, crystallized abilities are those abilities "such as vocabulary and cultural knowledge, that are a function of learning and experience in a given culture. Crystallized abilities are believed to depend on physiological condition somewhat less than do fluid abilities; thus they may be better sustained in old age. They are also believed by some to derive from fluid abilities."

Crystallized functions are considered to be: A. dependent on cultural factors B. dependent on learning and education C. culture-free D. independent of learning

B. Declarative memory is a type of long-term memory which one can report or declare. Another term for declarative memory is "explicit memory" because one can clearly or explicitly recall the memory. Implicit (A) memories, on the other hand, are retrieved without conscious awareness. Implicit memories are also referred to as "procedural" memories as these consist mostly of automatic skills or procedures. Semantic memory (D), which is knowledge about language and facts, is usually considered to be one type of declarative memory, but it is not a synonym for declarative memory. The other type of declarative memory is episodic memory, which is memory for events that have been personally experienced.

Declarative memory is also known as: A. implicit memory B. explicit memory C. working memory D. semantic memory

D. Hallucinogens (e.g., LSD), cocaine, and cannabis -- as well as alcohol, amphetamines, inhalants, opioids, PCP, and sedatives -- may produce Intoxication Delirium. Caffeine, however, does not cause Delirium.

Delirium can be the result of intoxication for all of the following substances except: A. cocaine B. LSD C. cannabis D. caffeine

B. Major Depressive Disorder with seasonal pattern (also known as seasonal affective disorder) is diagnosed when major depressive episodes consistently occur at the same time of year (most often in the fall and winter months in the northern hemisphere). Symptoms of seasonal affective disorder typically include: decreased energy, increased sleep, overeating, weight gain, and a craving for carbohydrates.

Depressive episodes that occur in a seasonal pattern most likely include: A. loss of weight B. increased sleep C. increased craving for protein-rich foods D. increased energy

A. imipramine (Tofranil) Desmopressin (DDAVP) and imipramine (Tofranil) are the primary drugs used to treat nocturnal enuresis. However, because they are associated with a high risk for relapse when discontinued, they are usually administered in conjunction with the urine (moisture) alarm

Desmopressin (DDAVP) and _________ are the primary medications used to treat nocturnal enuresis. © A. imipramine B. propranolol C. lithium D. methylphenidate

A. As described in the Federal Uniform Guidelines on Employee Selection, differential prediction is a potential cause of test unfairness. Differential prediction occurs when the use of scores on a selection test systematically over- or under-predict the job performance of members of one group as compared to members of another group. Criterion contamination occurs when a rater's knowledge of ratees' performance on the predictor biases his/her ratings of ratees' performance on the criterion (b.). Differential validity, also a possible cause of adverse impact, occurs when a predictor's validity coefficient differs for different groups (c.). When a test has differential validity (d.), there is a slope bias. Slope bias refers to differences in the slope of the regression line

Differential prediction is one of the causes of test unfairness and occurs when: A. members of one group obtain lower scores on a selection test than members of another group, but the difference in scores is not reflected in their scores on measures of job performance B. a rater's knowledge of ratees' performance on the predictor biases his/her ratings of ratees' performance on the criterion C. a predictor's validity coefficient differs for different groups D. a test has differential validity

A. There are many ways to assess the validity of a test. If we correlate our test with another test that is supposed to measure the same thing, we'll expect the two to have a high correlation; if they do, the tests will be said to have convergent validity. If our test has a low correlation with other tests measuring something our test is not supposed to measure, it will be said to have discriminant (or divergent) validity. Convergent and divergent validity are both types of construct validity.

Discriminant and convergent validity are classified as examples of: A. construct validity. B. content validity C. face validity. D. concurrent validity.

D. cognitive slowing and apathy. Neurocognitive Disorder Due to HIV Infection includes cognitive, personality, and motor symptoms. Common symptoms include cognitive slowing, impaired attention, and forgetfulness; apathy and social withdrawal; and clumsiness and leg weakness.

Early signs of Neurocognitive Disorder Due to HIV Infection are: A. disorientation and aphasia. B. speech impairments and inappropriate social behavior. C. irritability and obsessive-compulsive symptoms. D. cognitive slowing and apathy.

D. According to Ethics Standard 2.01(d), "When psychologists are asked to provide services to individuals for whom appropriate mental health services are not available and for which psychologists have not obtained the competence necessary, psychologists with closely related prior training or experience may provide such services in order to ensure that services are not denied if they make a reasonable effort to obtain the competence required by using relevant research, training, consultation, or study." Since Dr. Bean is the only mental health professional in town, this is the best course of action of those given in the responses since there is no indication that the clients have problems that Dr. Bean doesn't want to treat or that she would not be able to treat with consultation or supervision.

Dr. Bean is a newly licensed clinical psychologist who is practicing in a small rural town. She is the only mental health practitioner in a 150-mile radius. Dr. Bean finds that some of her clients have mental health problems that she has little or no experience treating. Her best course of action is to: A. refuse to treat the clients who have problems that she is unfamiliar with. B. obtain appropriate training before treating the clients who have problems that she is unfamiliar with. C. inform the clients of her lack of experience and let them decide what to do. D. see the clients but obtain supervision or consultation by telephone.

B. Although the Ethics Code does not specify exactly how to resolve this type of ethical dilemma, it does state, "If a psychologist finds that, due to unforeseen factors, a potentially harmful multiple relationship has arisen, the psychologist takes reasonable steps to resolve it with due regard for the best interests of the affected person and maximal compliance with the Ethics Code" (Ethical Standard 3.05[b]). Consistent with the intent of the Code, the best option would probably be to discuss the matter with your client immediately. After discussing it with your client, additional steps, such as referring the client, may be appropriate

During the 10th psychotherapy session with a client, you realize that your client has recently started dating your best friend. You should: A. refer the client to another therapist B. discuss this with your client immediately C. discontinue your relationship with your friend D. wait and address the problem with your client if a conflict becomes imminent

A. This question involves deception in research, distress and debriefing. Due to the use of deception and the resultant participant distress, debriefing immediately following participation is the best option as described by Standard 8.07(c) Deception in Research: "Psychologists explain any deception that is an integral feature of the design and conduct of an experiment to participants as early as is feasible, preferably at the conclusion of their participation, but no later than at the conclusion of the data collection." The Ethics Code on Debriefing, Standard 8.08(a), notes, "Psychologists provide a prompt opportunity for participants to obtain appropriate information about the nature, results, and conclusions of the research, and they take reasonable steps to correct any misconceptions that participants may have of which the psychologists are aware," and in Standard 8.08(c) "When psychologists become aware that research procedures have harmed a participant, they take reasonable steps to minimize the harm.

During the course of a research experiment, subjects witness someone who is apparently hurt, needing immediate help, and are unable to help or know if the person is assisted. The subjects become immediately distressed. In this situation: A. debriefing occurs at the conclusion of participation B. group debriefing occurs at the conclusion of collection of data C. debriefing occurs when subjects become distressed D. debriefing occurs at the conclusion of research

A. The general adaptation syndrome consists of three stages: alarm, resistance, and exhaustion. During the resistance phase, the pituitary gland releases ACTH which, in turn, signals the adrenal cortex to release cortisol and other stress hormones

During the resistance phase of the general adaptation syndrome, the ____________ releases cortisol and other stress hormones. A. adrenal gland B. pituitary gland C. pineal gland D. thyroid gland

B. Research investigating the impact of personal and contextual characteristics on the creativity of employees indicates increasing the physical distance between workers (a.), providing supportive supervision (d.), and being informed evaluations will be developmental rather than judgmental has a positive effect on employee creativity. The expectation that one's work will be critically evaluated tends to diminish creativity. Of the five personality factors, openness to experience (c.) has been most strongly related to creativity

Employee creativity is impacted by personal and contextual characteristics. Creativity is diminished by: A. reduced density of work environment B. expecting a critical evaluation of work C. scoring high on a measure of openness to experience D. supportive supervision

C. Criterion contamination occurs when a rater knows how a ratee did on a predictor test, and this knowledge affects the rating. For example, if an employee obtained a very high score on the post-training test and the supervisor knows this, the supervisor's ratings of the employee's on-the-job performance might be biased upward. To prevent criterion contamination, the rater should have no knowledge of the ratees' predictor scores. Contrast effect (A) refers to the tendency to give ratings on the basis of comparisons to other ratees. There is no indication that there is a restricted range of scores (B), and if there was, that would reduce the correlation between the predictor and criterion scores. Demand characteristics (D) are cues in the environment that inform research participants what behaviors are expected of them.

Employees are provided with a training program designed to improve the quality of their work. At the conclusion of the training program a supervisor administers a test to the employees. Three months later the same supervisor evaluates the employees on their job performance and correlates the two scores. The correlation between the scores is 0.70. The results are most likely biased due to: A. a contrast effect B. a restricted range of scores C. criterion contamination D. demand characteristics

C. Distributive justice refers to the perceived fairness of outcomes. The outcome in organizations is typically considered to be a decision made regarding the employee such as hiring, performance appraisals, raise requests, or layoff decisions. Procedural justice (a.) refers to the perceived fairness of the process by which outcomes were allocated. Interactional justice (b.) refers to the exchange between an individual and supervisor or third party. Recent research suggests interactional justice consists of two distinct justice dimensions, informational and interpersonal. Informational justice (d.) refers to the amount of information or the appropriateness of the explanations provided about why procedures were used or outcomes were distributed in a certain way. Interpersonal justice refers to how an individual is treated by a supervisor or third party involved in executing procedures or determining outcomes.

Equity Theory focuses on: A. procedural justice. B. interactional justice. C. distributive justice. D. informational justice.

D. Equity theory holds that, in any work situation we assess both our input (how much effort we are contributing to a work situation) and our output (how much we get out of it). We then compare our input/output ratio to what we perceive are the ratios of our co-workers. A state of balance exists when we perceive that our input/output ratios are equal to those of our co-workers. An unbalanced situation is one in which these ratios are not perceived as equal. For instance, underpayment is an unbalanced situation in which outcomes are perceived to be less than contributions, and overpayment occurs when we perceive that our outcomes exceed contributions. Equity theory holds that motivation is affected by these perceived input/output ratios. For instance, if you perceive that you are contributing more to a company than a co-worker who is getting paid the same amount, you would take action to correct this inequity, whether it be by working less, asking for a raise, leaving the job, re-evaluating the value of your contributions, etc. Similarly, if you are getting paid more than a co-worker whom you perceive as just as valuable as yourself, you may work more, do better quality work, decide that you are more important than the co-worker, etc. The point is that the theory suggests that the relationship between pay and motivation is complex and will be mediated by one's perceived input, the perceived input/output ratio of co-workers, and the strategy chosen to achieve a state of balance or equity.

Equity theory suggests that the relationship between pay and motivation: A. is positive; the more one is paid, the more motivated one is likely to be. B. is negative; the more one is paid, the less motivated one is likely to be. C. is negligible; pay has little or nothing to do with motivation. D. is complex; the relationship between pay and motivation is mediated by a variety of other factors.

A. Despite the mother's intentions, her reprimands appear to result in an increase in her child's undesirable behavior. The application of a stimulus following a response that increases the frequency of that response is, by definition, positive reinforcement. Punishment, by definition, always decreases the frequency of a behavior - which did not occur in this case. And the mother's behavior is not an example of intermittent reinforcement (D) because she is reprimanding the child every time he uses obscene language.

Every time a 6-year-old child uses obscene language his mother reprimands him. Unfortunately, the child's use of obscene language increases. The mother's reprimands can best be described as: A. positive reinforcement B. negative reinforcement C. positive punishment D. intermittent reinforcement

D. Response cost, a form of negative punishment, involves removal of a prespecified reward every time a behavior targeted for elimination is performed. Here, the prespecified reward being removed is a portion of the child's allowance, and the behaviors targeted for elimination are those on the list constructed by the parents and the therapist

Every time a child engages in one of the prohibited behaviors on a list of targeted behaviors constructed by the child's parents and his therapist, the child's allowance is reduced by 20 percent. The technique being employed in this case is: A. positive punishment. B. time-out from positive reinforcement. C. token economy. D. response cost.

B. Evidence that symptoms are due to exposure to extremes of insufficient care is a diagnostic criterion for two DSM-5 diagnoses: Disinhibited Social Engagement Disorder and Reactive Attachment Disorder

Evidence that symptoms are due to exposure to extremes of insufficient care is a diagnostic criterion for which of the following disorders? A. Separation Anxiety Disorder B. Disinhibited Social Engagement Disorder C. Disruptive Mood Dysregulation Disorder D. Dissociative Identity Disorder

A. Evidence for implicit memory arises in priming, a process from which individuals show improved performance on tasks for which they have been subconsciously prepared. Priming effects, or prior exposure increases retrieval accuracy, helps implicit memory. Explicit memories are created by "doing" something with your experiences such as thinking, talking, writing or studying about them. The more you do, the better you will remember and the internal cues used in processing the information, can be used to initiate spontaneous recall. For example: the words that you used when you've talked about a certain experience will help you a year later when trying to remember this experience. Declarative memory is another term for "explicit memory" because one can clearly or explicitly declare the recalled memory. Declarative or explicit memories are long-term memories that one can consciously recall. They include semantic (factual information) and episodic (personally experienced) memories. The hippocampus, medial thalamus, and prefrontal cortex are associated with explicit or declarative memory and damage to these area would most likely result in deficits in conscious recall of facts and events. Explicit memory deficits due to head trauma or disease usually affect recent long-term memory before affecting remote long-term memory.

Explicit memory is associated with each of the following except: A. priming B. recall or recognition tests C. an active or conscious role in processing information D. a sense of remembering a specific prior experience

C. Generally speaking, exposure treatment involves presenting an individual with anxiety-producing stimuli long enough to decrease the intensity of their emotional reaction so that the feared situation or thing no longer makes the individual anxious. Exposure procedures have two forms: exposure to environmental situations that each patient fears and exposure to the physical sensations associated with panic attacks. Interoceptive exposure involves the structured and repeated exposure to panic-like physical sensations such as hyperventilation, shaking head and body tension. Exposure treatment for feared objects and situations can be carried out in real situations, in vivo exposure (a.); or it can be done through imagination, imaginal exposure. The category of imaginal exposure includes systematic desensitization (b.), which asks an individual to imagine certain aspects of the feared object or situation in combination with relaxation. This is sometimes also called graded or graduated exposure, which refers to exposing an individual to the feared situation in a gradual manner. Flooding (d.) refers to exposing an individual to the anxiety-provoking or feared situation all at once and keeping him or her in it until the anxiety and fear subside.

Exposure treatment is a technique that is widely used in cognitive-behavioral treatment for panic disorder and anxiety disorders. The technique that induces the bodily sensations that mimic those experienced during a panic attack is called: A. in vivo exposure B. systematic desensitization C. interoceptive exposure D. flooding

B. In extinction interventions, you don't want anything to accidentally reinforce the behavior. Let's say you advised parents to ignore their child's whining attention-seeking behavior, and the behavior reduced in intensity. Then, Grandma comes for a visit and when the child whines she gives him a cookie. This would be reinforcing, and so we'd be back to square zero and would have to do the extinction all over again. So, accidentally applied reinforcement often interrupts the extinction procedure.

Extinction treatment paradigms are often limited by: A. improperly applied reinforcers. B. accidental reinforcement. C. client expectations. D. practical constraints such as time.

A. Feature integration theory is what it sounds like. It's a theory about how an object's features are integrated, and it proposes that focused visual attention is required for perception of an object to occur.

Feature integration theory predicts that the perception of an object as an entity rather than as a cluster of unrelated features depends on: A. focused attention. B. integrated attention. C. selective attention. D. divided attention.

D. This is an example of a question with more than one good answer. Choices "A", "B", and "C" may all be acceptable, depending on the circumstances. But because there are a number of acceptable answers to this question, you'd hope to find one general answer, such as "D", that covers most or all of the specific possible options. More importantly, of the choices listed, "D" is most consistent with the language of APA's Ethical Standard 6.04(a), which states: "As early as is feasible in a professional or scientific relationship, psychologists and recipients of psychological services reach an agreement specifying compensation and billing arrangements

Fee arrangements should be made with clients A. at the beginning of treatment. B. at the end of the first session. C. on the phone before the first session. D. as soon as possible.

C. In 2001-2002, the National Institute on Alcohol Abuse and Alcoholism (NIAAA) conducted the NESARC study gathering data from over 43,000 individuals, age 18 or older, in non-institutionalized and group housing populations. The survey data provides estimates on alcohol and drug use, alcohol use disorders (AUDs), utilization of alcohol treatment services, and comorbid DSM-IV disorders. In a trend analysis of AUDs over the past decade, Grant et. al. found the prevalence of alcohol abuse increased while the prevalence of dependence declined between 1991-1992 and 2001-2002. The prevalence of abuse was greater among Whites than among Blacks, Asians, and Hispanics and the prevalence of dependence higher among Whites, Native Americans, and Hispanics than among Asians. Both abuse and dependence were more common among men and younger respondents.

Findings from the National Epidemiologic Survey on Alcohol and Related Conditions (NESARC) indicated the highest prevalence of alcohol abuse was among: A. Asians B. Hispanics C. Whites D. Blacks

C. Studies investigating cognitive impairments due to prolonged, heavy alcohol use have found that it is more likely to cause problems in visuospatial skills than in verbal skills

Following prolonged heavy alcohol consumption, a person is most likely to exhibit: A. a comparable pattern of deficits in visuospatial and verbal skills. B. deficits in either visuospatial or verbal skills, depending on whether the person is left- or right- brain dominant. C. greater deficits in visuospatial skills than in verbal skills. D. greater deficits in verbal skills than in visuospatial skills.

C. The DSM-5 diagnosis of Separation Anxiety Disorder requires the presence of symptoms for at least four weeks in children and adolescents or six months in adults.

For a DSM-5 diagnosis of Separation Anxiety Disorder for adults, symptoms must be present for at least: A. three months. B. four weeks. C. six months. D. eight weeks.

D. The median age of onset of Panic Disorder is 20 to 24 years. However, a small number of cases begin in childhood. Although not all theorists agree on whether or not children have the cognitive ability to catastrophize, the research indicates that 6 to 12-year-olds are capable of experiencing panic attacks that are typically manifested as chest pain, tachycardia, shortness of breath, and refusal to go to school.

For pre-adolescents, Panic Disorder is: A. not diagnosed since pre-adolescents cannot exhibit any of the cognitive symptoms of Panic Disorder. B. impossible to distinguish from Separation Anxiety Disorder. C. most likely manifested as fear and a feeling of "going crazy." D. most likely manifested as chest pain, tachycardia, shortness of breath, and refusal to go to school.

C. Test-retest reliability, or the coefficient of stability, involves administering the same test to the same group on two occasions and then correlating the scores. Alternative forms reliability, or coefficient of equivalence (response "B"), consists of administering two alternate forms of a test to the same group and then correlating the scores. Internal consistency reliability (response "D") utilizes a single test administration and involves obtaining correlations among individual test items. Split-half reliability (response "A") is a method of determining internal consistency reliability.

Form A is administered to a group of employees in Spring and then again in Fall. Using this method, what type of reliability is measured? A. split-half B. equivalence C. stability D. internal consistency

D. Wolpe distinguished between several types of depression. He linked neurotic depression to anxiety and considered systematic desensitization to be an effective treatment.

From Wolpe's classical conditioning perspective, neurotic depression: A. is due to attributional biases that, through conditioning, have become associated with certain types of events and can be eliminated through reattribution training. B. results when there is an absence of response contingent reinforcement and is best treated by counterconditioning in which depression is paired with a variety of pleasure-producing (unconditioned) stimuli. C. is a conditioned response that can be alleviated through extinction trials in which the neutral (conditioned) stimulus is repeatedly presented without the depression-inducing (unconditioned) stimulus. D. is a response to anxiety and can, therefore, be alleviated by using systematic desensitization to eliminate the anxiety.

A. In any system, feedback is useful to make corrections. Even our nervous system makes use of this phenomenon: as your body temperature rises on a cold morning, you stop shivering. Likewise with institutions and organizations. This question asks about positive feedback, which, in systems terminology, refers to information that encourages disruption from the status quo. In organizational consulting and family therapy, positive feedback is used to correct or change a pattern; in response to positive feedback the system deviates from homeostasis, or the status quo. This question is not about reinforcing other patterns or extinguishing maladaptive patterns, as two of the choices indicate. These terms are related to behavioral learning theory, not systems theory.

From an organizational systems approach, positive feedback is most useful as an intervention in order to: A. provide information which helps plan for corrective actions. B. reinforce adaptive responses. C. extinguish maladaptive organizational interactions. D. establish productive work groups.

A. The technique of goal setting involves allowing workers to set their own performance goals, or setting performance goals for them that they accept. Research on goal-setting theory indicates that the technique is most effective when goals are at least moderately difficult and specific feedback regarding progress toward the goal is provided.©

Goal setting as a motivational technique is most effective when: A. performance goals are difficult and specific feedback is provided regarding progress toward the goal B. performance goals are difficult and general feedback is provided regarding progress toward the goal C. performance goals are easy and specific feedback is provided regarding progress toward the goal D. performance goals are easy and general feedback is provided regarding progress toward the goal

Unconscious inference

Helmholtz's term for perception that occurs outside of conscious awareness

B. Holland's six occupational themes represent both personality and environment types. According to Holland, job satisfaction, productivity, and other behaviors are affected by the congruence or fit between a person's personality and the nature of the work environment

Holland's approach to vocational guidance is based on the assumption that: A. interests motivate occupational choice B. behavior is a function of personality-environment congruence C. job satisfaction and performance are related to a match between aptitude and job requirements D. interests change in a predictable way over the course of the lifespan

C. Holland's theory categorizes individuals based on their resemblance to six personality types: realistic, investigative, artistic, social, enterprising, and conventional (you can remember the sequence as "RIASEC"). The personality types reflect a person's primary interest, or an environment's primary characteristic; for example, the investigative type enjoys working alone and solving complex problems; likes dealing with ideas more than people or things and has math or scientific abilities. There is overlap between the types, with some overlapping more than others. For example, the investigative type is more similar to the realistic and artistic (a.), and less similar to the enterprising, conventional (b.) and social (d.) types.

Holland's investigative type is least similar to which other one of his types? A. artistic B. conventional C. enterprising D. social

B. This question refers to the Hopelessness Theory of Depression (HTD) and research investigating it. According to this theory, individuals with a depressogenic attribution style are more vulnerable to depression that others. This style involves making internal, stable, and global attributions for, or causal explanations of, negative events, as opposed to external, unstable, and specific attributions. The first choice represents an internal attribution, the third a global attribution, and the fourth a stable attribution. Only the second choice attributes the colleagues' behavior to an external (the colleague not the person), unstable (he is busy) and specific (today) cause. Research does show that depressed individuals tend to explain negative events in terms of internal, stable, and global causes, though it has not been established that this truly represents a cognitive tendency which causes depression.

If a depressed person is ignored by one of his colleagues at work, he is least likely to explain the colleague's behavior with which of following statements? A. "It's like I'm invisible around here." B. "He is very busy today." C. "This company has a culture of rudeness." D. "What a jerk!"

C. A validity coefficient and the standard error of estimate are both measures of the accuracy of a predictor test. A validity coefficient is the correlation between scores on a predictor and a criterion (outcome) measure. A coefficient of 1.0 reflects a perfect correlation; it would mean that one would always be able to perfectly predict, without error, the scores on the outcome measure. The standard error of estimate indicates how much error one can expect in the prediction or estimation process. If a predictor test has perfect validity, there would be no error of estimate; you would always know the exact score on the outcome measure just from the score on the predictor. Therefore, the closer the validity coefficient is to 1.0, the smaller the value of the standard error of estimate, and if the coefficient were 1.0, the standard error of estimate would be 0.

If a predictor test has a validity of 1.0, the standard error of estimate would be: A. equal to the standard deviation of the criterion measure. B. 1.0 C. 0.0 D. unknown; there is not enough information to answer the question.

D. The effectiveness of persuasive communication depends on numerous factors. One important factor is the credibility of the communicator. Individuals who are perceived as trustworthy and possessing expertise are credible, and such communicators can induce attitude change in an audience even when there is a large discrepancy between the audience's initial position and the communicator's argument. However, research shows that the effectiveness of well-presented communication from a source low in credibility tends to increase over time, because people tend to remember the message but forget the source. This phenomenon is known as the "sleeper effect". The other choices all present factors likely to decrease the effectiveness of an argument from a source low in credibility. Message recipients who are very ego-involved in a particular issue are less-likely to change their opinion about that issue than those who are less ego-involved. And as noted already, individuals who are low in credibility are unlikely to induce opinion change when there is a large discrepancy between the communicator's and the audience's position. Finally, messages that arouse a moderate amount of fear are associated with greater attitude change than messages than induce very little or very high fear.

If a speaker with moderate to low credibility presents a well-stated argument in favor of a particular position, he can expect the greatest attitude change: A. if the audience is very ego-involved in the issue. B. if there is a large degree of discrepancy between his position and that of the audience. C. if his argument arouses a great deal of fear. D. as time passes.

C. One of the factors that affect the reliability coefficient is guessing. Guessing correctly decreases the reliability coefficient. Answers A, B, and D are not true in regards to the reliability coefficient.

If an examinee correctly guesses the answers to a test, the reliability coefficient: A. is not affected B. stays the same C. decreases D. increases

D. The relationship between age and satisfaction actually depends on what aspect of satisfaction you're measuring. However, the question refers to global satisfaction, and the research has found that higher levels of global job satisfaction are associated with older age and vice versa

If you administer a measure of global job satisfaction to employees at the Acme Corporation, you would expect to find the highest levels of satisfaction among workers aged: A. 20 to 25. B. 30 to 35. C. 40 to 45. D. 50 to 55.

D. This question is indirectly referencing McGuire's inoculation theory, which proposes that a particular attitude or belief can be strengthened by exposing someone to the opposing belief -- especially when the opposing argument is weak or the person is supplied with counter-arguments against the opposing belief. Note that this is analogous to medical inoculation, which involves injection of a weak form of a germ so the body can build up defenses against that germ

If you hear arguments against your opinion, followed by arguments against the opposing opinion, what is likely to happen? A. You will become very confused. B. You will become more dogmatic in your original opinion. C. You will change your opinion. D. Your resistance to future opposing arguments will be increased.

D. Type II errors occur when the null hypothesis is not rejected when it is in fact false; Type I errors are often considered more serious as the null hypothesis is wrongly rejected. For example, in the clinical trial of a new drug, this would be concluding that the new drug was better when in fact it was not. Type I and II errors are inversely related: as the probability of a Type I error increases, the probability of a Type II error decreases, and vice versa.

In a clinical trial of a new drug, the null hypothesis is the new drug is, on average, no better than the current drug. It is concluded that the two drugs produce the same effect when in fact the new drug is superior. This is: A. corrected by reducing the power of the test B. corrected by reducing the sample size C. a Type I error D. a Type II error

D. A finding of not guilty by reason of insanity means that the person was legally insane at the time of the crime. "Legally insane" does not mean the same thing as mentally ill; a person can be mentally ill at the time of a crime and not necessarily be legally insane. The term's definition will vary from state to state, but in almost all cases, it will be along the lines of not being able to distinguish right from wrong at the time of the crime due to a mental defect or disorder.

In a criminal case, the finding "not guilty by reason of insanity" means that the defendant: A. did not understand the charges against him. B. could be retried once the insanity passes. C. could be held for involuntary treatment for no longer than three months. D. was legally insane at the time of the crime.

D. Any limiting criterion is acceptable in a job selection procedure as long as the criterion is shown to be job-related. For instance, you could exclude people with very poor eyesight as air traffic controllers, you could exclude people with very little stamina and strength as firefighters, and so forth. Thus, if an employer can empirically demonstrate that being a certain age is a bona-fide occupational requirement, it could be used as a job criterion. Let's say you were hiring painters to work on the Golden Gate Bridge. You might very well find that people over age 60 couldn't do the job safely and well. If you could prove this finding empirically, you could use age as a criterion. Note, however, that if an employer is challenged on the use of a discriminatory exclusionary criterion, the employer bears the legal burden of demonstrating that it is job relevant

In a job selection decision, age can be used as an exclusionary criterion: A. never. B. if there is a ruling from the appropriate federal agency allowing it. C. if there is a significant difference in mean ages of incumbents and applicants. D. if age is directly related to job performance.

C. The results of Stajkovic and Luthans' meta-analysis found self-efficacy positively related to work-related performance and that the relationship is moderated by task complexity and situational factors present in work environments. The two moderators appear to weaken the relationship between self-efficacy and work-related performance, with the relationship weakest for the higher levels of task complexity and field settings. Situational factors and organizational practices identified as influences on the relationship between self-efficacy and performance include: accuracy of task descriptions, definitions and circumstances of the tasks; instruction on necessary technical means and how to use them for successful task performance; physical distractions in the work environment; training programs that enhance employees self-efficacy beliefs of what they can do with the skills they already possess; developing effective behavioral and cognitive coping strategies, becoming more task-diagnostic, conceptualizing ability as an incremental skill; timing of program implementation; clarity and objectivity of performance standards; and personal consequences contingent on performance.

In a meta-analysis of the relationship between self-efficacy beliefs and achievement outcomes, Stajkovic and Luthans (1998) found self-efficacy was positively and strongly related to work-related performance and: A. the relationship between self-efficacy and work-related performance is moderated by task complexity B. the relationship between self-efficacy and work-related performance is moderated by locus of performance C. the relationship between self-efficacy and work-related performance is moderated by task complexity and locus of performance D. the relationship between self-efficacy and work-related performance is not influenced by task complexity or locus of performance

A. This may be a difficult item until one is clear about the "just world" concept. Those scoring high on the "belief in a just world" scale tend to blame the victim for crimes, out of a need to believe that bad crimes should not occur to good people. Thus, such individuals might wonder if a short skirt or make-up invited the crime. Otherwise, these people are not systematically more prejudiced, punitive, or fair in their judgments

In a rape case, jurors highest in their "belief in a just world" are most likely to: A. wonder what the rape victim was wearing. B. assume the truth will come out. C. want to punish the defendant most harshly. D. try to be most fair in judging the defendant.

B. Knowing the difference between retroactive and proactive interference would have helped you narrow the choices down to responses A and B. (In retroactive interference subsequent learning interferes with previous learning). In studies on retroactive interference, experimental subjects learn material similar to the original list, while control subjects engage in a dissimilar activity (e.g., counting backwards) that keeps them from rehearsing the list

In a study, subjects are asked to memorize a list of 12 unrelated words. After a brief period of time, they are asked to count backwards from 100 and are then tested on the word list. These subjects are: A. the experimental subjects in a study on retroactive interference. B. the control subjects in a study on retroactive interference. C. the experimental subjects in a study on proactive interference. D. the control subjects in a study on proactive interference.

A. An approach-approach conflict occurs when a person is faced with two equally attractive and conflicting alternatives (e.g., "should I buy the Corvette or should I buy the Porsche?"). Once a person makes a choice, the tendency is to find the chosen alternative more attractive and the unchosen alternative less attractive. This is consistent with the phenomenon of cognitive dissonance, whereby a person is motivated to reduce psychological tension between competing attitudes by altering one of those attitudes.

In an approach-approach conflict, which of the following is most likely to occur after a choice is made? A. The person will find the chosen alternative more attractive and the unchosen alternative less attractive. B. The person will find the chosen alternative less attractive and the unchosen alternative more attractive. C. The person will find both alternatives equally attractive and more attractive than beforehand. D. The person will find both alternatives equally attractive and less attractive than beforehand.

C. If a person's pay is based on output, his or her output is likely to increase, but the quality of work will not necessarily increase. This makes sense -- if you are working so fast in order to produce more so that you can get paid more, the quality of your work is likely to suffer. In addition, increased pay is likely to decrease job dissatisfaction, as predicted by Herzberg's two-factor theory.

In an attempt to increase motivation among employees, a company begins to pay employees based on their output. This will most likely result in: A. increased job satisfaction and increased quality of work B. decreased job dissatisfaction and increased quality of work C. decreased job dissatisfaction and increased quantity of work D. increased job satisfaction and no change in quantity of work

D. A discriminative stimulus is a cue indicating some contingency (reinforcement or punishment) will occur if a particular behavior is emitted. That's the case here -- the light is serving as a cue that the person will be reinforced for finding the ace of spades. The alternative "stimulus" is too vague because that term is much less specific than discriminative stimulus. The situation isn't reinforcement because a reinforcer always occurs after the behavior, not before. Finally, fading is related to the discriminative stimulus, but refers to a process in which the signal is slowly withdrawn before the behavior occurs.

In an operant conditioning experiment, researchers flash a light to signal the subject that she will receive a dollar bill for finding the ace of spades in a deck of cards. This signal is called: A. a reinforcement. B. a stimulus. C. fading. D. a discriminative stimulus.

C. The paired comparison technique involves comparing each person to every other person in the organization in reference to one or more variables such as productivity. It is a time-consuming and expensive process but results in highly precise and differentiated evaluations of each person in the group evaluated.

In any organized setting of more than a few people, paired-comparisons is the preferred rating system when the goal is to: A. establish a standard performance criterion for the average productivity rate in that organization. B. rank-order the people in the organization in the most cost-effective manner. C. maximally differentiate among the people in the organization. D. set the most attainable and acceptable production goals for each person in that organization

D. Maslow's theory, as it applies to workplace settings, has not been supported by research. For instance, the notion that lower level needs, such as pay and job security, cease to be important once they are satisfied has not been shown to be true

In applied industrial settings, Maslow's theory has been shown to be: A. applicable for lower-level employees only. B. appropriate for supervisors but not managers. C. applicable across the board. D. not applicable.

A. Finkelhor (Sexually Victimized Children New York: Free Press 1979) reports that the abuser is typically either a relative or close family friend of the victim. In assessing for sexual abuse, clinicians take into account "power differentials" and thus in the case of a same-age sibling, (Answer C), it may be considered sexual exploration or play

In cases of sexual abuse, the abuser is typically: A. A relative or close family friend B. A stranger C. A same-age sibling D. An alcoholic

D. In his discussion of consultee-centered case consultation, Gerald Caplan notes that, especially when the target of the consultation is the consultee's lack of skill, this form of consultation most resembles "technical supervision."

In consultee-centered case consultation, the consultant's role is most similar to which of the following? A. collaborator B. coach C. confidant D. supervisor

A. Job enrichment is a motivational technique that involves giving employees increased responsibility, decision-making authority, and autonomy. It is based on Herzberg's two-factor theory of motivation, which classifies factors that affect job satisfaction into two categories: motivators, which increase satisfaction, and hygiene factors, which prevent dissatisfaction. Motivators are related to intrinsic conditions of the job itself, such as the autonomy and the opportunity for personal growth. Job enrichment attempts to provide motivators in order to increase satisfaction and ultimately performance. Job enlargement is also an attempt to increase an employee's satisfaction and motivation. It involves increasing the number of tasks an employee performs. The idea is to reduce the level of monotony and boredom; however, research suggests that job enrichment is the more effective technique.

In contrast to job enlargement, job enrichment involves: A. increasing an employee's decision-making authority. B. increasing the number of tasks done by an employee. C. an attempt to increase the employee's motivation. D. increasing external rewards for the employees such as pay and benefits.

C. According to equity theory, individuals compare the ratio of inputs they bring to a job (e.g., amount and quality of work) and the outcomes they receive (e.g., pay, praise) and attempt to keep that ratio equitable to their perceived input/outcomes of others. For example, if a person felt that he was doing more work than a colleague but both were getting the same pay, the person would perceive an underpayment inequity. The theory states that workers seek to restore equity if they perceive either an underpayment or an overpayment inequity. They may, for example, attempt to increase or decrease their inputs, seek more compensation or other positive outcomes, or change their perceptions of the value of the inputs or outcomes involved.

In equity theory, the key factor influencing motivation is: A. the desirability of rewards for tasks and the expectancy that tasks will be successfully accomplished. B. the degree to which job tasks satisfy higher-order needs. C. perceived self and other input/outcome ratios. D. the degree to which workers believe that their input into the chosen goals of the organization is valued.

D. In escape conditioning, an organism learns to emit a response in order to terminate an aversive stimulus. An example would be applying an electric shock to a rat and terminating that shock when the rat presses a lever. In other words, the aversive stimulus cannot be avoided. By comparison, the first choice describes avoidance conditioning, whereby a discriminative stimulus (i.e., a cue such as a light flashing) is presented before the aversive stimulus is applied, and the organism can learn to avoid the pain before it hits. The escape conditioning paradigm has been applied to a number of real-life problems, such as procrastination, whereby beginning the task in question is viewed as the aversive stimulus from which the person escapes, and marital problems, whereby particular spousal interactions are aversive and escaped from, leading to distance and isolation

In escape conditioning, A. an aversive stimulus can be avoided by escaping from it in response to a discriminative stimulus. B. experimental neurosis is usually the result. C. learned helplessness is usually the result. D. the aversive stimulus is unavoidable.

D. Job enrichment involves redesigning a job in order to enable workers to have more challenge, responsibility, decision-making authority, and opportunities for advancement. Although the research findings are mixed, they generally have found that job enrichment leads to increased job satisfaction, decreased tardiness, absenteeism and turnover, and in some cases, increased job performance in terms of quality rather than quantity. The benefits of job enrichment are greatest among younger, well-educated, employees who have a high need for achievement

In general, research has found that job enrichment: A. has no significant effects on job satisfaction or performance. B. increases job satisfaction but has no effect on job performance. C. increases job performance and, to a lesser degree, job satisfaction for most workers. D. has positive effects on job satisfaction and, for some workers, job performance.

B. A trainability test is designed to determine whether or not potential employees are suitable for training. It is not designed to be directly predictive of how well the person will do on the job itself. Instead, it would more directly indicate how well the person would do on a job sample, which is likely to be a part of the training sessions. In fact, trainability tests typically include job samples and are described as a type of job sample

In industrial settings, administration of a trainability test would be done for the purposes of determining whether: A. the examinee is likely to do well on the job. B. the examinee is likely to do well on a job sample. C. the examinee will be satisfied on the job. D. the examinee is suited to train others.

A. Cognitive tests are the most valid predictors of job performance across jobs and settings-and job performance would include job continuity. However, biodata is particularily useful for predicting turnover. In the specific area of predicting turnover, biodata has been found to have validity coefficients of .77 and .79 for predicting the turnover of whites and African-Americans respectively (Cascio 1976). Hunter and Hunter (1984) report an average validity coefficient of .53 when cognitive tests are used to predict performance ratings and .75 when they are used to predict performance on a work sample

In order to decrease job turnover, an interviewer should: A. Rely on biodata B. Rely on cognitive testing C. Hire only older people D. Rely on an interest inventory

C. Habituation is defined as a decrease in response strength due to repeated stimulation. Satiation (D) occurs when a physiological need has been met, resulting in a decreased desire for that reinforcement. Extinction (B), in operant learning, involves eliminating reinforcement that previously followed a response. In classical conditioning, extinction refers to the presentation of a CS without the US. Neither of these two models adequately apply to this question.

In order to get an infant's attention, a photographer shakes a rattle, which results in the infant turning toward the rattle and smiling. After numerous uses of this technique, the infant stops responding. This is most likely due to: A. adaptation B. extinction C. habituation D. satiation

D. In a structured interview, a series of job-related questions with predetermined "correct" answers are used consistently with all interviewees for a particular job. An advantage of this technique is the provision of individual item scores and a total score that are derived from prespecified criteria. To predict job performance, the interview score can then be combined with scores on other selection procedures in a multiple regression equation or similar technique. Another advantage of the structured interview is that it reduces the impact of interviewer subjectivity (a.). A utility analysis (b.) considers the procedure's validity coefficient, employee job performance variability (typically in dollar value of output or mean output), and the selection ratio to evaluate the practical value of a selection procedure. Subject matter experts (c.) are one of several methods for determining scores to responses to a structured interview.

In structured interviews, interviewees responses are appraised by: A. interviewer subjectivity. B. utility analysis. C. subject matter experts' consensus. D. pre-specified criteria.

B. There is some evidence that interviewers place less importance on interview information when biodata is not very favorable and more importance when it is supportive of a hiring decision. Apparently, a good interview cannot make up for a bad history, but a good history can be supported or canceled out by the results of the interview

In terms of interviews as selection techniques, which of the following is most consistent with the research? A. Panel interviews generally have the highest levels of validity, and they are especially valid when an average (versus consensus) rating is derived. B. When interviewers are given biodata information about an interviewee prior to the interview, interviewers give less credence to interview information when the biodata is not supportive of a decision to hire than when the biodata is very supportive of a decision to hire. C. Although the research is inconsistent, the best conclusion about interviews is that future- oriented (situational) interviews are more valid than past-oriented (behavior description) interviews regardless of whether the interview is structured or unstructured. D. One of the difficulties with interviews, even when they are structured, is that they are highly susceptible to gender biases, especially when the interviewee and the interviewer are of different genders.

D. Burgess and Holmstrom (1974) described a rape trauma syndrome that occurs in two phases: acute and long-term reorganization. In the acute phase, women may display either an expressed style or a controlled style. Expressed behaviors include crying, hysteria, volatility, and tenseness. The controlled style, which the fourth choice refers to, involves masking feelings and shock behind calm, composed and subdued behavior. The acute phase lasts for several weeks to several months, and is followed by the long-term reorganization phase, which may last for years and involves an attempt to integrate the trauma into one's life and continue on. The second and third choices describe two possible characteristics of the long-term reorganization phase

In the acute phase of rape trauma syndrome, reactions are often characterized by: A. blackout of memories for most or all of the events that occurred before the rape. B. attempts to retake control of one's life by, for example, taking self-defense classes. C. sudden disruptions and changes in lifestyle. D. calmness and subdued behavior not typically associated with trauma survivors.

A. Job enrichment is a motivational technique that involves giving employees increased responsibility, decision-making authority, and autonomy. Job enrichment is designed to reduce boredom, and increase workers' feelings of meaningfulness, responsibility, and competence. It is based on Herzberg's two-factor theory of motivation, which classifies factors that affect job satisfaction into two categories: motivators, which increase satisfaction, and hygiene factors, which prevent dissatisfaction. Motivators are related to intrinsic conditions of the job itself, such as the autonomy and the opportunity for personal growth. In the question, the manager is increasing the responsibility and autonomy of the mechanics, so job enrichment is the best answer.

In the auto service department at a car dealership, managers talk to customers, diagnose problems with the cars, and assign repair tasks to mechanics. A new manager takes over so that mechanics interact directly with customers and decide on their own what work needs to be done. The new manager is using which of the following motivational techniques? A. job enrichment. B. job enlargement. C. quality circle. D. assessment center.

D. The aversive conditioning of a fetish involves pairing an aversive stimulus with the fetish object until the object also comes to elicit the response of aversion. The fetish object is the conditioned stimulus, since it comes to evoke aversion through conditioning (i.e., pairings). The aversive stimulus is the unconditioned stimulus, since it elicits aversion naturally, without conditioning

In the aversive counterconditioning of a fetish, the fetish object is the: A. unconditioned response B. unconditioned stimulus C. conditioned response D. conditioned stimulus

B. While no Ethical Standards have been violated at this point; there is a strong indication that you are experiencing some type of "extra attraction" toward this client. You need to seek consultation and determine how to process and handle your countertransference. Cancelling the session may not be in the client's best interest.

In the last session, your gorgeous new client complimented you on how nice you look in your red outfit. You noticed you were very happy before this session-you even caught yourself singing, "Oh What a Beautiful Morning" while looking in the mirror to do a final 'visual check" before seeing your client in. Ethically, A. You need to cancel this session B. You need to consult with a colleague C. You need to behave more appropriately D. You need to get out more

D. Use of a multitrait-multimethod matrix is one method of assessing a test's construct validity. The matrix contains correlations among different tests that measure both the same and different traits using similar and different methodologies. The heterotrait-monomethod coefficient, one of the correlation coefficients that would appear on this matrix, reflects the correlation between two tests that measure different traits using similar methods. An example might be the correlation between a test of depression based on self-report data and a test of anxiety also based on self-report data. If a test has good divergent validity, this correlation would be low. Divergent validity is the degree to which a test has a low correlation with other tests that do not measure the same construct. Using the above example, a test of depression would have poor divergent validity if it had a high correlation with other tests that purportedly measure different traits, such as anxiety. This would be evidence that the depression test is measuring traits that are unrelated to depression.

In the multitrait-multimethod matrix, a large heterotrait-monomethod coefficient would indicate: A. low convergent validity. B. high convergent validity. C. high divergent validity. D. low divergent validity.

D. Use of a multitrait-multimethod matrix is one method of assessing a test's construct validity. The matrix contains correlations among different tests that measure both the same and different traits using similar and different methodologies. The heterotrait-heteromethod coefficient, one of the correlation coefficients that would appear on this matrix, reflects the correlation between two tests that measure different (hetero) traits using different (hetero) methods. An example might be the correlation between vocabulary subtest scores on the WAIS-III for intelligence and scores on the Beck Depression Inventory for depression. Since these measures presumably measure different constructs, the correlation coefficient should be low, indicating high divergent or discriminant validity.©

In the multitrait-multimethod matrix, a low heterotrait-heteromethod coefficient would indicate: A. low convergent validity B. low divergent validity C. high convergent validity D. high divergent validity

A. In cognitive-behavioral therapy (CBT) for Panic Disorder, a central focus is on misinterpretation of physical symptoms. From the perspective of CBT, catastrophic interpretations increase fear, leading to more physical reactions and thus causing a feedback loop that spirals out of control and causes panic attacks. One intervention in CBT for Panic Disorder is to induce physical symptoms such as tachycardia, while preventing the person from reacting to the symptoms. The idea is to expose the person to the symptoms, so she habituates to them and learns that they are nothing to fear. In classical conditioning terms, this can be viewed as classical extinction. In classical conditioning, an unconditioned stimulus (US), or a stimulus that naturally produces a response, is paired with a conditioned stimulus (CS) until the CS, when presented alone comes to elicit the same response as the US (conditioned response, or CR). Classical extinction involves eliminating the CR by "de-associating" the CS and the US; i.e., the CS is repeatedly presented by itself, not paired with the US, until it no longer elicits the CR. From the CBT perspective, in Panic Disorder, the US is an initial sense of anxiety and physical symptoms such as tachycardia are conditioned stimuli. Therefore, exposing the person to physical symptoms when anxiety is not present (i.e., the CS without the US) can be conceptualized as classical extinction

In the treatment of panic disorder, a person is asked to run up and down the stairs in order to induce tachycardia. The mechanism by which this intervention would be curative is: A. extinction. B. stimulus generalization. C. stimulus discrimination. D. higher-order conditioning.

A. The behavioral technique described in this question is referred to as "paradoxical intention" and is similar in form to the paradoxical techniques used by family therapists, but differs in terms of its theoretical underpinnings. As used by behavior therapists, paradoxical intention is based on the assumption that a person avoids a certain behavior because of the anticipatory anxiety the behavior arouses. When a person deliberately engages in the behavior, a condition of incompatibility is set up and this condition serves to eliminate the anticipatory anxiety. The idea is that engaging in the behavior (or a wish to engage in the behavior) is incompatible with fear of that behavior and, consequently, the fear is neutralized

In treating a client suffering from an Anxiety Disorder, a behavior therapist instructs the client to deliberately engage in the anxiety-arousing behavior as often as possible. The therapist's goal in this situation is most likely to A. neutralize the anticipatory anxiety that is the real source of the client's problem. B. eliminate the client's usual avoidance behavior by evoking his resistance. C. extinguish the anxiety-arousing behavior by removing the reinforcement that is maintaining it. D. diminish the client's anxiety through interoceptive conditioning.

B. Depression is associated with decreased slow-wave or non-REM sleep as well as early morning awakening, decreased sleep continuity, and decreased REM latency (earlier onset of REM sleep)

Individuals with Major Depressive Disorder who experience abnormalities in the sleep cycle are least likely to experience which of the following? A. sleep continuity decrease B. slow-wave sleep increase C. REM latency decrease D. early morning awakening

B. Pooled interdependence occurs when individuals have a shared source of resources but little else in common; sequential involves a series or chain of one-way interactions in which individuals rely on those who precede them in the chain; reciprocal is described in the question and comprehensive is the most complex form as everyone involved is reciprocally interdependent on one another. While a greater potential for conflict and the loss of individuals due to turnover becomes more important as the intensity and complexity of interdependence increases from pooled to comprehensive, comprehensive interdependence can stimulate greater flexibility and enable groups to adapt more quickly to environmental changes

Interdependence in the workplace usually takes one of four forms. A network of two-way relationships that tie a collection of people together describes which type of organizational interdependence? A. comprehensive B. reciprocal C. pooled D. sequential

A. The procedure used to determine what items will be retained for the final version of a test is the definition of item analysis. The degree to which items discriminate among examinees is the definition of Item Discrimination. A graph that depicts percentages of people is an item characteristic curve.

Item analysis is a procedure used to: A. Determine which items will be retained for the final version of the test B. Refer to the degree to which items differentiate among examinees C. Graph depictions of percentages of people D. Help the IRS with an audit

A. Research has found that people are more likely to recall the first item (primacy effect) and the last item (recency effect) in a series of presented items. The relative magnitude of these two effects varies depending on the length of the time interval. Within the first few seconds after presentation of a series of items, the recency effect is more powerful. However, for longer intervals, the primacy effect has been shown to be stronger

Jane describes her friend Mike to her other friend Mary, as "intelligent, hard-working, stubborn, and critical." A week later Mary is most likely to remember Mike as: A. intelligent B. hard-working C. stubborn D. critical

C. The term "job evaluation" refers to procedures used within a workplace to establish the relative worth of particular jobs and tasks within the organization. If a job evaluation were biased against work typically performed by women, this would mean that the value of such work to the organization would be unfairly set too low. The term "comparable worth" refers to the standard that wages for jobs with different titles and/or duties should be based on their actual worth to the organization rather then the gender makeup of the workers in those jobs. In other words, a gender biased job evaluation would contradict the standard of comparable worth. Regarding the other choices, employment equity refers to multidimensional efforts to promote fair representation of minority groups in the workplace--not only through hiring practices but also through retention, promotion, and training efforts. Procedural justice refers to perceptions of fairness in the processes that resolve disputes and allocate resources. It is not as good an answer as comparable worth because it usually refers to perceptions rather than actual practice and because comparable worth refers specifically to the undervaluation of tasks that women typically perform. A bias in a job evaluation method relates more to distributive justice, or fairness in outcomes, because a job evaluation is typically used to establish salary. And "job fairness" is a broad term that generally means that distribution of workplace rewards, such as salaries and selection for jobs, is based on merit rather than some form of bias.

Job evaluation methods have been criticized for containing biases against work typically performed by women. This criticism most directly pertains to the standard of: A. employment equity. B. procedural justice. C. comparable worth. D. job fairness.

Confirmatory bias

Journal reviewers who show strong bias against manuscripts that report results contrary to their theoretical perspective are demonstrating: A. self-serving bias B. confirmatory bias C. fundamental attribution bias D. self verification theory

B. Validity generalization, or generalized evidence of validity, is evidence of validity that generalizes to setting(s) other than the setting(s) in which the original validation evidence was documented and is accumulated through strategies such as synthetic validity/job component validity (a.), transportability (c.) and meta-analysis (d.). Synthetic validity/job component validity is based on previous demonstration of the validity of inferences from scores on the selection procedure or battery with respect to one or more domains of work (job components). Transportability refers to a strategy for generalizing evidence of validity in which demonstration of important similarities between different work settings is used to infer that validation evidence for a selection procedure accumulated in one work setting generalizes to another work setting

Justification for the use of a selection procedure or battery in a new setting without conducting a local validation research study is referred to as: A. synthetic validity. B. validity generalization. C. transportability. D. meta-analysis.

A. Kernberg's explanation for Borderline Personality Disorder attributes it to contradictory caregiver behaviors that cause the individual to rely on the primitive defense mechanism of splitting, which involves having two contradictory perceptions of the self and others as all good or all bad with an inability to integrate those perceptions.

Kernberg's explanation for __________ Personality Disorder attributes it to reliance on the primitive defense mechanism of "splitting." A. Borderline B. Antisocial C. Histrionic D. Obsessive-Compulsive

D. The Kuder-Richardson formula is one of several statistical indices of a test's internal consistency reliability. It is used to assess the inter-item consistency of tests that are dichotomously scored (e.g., scored as right or wrong).

Kuder-Richardson reliability applies to: A. split-half reliability. B. test-retest stability. C. Likert scales. D. tests with dichotomously scored questions.

C. The classic finding in the area of research into short-term memory (STM) is the "7 +/- 2" finding; i.e., that STM has the capacity of storing 7 plus or minus 2 pieces of information at one time. Chunking is a technique designed to increase the amount of information that can be kept in STM -- it involves grouping larger amount of information into single chunks. In other words, STM is used to store 7 +/- 2 chunks rather than 7 +/- 2 isolated bits of information. Mnemonics (choice D) are used to increase the efficiency of encoding newly learned information, which increases the ease with which that information is retrieved from long-term memory.

Large amounts of information can be kept in short-term memory through the use of: A. the "savings" method. B. eidetic memory. C. chunking. D. mnemonics.

A. Social loafing or Latane's "social disease" is the discovery that in regard to work, individual output declines when people are working together as a group. However, social loafing does not occur under all conditions. It is reduced or eliminated when participants believe that their individual contributions are identifiable or uniquely necessary for the group to succeed. (B) "groupthink" is the tendency for a group to make an irrational or impulsive decision in order to reach consensus. (C) "Group polarization" refers to the tendency of individuals who start with a similar view to end with a more extreme position after group discussion.©

Latane called this a social disease. It occurred when high-level employees were assessed on a combined effort. It turned out they produced less than when they were working individually. It's called: A. Social loafing B. Group think C. Social polarization D. Social facilitation

A. Leader member exchange theory (LMX) emphasizes that the nature of the relationship between a leader and member is the determining factor as to whether a member belongs to the "in-group" or the "out-group" and that not all members of the organization achieve the same quality relationship. Improved organizational effectiveness and achieving leader/employee needs satisfaction is the general objective. LMX theory suggests a developing leader-member relationship that moves through three phases: stranger, acquaintance, and partner. The relationship between the leader and members of the "out-group" is marked by a lower quality with both parties only completing their formal role obligations. Whereas, the relationship with the "in-group" members moves from low quality, one way, self-oriented and scripted to high quality, reciprocal, group oriented and negotiated exchanges. Given their status with the leader, in-group members have more decision-making influence, access to resources and responsibility than those in the out-group. The leader, in exchange for the additional effort of these members, provides more support, trust and initiative beyond the obligations of the job. Research indicates LMX theory is positively related to "higher job satisfaction, stronger organizational commitment, and better subordinate performance" (Yukl, 1998). LMX theory has been criticized as being discriminatory (i.e., increased opportunity for one group and formal expectations for another) and counter to principles of fairness among subordinates. The theory does not propose inequitable treatment of members rather it explains how relationships with various members can develop. LMX research suggests that leaders become cognizant of their attitudes toward all members and create methods for all employees to freely enter the in-group.

Leader-Member Exchange Theory (LMX) emphasizes: A. the relationships a leader has with members can be divided into two groups referred to as the "in-group" and the "out-group." B. adhering to formal organizational expectations yields increased access and opportunity C. principles of fairness in members achieving a quality relationship with a leader D. leaders and members use exchanges to achieve mutually beneficial outcomes

C. Kurt Lewin's process of organizational change involves three stages: unfreezing, changing, and refreezing. In 1980, Huse added several steps to Lewin's model, with entry and planning (d.) being added to the unfreezing stage and action was added as the primary step in the changing stage. Collecting data, providing feedback, and implementing or developing action plans are the three steps of survey feedback.

Lewin's (1951) model of organizational change includes the following three stages: A. data collection, feedback and action planning. B. planning, action, and evaluation. C. unfreezing, changing and refreezing. D. entry, planning and action.

D. In 1951, Kurt Lewin and his colleagues conducted a study assessing the effectiveness of three different leadership styles: authoritarian, where the leader remains aloof and uses orders to direct the group's behavior; democratic, where the leader actively encourages group members and participates in their activities; and laissez-faire, where the leader provides knowledge but does not direct or participate in activities. The subjects were 10-year-old boys participating in an arts and crafts club. A key finding was that the laissez-faire style was the least successful, resulting in significantly lower quality and quantity of work as compared to the other two styles. Other findings were that groups with democratic leaders produced the highest quality of work and its members were the most satisfied, and that autocratically led groups produced the highest quantity of work. Some children did prefer the autocratic style, so choice 3 would have been a better answer if laissez-faire was not included--laissez-faire style is not successful regardless of the personality of the group members

Lewin's classic study on leadership styles produced which of the following findings? A. Democratic groups produce a higher quantity of work than either autocratic or laissez-faire groups. B. In autocratic groups, the leader's absence does not affect the quality or quantity of work. C. The success of all three leadership styles studied--democratic, autocratic, and laissez-faire--depended on the personality of group members. D. Laissez-faire was the least successful leadership style in terms of both quantity and quality of work output.

B. According to Linehan, individuals with Borderline Personality Disorder have stronger emotions than other people that they are unable to control, and this emotion dysregulation contributes to all other problems associated with this disorder.

Linehan's (1993) biosocial theory proposes that the core feature of Borderline Personality Disorder is: A. behavioral inhibition. B. emotion dysregulation. C. reliance on primitive defenses. D. identity diffusion.

C. Linehan developed dialectical behavioral therapy (DBT) as a treatment for Borderline Personality Disorder. It combines group skills training, individual outpatient psychotherapy, and between-session telephone consultations.

Linehan's dialectical behavioral therapy combines cognitive and behavioral strategies and involves: A. in-patient individual and group therapy. B. individual and family therapy. C. individual and group sessions and telephone consultations. D. intensive individual sessions.

D. According to Fiedler's 1971 contingency theory, a leader's effectiveness is determined by a combination of the leader's style and the characteristics of the situation. Low LPC leaders are task and achievement oriented. Answers A and B could describe High LPC leaders who are primarily relationship oriented. Answer C could describe a transformational leader which Bass (1990) (see I/O section) described as change focused.

Low LPC leaders would be most likely to say: A. More beautiful paint colors in this office make us all feel better B. Flexitime was a brillant invention C. It's time for a change around here D. The first one to hand in this budget report wins the office lottery pool

A. Rater biases include the strictness, leniency, central tendency, and halo biases. The central tendency bias is the tendency to use the middle of a scale in rating all ratees. The halo effect (b.) occurs when evaluations of one aspect of a person's behavior influence evaluations of other aspects. The halo effect can be positive or negative; i.e., it can result in a bias in favor of or against a person. You may have considered the leniency bias (c.) as the correct response. This type of rater bias involves giving ratees high ratings on each dimension of performance regardless of their actual performance. However, the question does not indicate that all the ratings are high, only that Margaret is reluctant to give low scores for poor performance. Severity error (d.), or strictness bias, refers to the tendency to use the low end of a rating scale to rate all ratees

Margaret is reluctant to give poor ratings to her subordinates for fear of "ruffling feathers." Consequently, she tends to be unusually easy in her ratings. Margaret is committing what type of error? A. central tendency B. halo C. leniency D. severity

A. The three overlapping stages of stress inoculation training are sometimes referred to by similar but varied terms. In general, the first stage is referred to as the conceptualization, education, or cognitive phase; the second stage is referred to as the skills application or training phase or the skills acquisition and rehearsal phase; and the third phase is referred to as the application or application and follow-through phase. Response choice "a." lists the steps in Meichenbaum's self-instructional training and response choice "d." lists the three objectives of Rehm's self-control therapy

Meichenbaum's stress inoculation training was created for the purpose of helping people cope more effectively with stress by increasing their coping skills. This technique involves three stages including: A. conceptualization, skills training, and application. B. cognitive modeling, overt instruction, and covert instruction. C. formulation, problem focus, and termination. D. self-monitoring, self-evaluation, and self-reinforcement.

A. Group cohesiveness has been associated with less absenteeism and turnover and, in some situations, greater productivity. You should have been able to eliminate "C" (better decision-making) if you recalled the research on groupthink, which is characterized, in part, by high group cohesiveness, and is associated with poorer quality decisions.

Members of a cohesive group, as compared to members of a non-cohesive group, are more likely to have: A. lower absenteeism B. lower productivity C. better decision-making D. higher turnover

D. Episodic memory consists of autobiographical memories or memories for personal events. Semantic memory (response "A") contains memories of general knowledge. Episodic and semantic memory are the two divisions of declarative memory, one of the two types of long-term memory. The other type of long-term memory is procedural. Procedural memory (response "B") contains memories of skilled responses and actions. Implicit memory (response "C") refers to memories that are recalled without conscious effort.

Memories of one's personal experiences are contained in which aspect of long-term memory: A. semantic B. procedural C. implicit D. episodic

C. Eagly and colleagues conducted a series of meta-analyses on the effects of gender on leadership. They found that overall, male and female leaders are equally effective. However, certain gender differences in leadership were found to exist. For example, female leaders were more effective when the leadership role was defined as "feminine" and males were more effective when the role was defined as "masculine." Contrary to D, males were found to be more effective in first-level management positions and females were more effective in middle management positions. One interpretation of this finding is that lower-level management requires more technical skills, whereas middle-level management requires more human relations skills

Meta-analyses on gender differences in leadership have found that female leaders are: A. more effective than male leaders overall B. less effective than male leaders overall C. equally effective as male leaders overall D. more effective than male leaders in first-level management positions, but males are more effective in middle management positions

C. Metacognition refers specifically to knowledge we have about our own cognitive processes, rather than about the nature of human knowledge in general.

Meta-cognition has been defined as "knowing about knowing." A more precise definition might be: A. philosophical knowledge about the nature of knowledge, such as about limits of knowledge and the validity of our observations. B. scientific knowledge about human knowledge, such as the parts of the brain involved in long-term memory storage and the stages of cognitive development. C. an individual's knowledge about his or her own cognitive processing, such as strategies that enhance the efficiency of memorization. D. knowledge about what somebody else knows.

D. As you can imagine, what people do (or don't do) is the leading cause of industrial accidents. Not paying attention, daydreaming, not fixing the machine correctly, not training properly, etc., all can lead to accidents. It follows that to correct this problem one would want people trained better and one would want an environment in which attention is paid to possible accident-causing conditions.

Most studies of correlates of industrial accidents indicate that the strongest association is with: A. machine malfunction. B. organizational ambiance. C. supervisory attitude. D. human error.

A. While group differences exist between African-Americans and Caucasians on test performance (that is, while mean scores differ on these various standardized tests), it has not been shown conclusively that the predictive validity coefficients of the tests are differentially moderated by ethnic group membership. In other words, a test is likely to have the same validity coefficient for all subgroups of the same population -- even though those subgroups may score differently, on the average, from each other.

Most well-controlled studies of the effect of cultural moderators on the validities of job selection tests indicate that there are: A. no significant differences in validities between majority and minority groups. B. significant differences in validities between majority and minority groups. C. significant differences in validities between majority and minority groups on general aptitude tests but not on assessments of motor skills. D. significant differences in validities between majority and minority groups on assessments of motor skills but not on tests of general aptitude.

B. Evolutionary psychology posits a woman's waist-to-hip ratio communicates reproductive information (e.g., greater fertility and health) to potential mates. Related research from varied countries supports the suggestion that men tend to prefer women whose waist circumference is about 70 percent of their hip circumference.

Multi-national research findings show men generally prefer women with a waist-to-hip ratio of approximately: A. .85 B. .70 C. .45 D. .30

C. Narcolepsy involves irresistible sleep attacks of brief duration during the day, and Hypersomnolence Disorder is characterized by excessive sleepiness and sleep that may include unintentional daytime naps.

Narcolepsy and Hypersomnolence Disorder share which of the following symptoms? A. sleep paralysis B. cataplexy C. daytime sleep D. hypnagogic hallucinations

A. Narcolepsy is characterized by irresistible sleep attacks occurring on a daily basis for at least three months. It may involve cataplexy, which is a sudden loss of muscle tone, usually in association with intense emotion.

Narcolepsy is often associated with: A. loss of muscle tone. B. rigid muscle tone. C. decreased REM sleep. D. insomnia.

A. The concept of negative reinforcement involves increasing the likelihood of a behavior through the elimination of an aversive stimulus (the negative part). As you might recall, this is exactly what the dog does by learning to escape the electric shock by jumping to the other side of the cage. This is the classic example of escape conditioning. Jumping to the other side is reinforced by the elimination of the aversive stimulus. You may have chosen punishment, but this is the common error of confusing negative reinforcement with punishment. To avoid this error, keep in mind that reinforcement (whether negative or not) and punishment are antithetical.

Negative reinforcement is essential to: A. escape conditioning. B. punishment. C. aversion conditioning. D. phobias.

B. Normal aging has been found to most affect episodic memory which contains memory for specific events including one's autobiography. Semantic memory (a.) and remote long-term memory (c.) are not affected by normal aging. Cystalized intelligence (d.), the ability to recall and use learned information, remains stable or may increase during mid-adult years. In contrast, fluid intelligence, the ability to solve problems, form concepts, and see complex relationships, decreases with advancing age.

Normal aging is most likely to negatively affect: A. semantic memory B. episodic memory C. remote long-term memory D. crystalized intelligence

D. The question describes Krumboltz's Social Learning Theory of Career Decision Making (SLTCDM) which includes four major factors of influence on career decisions: genetic characteristics and special abilities; environmental conditions and events; learning experiences; and task approach skills, including performance expectations, values, work habits, and emotional responses. The social learning influences can be positive or negative factors. Holland (a.) and Roe (c.) both emphasized the importance of matching the individual's personality to the characteristics of the job although Roe also focused on needs. Herzberg's (b.) two-factor theory addresses worker motivation and satisfaction, not making a career choice.

Of the following, which proposed a social learning theory of career development in which career path is influence by the factors: genetic endowment; environmental conditions and events; learning experiences; and performance standards and values? A. Holland B. Herzberg C. Roe D. Krumboltz

B. A T-score is a standardized score. Standardization involves converting raw scores into scores that indicate how many standard deviations the values are above or below the mean. A T-score is a standard score with a mean of 50 and a standard deviation of 10. Results of personality inventories such as the MMPI-2 are commonly reported in terms of T-scores. Other standard scores include z-scores, with a mean of 0 and a standard deviation of 1, and IQ scores, with a mean of 100 and a standard deviation of 15. When values are normally distributed in a population, standardization facilitates interpretation of test scores by making it easier to see where a test-taker stands on the variable in relation to others in the population. This is because, due to the properties of a normal distribution, one always knows the percentage of cases that are within standard deviation ranges of the mean. For example, in a normal distribution, 68.26 % of scores will fall within one standard deviation of the mean, or in a T score distribution, between 40 and 60, so 68% is the best answer to this question. Another example: 95.44% of scores fall within two standard deviations of the mean; therefore, 4.56% will have scores 2 standard deviation units or more above or below the mean. By dividing 4.56 in half, we can see that 2.28% of test-takers will score 70 or above on any MMPI scale, and 2.28% will score 30 or below

On the MMPI-2, what percentage of the general population the test is intended for can be expected to obtain a T-score between 40 and 60 on the depression scale? A. 50 B. 68 C. 95 D. 99

A. While there is no current consensus over the dimensions of justice in the organizational justice literature, two meta-analyses that used three and four-construct concepts found overlapping results on justice dimensions that predicted different outcomes. Cohen-Charash & Spector (2001) used a three-construct concept (distributive, procedural, and interactional) of justice to investigate the role of justice in organization outcomes. (See: Cohen-Charash, Y. & Spector, P.E. (2001). The Role of Justice in Organizations: A Meta-Analysis. Organizational Behavior and Human Decision Processes, Vol 86; 2, November, 278-321.) Whereas Colquitt et al.'s (2001) meta-analysis reviewed 25 years of organizational justice research focusing on the operationalizations of justice using the dimensions of distributive, procedural, informational, and interpersonal justice. Despite the differences, results from both indicated procedural justice is the best predictor of work performance and counterproductive work behavior. Organizational citizenship behaviors were found to be related to all dimensions of justice in both studies and noted in Colquitt et al as support for interactional justice to be distinguished as two specific types of interpersonal treatment: interpersonal and informal justice.

Organizational justice research indicates: A. procedural justice is the best predictor of work performance and of counterproductive work behavior, but all justice forms are related to organizational citizenship behaviors. B. distributive justice is the best predictor of work performance and counterproductive work behavior, but all justice forms are related to organizational citizenship behaviors. C. procedural and distributive justice are the best predictors of work performance and counterproductive work behavior, but interactional justice is the best predictor of organizational citizenship behaviors. D. procedural and distributive justice are the best predictors of work performance, counterproductive work behavior, and organizational citizenship behaviors.

A. Utility analysis is a procedure used to translate the cost and benefits of different business alternatives into dollars and cents. In this case, an I/O psychologist would use utility analysis to compare how much money the company currently makes without using the psychological tests and how much money the company would be expected to make if a testing program were implemented. A job analysis (c.) is a detailed analysis and description of a job that specifically states what needs to be done (job-related tasks), the nature of the work (work environment), and the type of individual who can perform the work successfully (knowledge, skills, and abilities). Job evaluation (d.) is the process used to determine how much to pay each employee. A typical job evaluation process involves assessing the value of each job-related task to the business and then adding up the value of all tasks listed in the job description to determine the overall value of the job.

Organizational psychologists would most likely use which of the following when talking to business leaders about implementing the use of psychological tests in selecting employees? A. utility analysis B. utility evaluation C. job analysis D. job evaluation

C. Ouchi's Theory Z is an organizational management philosophy that incorporates aspects from traditional American (Theory A) and Japanese (Theory J) management philosophies. The theory represents a middle ground, for example, emphasizing long-term employment versus short-term or lifelong and a moderately specialized career path instead of specialized or nonspecialized.

Organizations that advocate individual responsibility, consensual-decision making, slow promotion, and holistic knowledge of the organization are using which of the following management philosophies: A. Theory J B. Theory A C. Theory Z D. TQM

internal locus of control

People with ____________ tend to view positive and negative outcomes as the result of their own actions; i.e., they believe they have control over their own fate (Rotter). People with ___________ of [responsibility] credit or blame themselves for what happens to them (D. W. Sue).

C. In general, clinic programs are more successful in reducing teen pregnancy than school programs. Also, in terms of program content, focusing on contraceptive use and distributing contraceptives seems best.

Overall, the research on methods for reducing teen pregnancy has shown that the best approach is: A. a school program that focuses on the risks associated with unprotected sex. B. a school or clinic program that addresses the consequences of early parenthood and includes teen mothers as speakers. C. a clinic program that provides information about contraceptive use. D. a clinic program that advocates sexual abstinence.

B. Landy's (1978) Opponent Process Theory of job satisfaction hypothesizes that job attitudes emanate from a person's physiological state. Opponent process theory assumes that when a person experiences an extreme emotional state, his or her central nervous system mechanisms attempt to bring him or her back to a state of emotional equilibrium or neutrality. In returning to neutrality, the emotional state may even surpass equilibrium and progress to the opposite emotional state. In this case, Patrick initially felt elated when informed of the praise then when the positive emotional state waned over time it continued to the opposite emotional state of anger. (See: Landy, F. J. (1978). An opponent process theory of job satisfaction. Journal of Applied Psychology, 63, 533-547.) Lawler's (1973) Model of Facet Satisfaction (a.) posits that job satisfaction is conceived in terms of different facets of an individual's job; a compilation of feelings of satisfaction on an array of situational, biological, social and educational factors. Lawler's model specifies that workers compare what their jobs should provide in terms of job facets to what they currently receive from their jobs. Examples of facets include: gender, age, race, work load, job security, working conditions, workplace location, supervisor-subordinate relations, status and prestige of job, compensation, etc. (See: Lawler, E.E. (1973), Motivation in Work Organizations, Brooks/Cole, Monterey CA.). Warr's (1987) Vitamin Model (c.) of work and mental health suggests that certain features in the workplace act in much the same way as vitamins. The level and combination of stressors have an effect on well-being and as with vitamins, some of the features start to have a negative effect if they increase beyond a recommended daily dose. The nine proposed stressors include: opportunity for control, opportunity for skill use, externally generated goals, variety, environmental clarity, availability of money, physical security (freedom from physical threat or danger), opportunity for interpersonal contact and valued social position . There is substantial empirical support for the particular stressors identified by Warr as he developed the stressor content of his model by reviewing the empirical literature on workplace stressors. (See: Warr, P. (1987). Work, Unemployment, and Mental Health, Clarendon Press, Oxford.). The job satisfaction/job dissatisfaction (d.) theory is the basis of Herzberg's concept of job satisfaction which distinguishes two separate groups of factors influencing individual job satisfaction and dissatisfaction. The first group, called "motivators," leads to job satisfaction; the second group, called "hygienes," leads to job dissatisfaction. Motivator or "intrinsic" factors of job satisfaction include achievement, recognition, the work itself, and the intrinsic interest of the job. Hygiene or "extrinsic" factors of the job include pay, job security, working conditions, policy and administration, and relationships with peers and supervisors.

Patrick was elated, in a meeting with senior management, when he was told that the company president was impressed with his recent contributions. After he left the meeting and was alone in his office, however, Patrick started to experience feelings of anger. Which of the following best explains Patrick's circumstances? A. Model of Facet Satisfaction B. Opponent Process Theory C. The Vitamin Model D. Job Dissatisfaction Theory

A. decreases in the presence of others if the desired behavior is not well-learned

Performance generally: A. decreases in the presence of others if the desired behavior is not well-learned. B. increases in the presence of others if the desired behavior is not well-learned. C. decreases in the presence of others if the desired behavior is well-learned. D. remains unchanged in the presence of others.

B. The prefixes ("pro" and "retro") indicate which material is being interfered with. Proactive interference or inhibition occurs when previous learning interferes with new learning. Retroactive interference or inhibition occurs when new learning interferes with previous learning. Retrograde amnesia (C) is a failure to remember events that occurred prior to a trauma. And anterograde amnesia (D) is an impaired ability to learn new information.

Poor recall for information learned yesterday as the result of learning that took place last month is referred to as: A. retroactive interference B. proactive interference C. retrograde amnesia D. anterograde amnesia

B. Depending on the goals, prevention programs are frequently described as either primary, secondary, or tertiary. Primary preventions (b.) focus on the prevention of developing a physical or mental disorder. Secondary preventions (c.) have the goal of reducing the prevalence of mental disorders through early identification and intervention. Tertiary preventions (d.) strive to reduce the prevalence of mental disorders by reducing their duration or preventing their recurrence. Crisis interventions (a.) are utilized to reduce the negative consequences of crises

Programs designed to prevent the development of a physical or mental disorder are described as: A. crisis intervention. B. primary prevention. C. secondary prevention. D. tertiary prevention.

B. A simple way to answer this question is with reference to a chart such as the one displayed under the topic "Criterion-Related Validity" in the Test Construction section of your materials. If you look at this chart, you can see that increasing the predictor cutoff score (i.e., moving the vertical line to the right) decreases the number of false positives as well as true positives (you can also see that the number of both true and false negatives would be increased). You can also think about this question more abstractly by coming up with an example. Imagine, for instance, that a general knowledge test is used as a predictor of job success. If the cutoff score on this test is raised, fewer people will score above this cutoff and, therefore, fewer people will be predicted to be successful. Another way of saying this is that fewer people will come up "positive" on this predictor. This applies to both true positives and false positives.

Raising the cutoff score on a predictor test would have the effect of: A. increasing true positives B. decreasing false positives C. decreasing true negatives D. decreasing false negatives.

D. A topic of recent interest in the I/O literature is the difference between leaders and managers. Characteristics that are considered particularly important for leaders include: visionary, creative, flexible, inspiring, courageous, and independent.

Recent literature comparing leadership and management has identified several similarities and differences. Which of the following set of characteristics is more critical for today's leaders than for managers? A. rational, persistent, and tough-minded B. problem-solving, persistent, and independent C. visionary, authoritative, and tough-minded D. independent, innovative, and flexible

B. According to APA's Record Keeping Guidelines (1993, 48 (9), 984-986) records should minimally include: identifying data, dates and types of service, fees, any release of information obtained, any assessment, plan for intervention, consultation, summary reports, and/or testing reports. Process notes, which contain intimate details of psychotherapy sessions, including the psychotherapist's personal notes, are generally considered inappropriate for the medical record and are not recommended for minimal inclusion in the Guidelines

Records of psychological services should minimally include all of the following except: A. fees B. process notes C. types of services provided D. plans for treatment interventions

B. Research on the effects of the CWW is mixed. A meta-analysis by Baltes and colleagues (1999) found the CWW to have positive effects on supervisor ratings of employee performance and employee overall job satisfaction.

Research about job experience with the four day workweek suggests that it is correlated with: A. supervisor ratings of employee performance only B. supervisor ratings of employee performance and employee overall job satisfaction C. job performance only D. job performance and absenteeism

D. Foa et. al. (1999) compared prolonged exposure, stress inoculation training, combined prolonged exposure and stress inoculation training, and a wait list control group. The results indicated that all three treatments were effective for alleviating the symptoms of PTSD. However, prolonged exposure was superior to either stress inoculation training or the combined treatment.

Research by Foa et al. (1999) found which of the following treatments most effective for Posttraumatic Stress Disorder? A. implosive therapy B. stress inoculation training C. stress management training D. prolonged exposure

D. ....Anxiety is the most common psychiatric disorder in older adults, with Generalized Anxiety Disorder being the most prevalent anxiety disorder. Research indicates comorbid symptoms of depression frequently occur in all adults with anxiety, but occurs more often in older adults.

Research comparing anxiety over the lifespan indicates: A. younger adults are more likely to be underdiagnosed and older adults more likely to be misdiagnosed as having an anxiety disorder. B. younger adults are more likely to benefit from cognitive-behavioral therapy, whereas older adults are more likely to benefit from pharmacotherapy. C. younger and older adults are equally likely to perceive symptoms as result of physical health problems. D. younger adults are less likely to have comorbid symptoms of depression than older adults.

B. As noted in the DSM-5, individuals who were sexually or physically abused as children are at greater risk for Bulimia

Research investigating the relationship between sexual abuse in childhood and the development of Bulimia Nervosa in adulthood has found: A. a significant relationship between sexual abuse and Bulimia for men but not for women. B. a higher rate of Bulimia for individuals who were sexually abused as children C. the same rate of Bulimia for individuals who were sexually abused and individuals in the general population. D. no significant relationship between childhood sexual abuse and Bulimia.

C. Only one of the responses is consistently cited by studies as a primary or most frequently mentioned barrier and that is factors related to children. Research has confirmed that women with substance abuse problems are more likely than men to be caring for dependent children. Many women are concerned about custody issues; others are concerned about childcare while they receive treatment. According to a recent United Nations report on substance abuse treatment, "lack of childcare is probably the most consistent factor restricting women's treatment access identified in the literature" [United Nations Office on Drugs and Crime, Substance Abuse Treatment and Care for Women: Case Studies and Lessons Learned, New York, United Nations, 2004, p. 18]. While sexual harassment (d.) of women has been identified as a problem at some treatment centers and some women may be deterred by concerns over treatment effectiveness (a.), neither has been identified as a primary barrier to treatment. Women are more likely to express shame and guilt about their substance abuse than to deny that they have an abuse problem (b.).

Research has identified the following factors as barriers that reduce the likelihood that a woman will seek or remain in substance abuse treatment. Of the following, which has been most often cited as a primary barrier to substance-abuse treatment for women: A. a belief that treatment is ineffective B. denial of a substance abuse problem C. child and childcare concerns D. sexual harassment from male counselors

C. Elaborative rehearsal is a method for improving long-term memory that makes new information more meaningful by associating it with previously acquired or learned information. Research indicates elaborative rehearsal is an effective way for ensuring that material is transferred from short-term memory to long-term memory.

Research indicates elaborative learning is most effective for: A. transferring sensory input from sensory memory to short-term memory. B. temporary storage of incoming sensory signals. C. transferring information from short-term memory to long-term memory. D. maintaining information in working memory.

D. Summarizing the predictive validity of interviews and other selection techniques in a meta-analysis of the research, Schmidt and Hunter (1998) report measures of general mental ability are the most valid predictors across different jobs (a.). For structured and unstructured interviews, they note corrected validity coefficients of .51 and .38, respectively (b.). When an interview is combined with another selection procedure, especially a general mental ability test, the predictive validity of interviews is increased

Research indicates the validity of structured interviews as predictors of performance: A. are the most valid across different jobs. B. is increased when the interview includes both structured and unstructured items. C. are no more valid than unstructured interviews when used alone. D. is increased when the interview is used in combination with a measure of general mental ability.

D. Research indicates that intellectual functioning, age, and somatic symptoms of depression are significant predictors of HIV progression and prognosis. Specifically, lower IQ, older age and the presence of somatic symptoms of depression are associated with a more rapid progression from HIV infection to AIDS, HIV-related dementia, and death

Research on HIV infection has examined the role of psychosocial risk factors on disease progression and prognosis. With regard to progression of the disorder, studies of infected adults have most consistently found: A. somatic symptoms of depression and younger age are related to a more rapid progression B. lower intellectual functioning and younger age are related to a more rapid progression C. higher intellectual functioning and older age are related to a more rapid progression D. somatic symptoms of depression and older age are related to a more rapid progression

A. A predictor test has differential validity when it is a valid predictor of outcome for one subgroup of examinees, but invalid or significantly less valid for another group. For example, if a test designed to predict college success predicted the success of whites fairly accurately, but provided little or no predictive information for African-American examinees, the test would have differential validity. Validity generalization studies have suggested that differential validity rarely occurs. Differential validity is not the same as differential prediction, which is a greater concern these days than differential validity. In technical terms, differential prediction means that the regression line (either is slope or its y-intercept) is different for different subgroups. In less technical terms, it means that given scores on a predictor test are associated with different outcomes for different subgroups. In differential prediction, a test can be equally valid for two subgroups, but the actual outcome or criterion scores are different for a given result on the predictor. Research on differential prediction also suggests that it is rare, though not as rare as differential validity, and some research shows that when it does occur, it tends to favor minority groups over majority groups.

Research on differential validity of predictor tests has suggested that it: A. is rare. B. when present, tends to favor minority groups over majority groups. C. is a common cause of test bias and unfairness that negatively impact minority groups. D. only affects the construct validity of tests.

A. Most of the research on group heterogeneity has focused on its impact on creativity and problem-solving and has found it to have beneficial effects on both.

Research on group performance has examined the effects of the heterogeneity of group members in terms of personality, gender, experience, and so on. Overall, these studies have found that heterogeneity is: A. positively related to creativity and decision-making effectiveness. B. positively related to creativity but negatively related to decision-making effectiveness. C. negatively related to creativity but positively related to decision-making effectiveness. D. negatively related to creativity and decision-making effectiveness.

A. ...smokers undergoing behavioral treatments, as compared to control subjects, have about double the quit rate at a six-month follow-up. Examples of behavioral interventions include reinforcement for abstinence and withholding of reinforcement for non-abstinence; stimulus control, whereby stimuli or situations that cue smoking are identified and avoided; and aversive therapy, such as rapid smoking, whereby the person smokes to the point of discomfort or nausea

Research on psychosocial treatments for smoking cessation have provided the most support for the use of which of the following types of interventions? A. behavioral. B. group therapy. C. hypnosis. D. acupuncture.

A. Research on cigarette smoking has found that most smokers stop on their own or with minimal professional help (see M. C. Fiore et al., Methods used to quit smoking in the United States, Journal of the American Medical Association, 1990, 263, 2760-2765). The factors most important for successful cessation are a strong desire to quit, awareness of the negative health consequences of smoking, and social support for quitting

Research on smokers who are trying to quit has found that: A. most people quit on their own. B. cognitive restructuring is the most effective treatment. C. relaxation training is the most effective treatment. D. flooding is the most effective treatment.

B. Research findings show having a biological parent with depression increases an offspring's risk for major depression. What may be surprising however is that the risk is similar whether one or both parents experience major depression. Studies also indicate maternal and paternal depression affect biological offspring similarly in terms of rates of major depression.

Research on the genetic contribution to Major Depressive Disorder indicates that the risk for depression for biological offspring is: A. higher if they have one parent with depression than if they have two parents with depression. B. essentially the same whether they have one or two parents with depression. C. higher if they have a mother with depression than if they have a father with depression. D. substantially higher if they have two parents with depression than if they have only one parent with depression.

A. Diffusion of responsibility refers to the notion that the presence of others reduces people's feeling of personal responsibility for events, as individuals tend to shift responsibility to others present. Both laboratory and field studies have shown that the presence of others does in fact reduce individual helping, charitable giving, task effort, and other types of non-mandatory giving behaviors, including restaurant tipping. The results of such studies are often explained as examples of diffusion of responsibility.

Research on the relationship between group size and the size of tips in restaurants has lent credence to the construct of diffusion of responsibility in its findings that as group size increases, the average tip per customer: A. decreases. B. increases. C. decreases only when diners are charged separately. D. increases only when diners are charged separately.

D. The self-serving bias refers to the tendency to attribute one's own successes to internal factors and one's failures to external factors--i.e., to take credit for one's successes and place blame for one's failures. Research shows that the self-serving bias does not operate the same way across cultures, and in fact may be reversed in more collectivist cultures. For example, a study of Japanese students found that on a difficult memory task, they were more likely to attribute their failures to internal factors and their successes to external factors. Some authors therefore have concluded that a modesty bias rather than a self-serving bias occurs in some cultures. The other choices are true of the self-serving bias. Self-esteem mediates its likelihood of operating; those with high self-esteem are more likely to display it than those with low self-esteem. And as stated by the second choice, it has been shown equally likely to operate under conditions of anonymity and in the presence of an audience (however, in collectivist cultures, conditions of anonymity may evoke it when a modesty bias might otherwise be more likely). Finally, expectation of future improvement is another mediating variable--attribution of failures to internal factors is more likely when a person expects future successes

Research on the self-serving bias has found support for all of the following conclusions except: A. it is more likely to occur in individuals with high self-esteem than low self-esteem. B. it may be present both under conditions of anonymity and in the presence of an audience. C. failure is more likely to be attributed to external factors when the person expects future improvement. D. it operates in the same manner across cultures.

D. In the context of item response theory, differential item functioning (DIF), or item bias analysis, refers to a difference in the probability of individuals from different subpopulations making a correct or positive response to an item, who are equal on the latent or underlying attribute measured by the test. The SIBTEST or simultaneous item bias test, Mantel-Haenszel, and Lord's chi-square are statistical techniques used to identify DIF. Cluster analysis is a statistical technique used to develop a classification system or taxonomy. This method wouldn't detect item bias or differences.

Researchers are interested in detecting differential item functioning (DIF). Which method would not be used? A. SIBTEST B. Mantel-Haenszel C. Lord's chi-square D. cluster analysis

C. Bem's self-perception theory contradicts the more intuitive belief that we make conclusions about ourselves based solely on internal states. Based on the work of B. F. Skinner, Bem's self-perception theory proposes that when internal cues are weak or difficult to interpret, we rely on observations of our behavior and/or circumstances in which this behavior occurs to interpret our attitudes, emotions, and other internal states.

Self-perception theory proposes that people base their: A. beliefs on their feelings B. behaviors on their attitudes C. attitudes on their behaviors D. attitudes on their feelings

D. Group polarization is the tendency of individuals who start off with similar views to end up with a more extreme position after group discussion. Negative framing (a.) refers to presenting a problem in negative terms. Social loafing (b.) refers to decreased individual productivity that occurs when the size of the work group increases. Social facilitation (d.) refers to improved performance on tasks due to the presence of other people. ©

Several members of a homeowners association are talking at a party when a few express their frustration about new rules and the association's new supervisor. By the end of the party, all the group members agree that the supervisor is difficult and unqualified. This is an example of: A. negative framing B. social loafing C. social facilitation D. group polarization

B. Sherif's social judgment theory posits people evaluate persuasive messages through three categories of judgment (d.) — latitude of acceptance, latitude of non-commitment, and latitude of rejection (a.). and is most likely to be persuaded when a message is within his/her latitude of acceptance.

Sherif's social judgment theory suggests an individual is most likely to be persuaded when a message is within his/her: A. latitude of rejection B. latitude of acceptance C. level of self-monitoring D. categories of judgment

A. The progression of Alzheimer's Disease is often divided into three stages. In Stage 1, which lasts 2-4 years, short-term memory loss begins. Patients in this stage frequently complain about forgetting where they placed things and they begin having trouble with complex tasks. In Stage 2, which lasts 2-10 years, there is further memory impairment (mostly explicit rather than implicit), resulting in retrograde and anterograde amnesia. During this intermediate stage, patients often exhibit restlessness, flat or labile mood, and fluent aphasia and they have trouble with simple tasks and get lost in familiar places. In Stage 3, which lasts 1-3 years, there is serious impairment in most areas. During Stage 3, they may lose the ability to speak and become unable to recognize family, friends, or even themselves. They lose all capacity to care for themselves and have difficulty walking, are incontinent, and are ultimately bedridden and often die of an opportunistic respiratory infection

Significant retrograde and anterograde amnesia, restlessness, and fluent aphasia are most characteristic of which stage of Neurocognitive Disorder Due to Alzheimer's Disease? A. second B. fourth C. first D. third

D. According to terror management theory (TMT), individuals utilize two things to manage the terror associated with the fear of death: a cultural worldview and self-esteem. Consensual validation (b.), or others' agreement with one's own worldviews and self-concept, is critical for the effectiveness of cultural worldviews and self-esteem. An outside group, person or differing ideas that clash may threaten the validation system (religion, political ideology) that provides a sense of security. Studies have found increased mortality salience (c.), encountering or recently thinking about death, results in people strongly connecting to their worldview elements like religious beliefs or national pride and increased emergence of prejudices against an outside group in addition to creating a longing for structure, order, acceptance of quick, easy answers to problems and foregoing careful consideration of all options

Solomon, Pyszczynski, and Greenberg's terror management theory proposes anxiety or "terror" results from the conscious awareness that human beings have of the inevitability of their death and is best dealt with by: A. religious faith B. consensual validation C. mortality salience D. self-esteem and a cultural worldview

B. Euthanasia is generally classified as "active" or "passive," and as either "voluntary" or "involuntary." The difference between "active" and "passive" euthanasia is that in passive euthanasia, something is not done that would have preserved the patient's life whereas in active euthanasia, something is done to end the patient's life. The classic example of passive euthanasia is a "do not resuscitate order." Another common method of euthanasia, which is also routinely considered passive euthanasia, is withholding food and fluids. Active euthanasia (a.) involves taking specific steps to cause the patient's death, such as injecting the patient with an overdose of pain-killers or sleeping pills. Voluntary euthanasia (c.) is when the patient requests that action be taken to end his life, or that life-saving treatment be stopped, with full knowledge that this will lead to his death. Involuntary euthanasia (d.) is when a patient's life is ended without the patient's knowledge and consent. In practice, this usually means that the patient is unconscious, unable to communicate, or is too sick and weak to be aware of what is happening or to take any action on his own behalf. Euthanasia differs from "assisted suicide." In assisted suicide, a patient is provided with the means to end his own life, but the provider does not administer it. For example, if a doctor gives an injection of morphine sufficient to cause a patient's death, this is euthanasia. However, if the doctor puts the hypodermic needle beside the patient's bed, explains what it is, and leaves, then later the patient injects himself, this is considered assisted suicide

Stephanie was in a major auto accident and, as a result, is showing little brain activity and cannot live without the assistance of a respirator. After a year without any signs of improvement, Stephanie's parents decide to have the respirator turned off. Stephanie dies less than 2 days later. This is an example of: A. active euthanasia B. passive euthanasia C. voluntary euthanasia D. involuntary euthanasia

D. Higher-order conditioning is a classical conditioning procedure in which two stimuli (CS and US) are paired until the conditioned stimulus (CS) produces the conditioned response (CR) and then the CS (which is now referred to as a US) is paired with another CS to elicit the same response. All of the other choices in this question are operant conditioning terms. Operant conditioning involves behaviors and the consequences that follow, rather than the pairing of stimuli. Chaining (A) is the operant procedure that enables complex behaviors to develop through reinforcement of a sequence of simple behaviors. That is, Behavior A is followed by a reinforcer, which serves as a discriminative stimulus for Behavior B, which is followed by a reinforcer, and so on. Shaping (B) involves reinforcing successive approximations of a desired behavior.

Stimulus A is paired with stimulus B. Stimulus B is then paired with stimulus C until stimulus C elicits the same response that was elicited by stimulus A. This is an example of: A. chaining B. shaping C. secondary reinforcement D. higher-order conditioning

A. The Stroop task is a method for distinguishing between automatic and controlled processes. The Stroop Color-Word Test is used to screen for neurological damage and relies on verbal identification of a list of color words with each word printed in colored ink not denoted by the word (for example, the word "blue" is printed in green, red or yellow ink). The task is to name the ink color in which each word is printed as quickly as possible with a correct response requiring the inhibition of reading the color name. Individuals with left frontal lesions are often unable to inhibit reading the words and others with attention deficits, including ADHD, also perform more poorly on the Stroop Test compared to those without attention deficits.

Stroop color-naming task has been used to study which of the following cognitive processes or phenomena? A. automatic processing B. encoding specificity C. explicit memory D. heuristics

B. A current emphasis in the I/O literature is on team work, and the increased reliance on teams in organizations is supported by research showing that teams are more productive than individuals. Note that, in some situations, group performance is not as good as individual performance (e.g., in certain types of decision-making). However, this question is asking specifically about research related to goal-setting theory, so this is the best response.

Studies examining the predictions of goal-setting theory for work groups suggests that: A. having employees work as a team with team goals results in lower productivity than having employees work as individuals with individual goals. B. having employees work as a team with team goals results in higher productivity than having employees work as individuals with individual goals. C. having employees work as a team with team goals results in similar levels of productivity as having employees work as individuals with individual goals, as long as the goals have been consciously accepted by the employees. D. having employees work as a team with team goals results in similar levels of productivity as having employees work as individuals with individual goals, as long as the goals have been determined by the employees.

A. Quitting smoking has immediate and long-term benefits throughout the lifespan. For example, quitting smoking at age 65 or older reduces the risk of dying from a smoke-related illness by about 50%.

Studies have found that, when people quit smoking at age 65 or older, this reduces their risk of dying from a smoke-related illness by about _____ percent. A. 50 B. 85 C. 35 D. 20

D. Borderline Personality Disorder is characterized by a pervasive pattern of instability in interpersonal relationships, self-image, and affects, and marked impulsivity. Longitudinal studies (e.g., Paris, 2002) indicate that the severity of symptoms are most severe during early adulthood

Studies indicate that the severity of Borderline Personality Disorder symptoms typically are: A. consistent throughout adulthood B. most severe in older adulthood C. most severe in middle age D. most severe in early adulthood

A. Research by N. R. Crick and K. A. Dodge (A review and reformulation of social information-processing mechanisms in children's social adjustment, Psychological Bulletin, 1994, 115,74-101), for example, found that aggressive children often misinterpret the ambiguous or prosocial behaviors of peers as hostile in intent and respond in aggressive ways. The peers then respond negatively, thereby, creating a self-fulfilling prophecy effect

Studies investigating the social cognition correlates of aggression in children suggest that the cognitive biases of these children tend to create a: A. self-fulfilling prophecy effect. B. contrast effect. C. personal fable. D. rebound effect.

B. Super's (1990) life-span, life-space approach to career development uses the life-career rainbow and the archway model to communicate the personal (e.g., values, needs, and interests) and situational (e.g., community, school, and social policies) determinants that influence when and how people play particular life roles and the degree of importance any life role holds. The rainbow is a longitudinal two-dimensional graphic depicting nine life roles that are played in four theaters as one progresses through five developmental stages. Super (1980) identified the nine primary life roles as: child, student, worker, partner, parent, citizen, homemaker, leisurite, and pensioner; the four theaters as: home, school, community, and workplace; and the five life stages as: growth, exploration, establishment, maintenance, and decline. The conceptual model emphasizes that individuals are involved in several roles simultaneously and that roles affect each other. Super (1990) later created the Archway model (1990) to clarify the influence of biographical-geographical, psychological, and socioeconomic components on career development.

Super's conceptual model of the life rainbow depicts: A. life stages. B. life roles. C. self-concept growth. D. occupational interests.

D. The diagnostic criteria for a DSM-5 diagnosis of Oppositional Defiant Disorder include a pattern of angry/irritable mood, argumentative/defiant behavior, and vindictiveness for at least six months. At least four characteristic symptoms must be present and must be exhibited during interactions with at least one person who is not a sibling.

The DSM-5 diagnosis of Oppositional Defiant Disorder requires a duration of symptoms for at least _____ months. A. three B. nine C. twelve D. six

B. The DSM-5 diagnosis of Schizophrenia requires the presence of at least two characteristic symptoms during a one-month period with at least one symptom being delusions, hallucinations, or disorganized speech plus continuous signs of the disturbance for six months or longer

The DSM-5 diagnosis of Schizophrenia requires the presence of at least two characteristic symptoms during a one-month period, with at least one symptom being: A. delusions, hallucinations, or disorganized behavior. B. delusions, hallucinations, or disorganized speech. C. grossly disorganized behavior or inappropriate affect. D. inappropriate affect, disorganized speech, or a negative symptom.

D. For the diagnosis of Delirium, the DSM-5 requires the presence of impaired attention and awareness and at least one additional disturbance (e.g., memory deficit, language disturbance, disorientation) that are the direct physiological consequence of another medical condition, substance intoxication or withdrawal, and/or exposure to a toxin.

The DSM-5 requires _______________ plus at least one additional disturbance in cognition for a diagnosis of Delirium. A. executive functioning deficits B. impaired memory C. language impairments D. impaired attention and awareness

C. Dawis and Lofquist's (1964) Theory of Work Adjustment defines satisfaction as an individual's satisfaction with a job based on how well it meets the individual employee's needs. Satisfactoriness refers to the degree to which the employee's skills meet the requirements of the job and to which the employee successfully performs the job. According to TWA, both are influences of employment tenure.

The Theory of Work Adjustment (TWA) is a person-environment fit theory of career development that indicates employment tenure is tied to: A. organizational culture B. organizational justice C. satisfaction and satisfactoriness D. satisfaction and ability

B. The cocktail-party phenomenon is a characteristic of selective attention which indicates that even when you are intently focusing on one conversation and unaware of another, the mention of your name will immediately get your attention. Change blindness (a.) refers to difficulty perceiving major changes to unattended-to parts of a visual image when the changes are introduced during brief interruptions in the presentation of the image (See: Rensink, R.A. (2005). Change Blindness. In L. Itti, G. Rees, and J.K. Tsotsos (eds). Neurobiology of Attention. (pp. 76-81). San Diego, CA: Elsevier.) Exogenous attention (c.) refers to the automatic attraction of attention, due to, among others, the sudden appearance of a stimulus. This is a typical bottom-up process, controlled by external stimulus presentation, and not under subjects' control. Endogenous attention (d.) refers to a typical top-down, attentional effort under control of the individual, for example, when attention is being focused on the basis of instructions.

The ability to unconsciously monitor the contents of one conversation while consciously focusing on another is referred to as: A. change blindness B. the cocktail-party phenomenon C. exogenous attention D. endogenous attention

D. Two models of decision making are the rational-economic (or classical) and the administrative (or behavioral) model. The rational-economic model deals with how decisions should be made. It assumes the decision-maker is completely rational, has access to and seeks all information relevant to the decision, considers all possible alternatives, and makes the best possible choice. The administrative model deals with how decisions are actually made. It assumes that, due to limited cognitive and external resources, decision-makers simplify problems, work with a limited range of options and alternatives, and employ "satisficing", or seeking the solution that meets minimally acceptable criteria, as opposed to an optimal solution

The administrative model of decision making, as compared to the rational-economic model emphasizes: A. obtaining all available information before making a decision. B. ensuring that the decision-making process is not biased. C. ensuring that decision-making process is not overly simplified. D. a satisficing approach.

B. The psychology licensing exam is considered a measure of knowledge of various areas in the field of psychology and, therefore, is essentially an achievement-type test. Measures of content knowledge should have adequate content validity.

The appropriate kind of validity for a test depends on the test's purpose. For example, for the psychology licensing exam: A. construct validity is most important because it measures the hypothetical trait of "competence." B. content validity is most important because it measures knowledge of various content domains in the field of psychology. C. criterion-related validity is most important because it predicts which psychologists will and will not do well as professionals. D. no evidence of validity is required.

A. According to Tajfel (1982), people strive to maintain and enhance their self-esteem, and this is associated with two components: personal and social identity. Social identity theory states that social identity, the aspect of self-esteem based on group membership, is enhanced by believing one's own group (the ingroup) is attractive and belittling the members of the other groups (the outgroups). Social comparison theory (b.) proposes people self-evaluate by comparing themselves with similar others when objective information is not available. Self-perception theory (c.) proposes when clear internal cues are absent, individuals infer feelings and beliefs by observed behavior or external cues. Self-verification theory (d.) proposes individuals need and seek confirmation of their self-concept, whether the confirmation is positive or negative

The assumption that prejudice and discrimination are outgrowths of the drive to enhance one's own self-esteem is most consistent with the perspective of which theory? A. social identity theory B. social comparison C. self-perception theory D. self-verification theory

C. The Rescorla-Wagner model is built on the idea that learning depends on the surprisingness of the US. When the occurrence of the US is surprising - a larger amount of conditioning occurs and if the occurrence of the US is less surprising - a relatively smaller amount of conditioning occurs

The basic principle behind the Rescorla-Wagner model of learning is: A. the less unexpected or surprising the US, the more conditioning will occur. B. the more unexpected or surprising the US, the less conditioning will occur. C. surprise determines not only if conditioning occurs but how much conditioning occurs. D. surprise determines if conditioning occurs but not how much conditioning occurs.

C. In flooding, the patient is exposed to a feared stimulus. The technique is based on the principle of classical extinction, which involves repeatedly presenting a conditioned stimulus without the unconditioned stimulus. The idea is that the fear developed through classical conditioning, or a pairing of a conditioned stimulus (the feared stimulus) and an unconditioned stimulus (a stimulus that naturally causes fear). For instance, a fear of dogs might have been developed through a pairing of a dog (the conditioned stimulus) and a frightening event associated with a dog, such as a dog bite (the unconditioned stimulus). Classical extinction involves "unpairing" the conditioned and the unconditioned stimulus -- for instance, repeatedly exposing the person to dogs that don't bite.

The behavioral technique known as flooding is based on the theoretical principle of: A. reciprocal inhibition. B. covert desensitization. C. classical extinction. D. negative reinforcement.

B. McGregor's (1960) Theory X and Theory Y management theories reflect the key differences between the scientific management and human relations administrative models. The scientific management model assumes workers are primarily motivated by financial self-interest or pay; job demands must match workers' skills; and workers need constant supervision and detailed guidelines as they are incapable of self-regulating or assuming personal responsibility. Theory X is similar to scientific management in that Theory X managers believe that workers dislike work and avoid it whenever possible, thus workers must be directed and controlled. Theory X advises managers to be very strict, closely monitor employee performance, diligently enforce policies and procedures, and carefully match employees to jobs. A military drill sergeant would be an example of this approach to management. In contrast, Theory Y's (c.) management approach is based on the assumptions that workers are not naturally passive or lazy, they enjoy autonomy, and are willing to accept responsibility. Theory Y advises managers to smile, to treat workers with respect, to bend some of the rules to get the job done, and to be easy-going in their management styles. While Theory X and Y talk about workers from the perspective of how to manage them, Theory Z (d.) examines how workers contribute and proposes that workers naturally wish to cooperate and are loyal to the organization. McClelland's three needs, or Learned, theory (a.) proposes individuals learn needs from their culture and when one of these needs is strong in an individual, then it has the potential to motivate behavior that leads to its satisfaction. The theory's three primary needs are: the need for affiliation (n Aff), which is a desire to establish social relationships with others; the need for power (n Pow), or a desire to control one's environment and influence others; and the need for achievement (n Ach) or a desire to take responsibility, set challenging goals, and obtain performance feedback.

The belief that workers have just enough self motivation to show up at work, punch the time clock, and do only the minimum necessary to get the job done best exemplifies which theory? A. Three-needs theory B. Theory X C. Theory Y D. Theory Z

B. Secondary prevention involves early detection and intervention for a problem in order to reduce its duration and keep it from getting worse. Crisis intervention, suicide hotlines, and screening tests are all examples of secondary prevention

The best example of a secondary prevention program is: A. a rehabilitation program. B. crisis intervention. C. a community education program. D. Head Start.

B. Research on speed and accuracy in learning complex motor skills suggests the best approach is to emphasize speed of performance initially although, to a certain degree, the optimal approach depends on the specific skill.

The best initial strategy for teaching complex motor skills that require speed and accuracy to be successfully performed is to: A. emphasize accuracy over speed B. emphasize speed over accuracy C. emphasize accuracy and speed equally D. emphasize an alternation between speed and accuracy

C. Work samples are good predictors of job success, assuming of course that there is a way to devise an accurate work sample. While interviews and reference letters are the most widely used selection devices, they are not valid predictors by themselves. The popularity of personality tests (particularly personality inventories) has declined, partly due to the non-standardized way they are employed in private industry. They are also not good predictors of job success

The best predictor of job success among the following are: A. reference letters. B. in-depth interviews. C. work samples. D. personality tests.

D. Treatment for very shy individuals who have trouble in interpersonal settings (as implied by the fact that the person cannot hold a job) typically involves social skills and assertiveness training. Although both of these treatments can be administered in individual therapy, the group format has a number of advantages: there are multiple models, and opportunities for feedback, support, and vicarious learning. These aren't available in individual training sessions. Thus, D is the best answer.

The best treatment for someone who has difficulty making friends, is shy, and can't maintain steady employment is: A. individual social skills training. B. paradoxical intention. C. token economy. D. group social skills training.

B. The question describes social loafing. The theory of social facilitation suggests the mere presence of others facilitates (enhances) performance. More specifically, if the dominant response is the right one, then the presence of others will enhance performance and when the dominant response is incorrect, the presence of others results in social inhibition.

The concept that individuals will work harder when alone than when in a group is called: A. group think B. social loafing C. social facilitation D. by-stander effect

C. Client-centered case consultation focuses on a specific client and the consultant advises the consultee in regard to the best course of action in working with the client. The consultant gathers information about the client and then makes recommendations to the consultee regarding a treatment plan or course of action. The consultant may directly assess the client but does not administer treatment (response "D"). Responses "A" and "B"describe consultee-centered consultation, which focuses on the skills, knowledge, and ability of the consultee

The consultant's role in client-centered case consultation is to: A. educate the consultee in methods for evaluating treatment outcomes for a particular type of diagnosis or client B. advise the consultee on the assessment and treatment of a particular type of diagnosis or client C. assess one of the consultee's clients and advise the consultee of the best treatment approach for the specific client D. assess and treat a specific client of the consultee

D. Heart disease is the leading cause of death in the U.S. (31%) when all age groups, races, and sexes are combined. Cancer is the second leading cause of death (23%), followed by stroke (7%). Suicide is the 8th leading cause (1.3%).

The leading cause of death in the United States for all age groups combined is: A. cancer B. stroke C. suicide D. heart disease

C. Acceptance of goals is paramount importance goal-setting theory. Acceptance is the critical factor: A person must consciously accept the goals and intend to achieve them. Participation in goals (d.) may increase one's acceptance and intentions, but it is not considered required. Self-efficacy beliefs (b.) and consistency with one's self-concept are not goal-setting theory principles. © Academic Review

The fundamental principle underlying goal-setting theory is: A. self-efficacy beliefs are related to an individual's commitment to organizational goals. B. organizational goals are only achieved when they are consistent with a person's self-concept. C. behavior is regulated by a person's conscious goals and intentions. D. participation is necessary to ensure commitment to goals.

D. Client-centered case consultation involves working with the consultee (here, the therapists) to develop a plan to work more effectively with a particular client or clients (here, the clinic's patients). By contrast, in consultee-centered case consultation, the focus is on problems in the consultee (e.g., psychological problems, lack of skill) rather than on problems in the clients. And in both client-centered and consultee-centered administrative consultation, the focus is on program-wide administrative problems, rather than on problems in individual clients or therapists

The head of a psychological clinic hires a consultant to help therapists deal with some particularly difficult cases at the agency. This is an example of: A. consultee-centered administrative consultation. B. client-centered administrative consultation. C. consultee-centered case consultation. D. client-centered case consultation.

C. In the 1930s, Edward Tolman demonstrated the concept of latent learning in maze studies with rats. He showed that rats previously exposed to a maze, but with no reward for solving it, solved the maze much more quickly in later trials with reward, as compared to rats without previous exposure. The idea of latent learning, contrary to previous models, was that learning can occur without an explicit reward and such latent learning will manifest itself through behavior at a later time when a reward for it is available. Tolman proposed that the model applies to humans--for example, when we drive or walk the same route daily and learn the locations of various buildings and objects, but only display that knowledge when we need to find a particular object

The idea that a behavior can be learned at one point in time but not displayed until reinforcement is available is consistent with which of the following models of learning? A. Kohler's "a-ha" model B. Thorndike's law of effect C. Tolman's Latent Learning D. Pavlov's Classical Conditioning

D. Organizational transformation is a term for an intervention that is one of the strategies in organizational development. Organizational development (OD) refers to a planned, organization-wide effort designed to improve the organization's effectiveness and targets the organization as a whole rather than an individual unit such as an employee or job. Organizational transformation is a relatively newer method of OD that was developed in response to challenges such as layoffs, mergers, and rapidly changing markets in the corporate world. In organizational transformation, the target of change is the entire organization's vision of its beliefs, purpose, or mission. It attempts to alter the viewpoints of the organization's members. For example, it may involve mission statement changes, consciousness raising, and training exercises designed to help employees shift to an entirely new way of doing business, such as a change to a pay-for-performance pay scale or a new emphasis on vertical rather than horizontal markets

The intervention known as organizational transformation typically focuses on: A. identifying and implementing specific target areas for change. B. increasing cohesiveness of the organization's members. C. group processes and dynamics within the organization. D. the entire organization's vision.

A. The California Supreme Court's Tarasoff decision established the duty of a psychotherapist to protect the intended victim whenever a patient poses a serious danger of violence to another. In ruling that the need to protect the intended victim supercedes a client's confidentiality rights, the Court wrote that "the protective privilege ends when the public peril begins

The legal basis of the Tarasoff decision was that: A. the protective privilege ends when the public peril begins. B. psychologists have a legal and ethical responsibility to uphold the general welfare. C. a psychotherapist's duty to enforce the law supercedes his or her duty to protect a patient's confidentiality. D. a psychotherapy patient's right to confidentiality is absolute.

C. The basis of the harm reduction approach is the pragmatic recognition that active substance users must be met "where they are" in terms of their needs and personal goals in treatment. Consistent with a strengths perspective and inspired by cognitive-behavioral therapy, personality theory, traditional addiction treatment, motivational interviewing, relapse prevention, and public health, the emphasis is on reducing the harm of substance abuse and progress over pathology. An alternative to the disease approach, harm reduction approaches embrace the full range of harm-reducing goals including, but not limited to, abstinence and small incremental positive changes are viewed as steps in the right direction. Marlatt (1998) called the philosophy of harm reduction "compassionate pragmatism".

The main principle behind the harm reduction approach is: A. prevention is more cost effective than treatment. B. inpatient treatment for substance addicted individuals reduces dangerousness. C. helping individuals not ready to give up substances lead safer lives. D. working towards zero use of substances.

B. A number of similar characteristics has been identified by research on child sex abusers and studies indicate that the majority of child sex abusers and other sex offenders minimize, rationalize, or justify their abusive behaviors. One such study presented at the 15th Annual Symposium of the American College of Forensic Psychology in 1999, found 57% of subjects admitted engaging in sexual behaviors with a child but minimized the behavior in some way.

The majority of child sex abusers when asked about their abusive behaviors: A. deny the abuse but eventually admit their actions and express remorse after psychotherapy B. admit their behavior but minimize the harmful effects on or blame the child C. admit their actions, express shame and embarrassment, and say they will never do it again D. claim they were unable to control their impulses

D. The term automaticity refers to the ability to chunk or to move information between working memory and long-term so rapidly and efficiently that the processes entails virtually no attention on the part of the individual. Automaticity is developed through overlearning which minimizes cognitive load in working memory (a.) and allows for higher order processing of information. Some of the other advantages of automaticity include: long-term retention of associated skills, robust under stress and low effort performance. Some limitations associated with automaticity include: the requirement of extended training or a long time to acquire; little memory modification or new learning; individual components of an automatized task or skill become relatively inaccessible to consciousness and therefore are difficult to control; the separate components are difficult to analyze or explain to others; and automatic behaviors are difficult to suppress or modify

The memory phenomenon of automaticity is associated with all of the following except: A. reduced the demand on the working memory B. no new learning or little memory modification C. robust and long-term retention of associated skills D. new skills can be acquired quickly

A. To answer this question, you need to understand what retroactive and proactive interference are. Retroactive interference occurs when your ability to recall X is difficult because of interference by something you learned after X. The longer the period of time between learning X and being tested on it, the greater the opportunity for retroactive interference. Proactive interference occurs when the ability to recall X is impaired by previously learned material. Proactive interference can occur regardless of how long the interval is between learning X and recalling it. Finally, retroactive and proactive interference are most likely to be a problem for information that is not inherently meaningful, which would be the case for a set of unrelated words.

The misinformation effect (hindsight bias) may be a form of: A. retroactive interference. B. motivated forgetting. C. repression. D. proactive interference.

C. According to Loftus' Discrepancy Detection principle, susceptibility to misinformation is inversely related to the ability to notice discrepancies. Therefore, if an individual is aware that post-event information may not be correct then the probability of the misinformation effect is reduced. Warning individuals before they receive post-event information that it might be inaccurate or misleading, increases vigilance and the likelihood that discrepancies between actual and suggested events will be spotted. Resistance to post-event suggestion is greatest when an individual has a strong, accurate original memory. Consistent information also improves memory performance. Research indicates greater susceptibility to misinformation is associated with: the passage of time, it lowers discrepancy detection ability (d.) and the impairing effect of misleading information gets stronger over time (a.); longer retention times, which decrease memory performance; timing of reporting/testing, individuals misled immediately before being tested tend to perform worse than those misled immediately after witnessing the event (b.). Age is also associated with varying susceptibility to misinformation with young children more susceptible than older children and adults and the elderly more susceptible than are younger adults.

The misinformation effect refers to the impairment in memory for the past that occurs after exposure to misleading information. Research on susceptibility and resistance to this effect has found: A. the impairing effect of misleading information gets weaker over time B. subjects misled just before testing tend to perform better than those who are misled just after witnessing the event C. warnings about potential misinformation may inhibit its impairing effect D. the passage of time appears to increase discrepancy detection ability

A. A. Research suggests that high levels of work-family conflict are related to negative outcomes for the individual (life dissatisfaction, anxiety, poor health), for relationships (increased interpersonal conflict, divorce), and for the organization (absenteeism, tardiness, loss of talented employees). Work-family conflict can be time-based, strain-based, or behavior-based. Time-based conflict, which occurs when role pressures stemming from the two different domains compete for the individual's time, is the most common type of work-family conflict. It is based on the scarcity hypothesis, that the sum of a person's energy is fixed; therefore multiple roles inevitably reduces the time and energy available to meet all role demands, resulting in strain and work-family conflict. Much of the research on work-family conflict has been based on this premise proposed by role theory and role scarcity. Work-family conflict is bi-directional; work can interfere with family and family can interfere with work. These are referred to as work-to-family conflict and family-to-work conflict. Recent research, based on the enhancement theory, suggests quality multiple roles provide additional sources of social support, increased skills, and heightened self-esteem and well being. Evidence of positive spillover, both from work to home and from home to work, continues to be found. Strain-based (b.) conflict occurs when the strain experienced in one role domain interferes with effective performance of role behaviors in the other domain. Behavior-based (c.) conflict is described as conflict stemming from incompatible behaviors demanded by competing roles. Recent research, using a systems perspective, demonstrates that one member of a dyad can impact the other's experience of work-family conflict. The effects of this work and family stress and strain are referred to as "crossover effects."

The most common type of work-family conflict is: A. time-based B. strain-based C. behaviorally-based D. systemically-based

C. Studies have shown that threats of retaliation for aggressive behavior work in some situations but not in others. When the retaliator is of high status or has a great deal of power, threats are more likely to suppress aggression. However threats are more likely to increase aggression if the retaliator has previously provoked the person

The studies have shown that threats of retaliation: A. almost always increase aggression. B. almost always decrease aggression. C. are more likely to deter aggression when the retaliator has high status or power. D. are more likely to deter aggression when the retaliator has previously provoked the person.

A. Research on risk factors is not entirely consistent. However, most studies concur with Gould (1992) who found the most consistent predictors to be a diagnosis of depression, use of drugs and alcohol, and antisocial behavior.

The most consistent predictors of adolescent suicide are: A. Depression, use of drugs and alcohol, antisocial behavior B. Depression, loss of popularity, avoidance of social situations C. Use of drugs and alcohol, body weight, level of social acceptance D. Use of drugs, cigarette smoking, depression

B. According to Locke's 1970 goal-setting theory, goals serve two purposes-they are a basis for motivation and they direct behavior. Goal attainment is maximized when goals are specific and moderately difficult and when frequent feedback about progress toward goal achievement is provided. Goal-setting theory proposes that, when people accept goals, they intend to achieve them and, therefore, are willing to put forth the effort to do so. Response choices "A" and "D" are too vague and choice "C" is excessively difficult.

The most effective approach to encourage students to learn new vocabulary words in a foreign language would be to instruct them to: A. learn as many words as possible. B. take their time and plan to learn 5 words a day. C. apply pressure on themselves to learn 20 words a day. D. take it easy and not worry about specific number of words.

C. Cognitive-behavioral therapy (CBT) has been found to be the most effective treatment for GAD, probably because it combines a number of behavioral and cognitive interventions.

The most effective treatment for Generalized Anxiety Disorder is: A. relaxation and graduated exposure. B. a benzodiazepine. C. cognitive-behavioral therapy. D. biofeedback.

D. Electrolytes are salts that conduct electricity; they are found in the body fluid, tissue, and blood. Examples are chloride, calcium, magnesium, sodium, and potassium. Electrolyte imbalance can be caused by the frequent induced vomiting and overuse of laxatives that occur in Bulimia Nervosa. The most frequently found electrolyte disturbance in Bulimia is hypokalemia, a reduced level of potassium in the blood. Electrolyte imbalances can cause cardiac irregularities and cardiac arrest, potentially leading to death. Dehydration and fatigue are also potential side effects of Bulimia, but their potential consequences are not as serious as those of electrolyte imbalance.

The most serious potential medical side effect of Bulimia Nervosa is: A. malnutrition. B. fatigue. C. dehydration. D. electrolyte imbalance.

B. The ERG theory of motivation proposes three categories of needs: existence, relatedness, and growth needs. Existence needs are material and are met by environmental factors such as (in the workplace) pay and benefits. Relatedness needs are met by relationships with superiors, co-workers, and subordinates. Growth needs are met by opportunities for personal development such as the chance to develop abilities that are important to the person. These three categories exist on a continuum from concrete to abstract; existence needs are the most concrete and relatedness needs are the most abstract. The theory is similar to Maslow's Need Hierarchy Theory in that it proposes a continuum of needs, but differs in a number of ways as well. For instance, in ERG theory, a person can move back and forth on the needs continuum. Specifically, according to ERG theory, difficulties in satisfying the more abstract needs causes frustration regression, or a greater focus on fulfilling needs on the next level below.

The motivation theory that proposes a continuum of needs that is comparable to Maslow's Need Hierarchy Theory but attempts to overcome shortcomings of Maslow's theory is: A. Equity theory. B. ERG theory. C. Expectancy theory. D. Goal-Setting Theory.

B. Of the theories listed in the responses, only one -- equity theory -- predicts that worker motivation is related to the comparisons we make between our own situation and that of others

The notion of social comparison is central to which of the following theories? A. expectancy theory B. equity theory C. path-goal theory D. ERG theory

A. Procedural justice is commonly defined by way of a distinction from distributive justice. Judgments of distributive justice are based on comparisons between one's own outcomes to those of others; fairness is perceived when those outcomes are perceived as equal. By contrast, judgments of procedural justice depend on perceptions of fairness in the processes that lead to outcomes. The question describes a situation in which outcomes are equal but the processes that led to them were different. If the person had been focused on distributive justice, she would have been equally upset about losing money in either circumstance, in the trial or to the thief. However, if she is less upset losing the money in a civil judgment, her judgment of fairness may be related to the process by which the money was lost; for example, she may have perceived the trial as reasonably fair. Therefore, procedural justice is the best answer of the choices listed.

The notion that a person would not mind losing money in a civil judgment following a trial as much as she would mind losing it to a thief is related to the concept of: A. procedural justice. B. distributive justice. C. social exchange theory. D. cognitive dissonance.

C. The term "organizational development" (OD) refers to a planned, organization-wide effort, managed from the top-down, designed to improve organizational health and effectiveness by applying behavioral science knowledge. It is distinct from other industrial-organizational interventions in that the target of intervention is an entire organization (or entire work group within an organization), as opposed to an individual unit such as an employee or job. There are a number of intervention strategies associated with OD, including team building, process consultation, and survey feedback. Team building is designed to improve the problem-solving abilities of goal-oriented work groups. It can involve a variety of exercises, activities, and games. The specific goals of the technique include increasing motivation, increasing team cohesiveness, reducing interpersonal conflict and, clarifying team members' roles. Thus, of the choices listed, only the third identifies a specific goal of team building. You may have thought the first choice was a good answer, and team building exercises do often have a positive effect on morale. However, increasing morale is usually not identified as a specific goal of team building

The organizational developmental intervention called team building is most appropriate for addressing which of the following issues? A. increasing the morale of an entire organization. B. diagnosing group interactional issues that interfere with group effectiveness. C. reducing role ambiguity or role confusion in work groups. D. providing feedback to group members about group norms of which they are unaware.

A. Lack of age-appropriate interest in the environment is one of the earliest signs of Intellectual Disability. [A failure to cuddle and lack of eye contact are more characteristic of Autism Spectrum Disorder, and the persistence of primitive reflexes is more likely in cerebral palsy or other serious motor disorder.]

The parents of a child with Intellectual Disability are most likely to say that, during infancy, one of the earliest signs that something "was wrong" was: A. the child's lack of interest in the environment. B. the child's failure to cuddle. C. the persistence of primitive reflexes. D. the child's lack of eye contact.

C. The term "positive" refers to individuals selected or hired and "negative" to those who are not. "True" refers to a correct decision and "false" refers to an incorrect decision. When a predictor cutoff is lowered, more people are selected overall, or there are more positives. Since the predictor criterion is lower there are more true and false positives. Therefore, the probability that those hired will be successful (true positives) decreases and the probability of unsuccessful hiring (false positives) increases

The predictor cut-off score of a selection test is lowered. How will this impact successful hiring? A. increase the probability of false negatives B. decrease the probability of false positives C. decrease the probability of true positives D. increase the probability of true positives

B. Prior to puberty the rates of Major Depressive Disorder are about equal for boys and girls. However, at puberty, the gender difference begins with the disorder occurring about 1.5 go 3 times more often in females than males

The prevalence rate of Major Depressive Disorder for adult females is about 1.5 to 3 times the rate for adult males. This gender difference: A. is the opposite for children under 10 years of age. B. does not become evident until puberty. C. does not become evident until early adulthood. D. is the same for children under 10 years of age.

C. The specifier "with delayed expression" is applied when all of the diagnostic criteria for PTSD are not evident until at least six months after the traumatic event occurs.

The specifier "with delayed expression" is applied to the DSM-5 diagnosis of Posttraumatic Stress Disorder when the full diagnostic criteria for the disorder are not met until at least _____ months after exposure to the traumatic event. A. three B. nine C. six D. twelve

A. The specific symptoms of the Major and Mild Neurocognitive Disorders depend on etiology, but the DSM-5 identifies criteria that apply to all disorder regardless of cause. For example, the Major Neurocognitive Disorders all involve a significant cognitive decline from a previous level of functioning, while the Mild Neurocognitive Disorders involve a moderate decline. In addition, the Major Neurocognitive Disorders produce cognitive deficits that interfere with independence in everyday activities, while the Mild Neurocognitive produce cognitive deficits that do not interfere with independence in everyday activities but may require greater effort, compensatory strategies, or accommodation.

The primary difference between the DSM-5 diagnoses of Major Neurocognitive Disorder and Mild Neurocognitive Disorder is: A. the effects of cognitive deficits on the person's independence in everyday activities. B. the effects of cognitive deficits on the person's adaptive functioning in conceptual, social, and practical domains. C. the degree to which symptoms are reversible or likely to remit. D. the degree to which symptoms have an acute versus an insidious onset.

A. Organization development (OD) is a process used to facilitate organizational change. The phases of OD have been described by Burke as: entry, contracting, diagnosis, feedback, planning change, intervention, and evaluation. The purpose of the feedback phase is to help clients understand the information that the consultant has gathered and diagnosed so the clients can decide what actions to take

The primary purpose of feedback in the context of organizational development is to: A. help clients understand the diagnostic information that has been collected B. provide clients with information on the effectiveness of an OD intervention C. provide employees with information about their individual performance D. provide managers with information on the employee's concerns

B. Of the drugs listed, mescaline is the only hallucinogen.

The recurrence of hallucinations following cessation of substance use is most likely to result from the use of: A. methamphetamine B. mescaline C. cannabis D. cocaine

A. The studies have consistently shown that the combination of hyperactivity and conduct problems is most associated with antisocial behavior and other serious problems in adulthood

The research has shown that the extent and severity of the long-term problems associated with ADHD are strongly related to whether or not the child also had: A. conduct problems. B. a "difficult" temperament. C. a learning disability. D. symptoms of depression.

C. In a structured interview, a series of job-related questions with predetermined "correct" answers are used consistently with all interviewees for a particular job. An advantage of this technique is the provision of individual item scores and a total score that are derived from prespecified criteria. To predict job performance, the interview score can then be combined with scores on other selection procedures in a multiple regression equation or similar technique. Another advantage of the structured interview is that it reduces the impact of interviewer subjectivity (a.). A utility analysis (b.) considers the procedure's validity coefficient, employee job performance variability (typically in dollar value of output or mean output), and the selection ratio to evaluate the practical value of a selection procedure. Subject matter experts (d.) are one of several methods for determining scores to responses to a structured interview.

The responses of interviewees, when using a structured interview, are evaluated by: A. interviewer subjectivity B. "utility analysis" C. pre-specified criteria D. "subject matter experts" consensus

A. A family history of enuresis has been linked to an increased risk for the disorder in offspring. The risk is highest if both parents have a history of the disorder; but, when only one parent had the disorder, risk is higher when that parent is the father. The DSM-5 notes that the risk for nocturnal Enuresis is about 10.1 times higher for children of enuretic fathers and 3.6 time higher for children of enuretic mothers

The risk for nocturnal Enuresis: A. is greater for children of enuretic fathers than for children of enuretic mothers. B. is greater for children of enuretic mothers than for children of enuretic fathers. C. is increased to the same degree for children of enuretic fathers or enuretic mothers. D. is increased only for children whose mothers and fathers are both enuretic.

D. Remembering that females attempt suicide three times more often than males should have helped you eliminate two of the choices. You should also know that among adolescents, suicidal behavior increases with age (although in recent years the rate among younger adolescents has grown, the attempt rate is still greater with increasing age). Suicide attempts are also related to parental separation and divorce, poor social skills, and poor academic achievement, though, not necessarily low I.Q.

The risk for suicide attempts is greatest for a: A. 13 year-old female whose parents recently separated and who has an above-average I.Q. B. 16 year-old male whose parents recently separated and who has a below-average I.Q. C. 14 year-old male who frequently argues with his parents and who has diabetes D. 16 year-old female whose parents are divorced and whose performance in school is below average

D. An oblique rotation is used when the variables included in the analysis are considered to be correlated. When the variables included in the analysis are believed to be uncorrelated (c.), an orthogonal rotation is used. Response choice "a." describes semi-partial correlation and "b." describes partial correlation.

The rotation of factors can be either orthogonal or oblique in factor analysis. An oblique rotation would be chosen when the: A. effects of one or more variables have been removed from X and Y. B. effects of one or more variables have been removed from X only. C. variables included in the analysis are uncorrelated. D. variables included in the analysis are correlated.

D. The item response curve provides information about an item's difficulty; ability to discriminate between those who are high and low on the characteristic being measured; and the probability of correctly answering the item by guessing. The position of the curve indicates its difficulty (c.) and the steeper the slope of the item response curve, the better its ability to discriminate (d.) between examinees who are high and low on the characteristic being measured. The item response curve does not indicate reliability (a.) or validity (b.).

The slope of the item response curve, with respect to item response theory, indicates an item's: A. reliability B. validity C. difficulty D. discriminability

A. The DSM-5 provides three age-related specifiers for Conduct Disorder. "Childhood-onset type" applies when the individual had at least one symptom of Conduct Disorder before age 10; "adolescent-onset type" applies when the individual had no symptoms of Conduct Disorder before age 10; and "unspecified onset" applies when it is unknown if the first symptom was before or after age 10

The specifier "adolescent-onset type" is applied to the DSM-5 diagnosis of Conduct Disorder when the person had no symptoms of the disorder before _____ years of age. A. 10 B. 12 C. 13 D. 15

C. The home advantage in team sports has generated various explanations; however, it most consistently has been linked to a supportive home audience.

The tendency of sports teams to win more often when they are playing on their home court is referred to as the "home advantage." This phenomenon is best explained by: A. home court familiarity B. expectations of the players C. social support from fans D. demoralization of the visiting team

ultimate attribution error

The tendency to make dispositional attributions to an entire group of people is referred to as: A. self-serving bias B. fundamental attribution error C. ultimate attribution error D. actor-observer effect

psychological reactance

The tendency to react in a way that is opposite of what is requested or desired. Is most likely to occur when the individual feels that his/her personal freedom is being restricted, which would probably be the case in the situation described in the question.

A. In classical conditioning, stimulus generalization is the tendency to respond to a new stimulus as if it's the original conditioned stimulus. In operant conditioning, it's the tendency to respond to a new stimulus as if it's the original discriminative stimulus. In classical conditioning, higher order conditioning (b.) refers to the process by which a neutral stimulus comes to act as a conditioned stimulus by being paired with another stimulus that already evokes a conditioned response; and spontaneous recovery (c.) refers to the reappearance of an extinquished conditioned response. Stimulus discrimination (d.) refers to the tendency not to have a conditioned response to a new stimulus that's similar to the original conditioned stimulus. In operant conditioning, it's the tendency for a response to happen only when a particular stimulus is present

The tendency to respond to a new stimulus as if it's the original conditioned stimulus is referred to as: A. stimulus generalization B. higher-order conditioning C. spontaneous recovery D. stimulus discrimination

C. Organizational commitment is related to an employee's willingness to exert extra effort on behalf of the organization and, like job satisfaction, appears to have the highest correlation with job turnover.

The term organizational commitment refers to the degree of an employee's involvement and identification with the organization. An employee's level of organizational commitment is most predictive of: A. job satisfaction. B. work-family life satisfaction. C. job turnover. D. quality and quantity of work.

D. The concepts of business necessity and job relatedness represent conditions that may allow the use of a selection procedure that has an adverse impact for members of a protected group. According to the Americans with Disabilities Act and the Federal Uniform Guidelines on Employee Selection Procedures, if an employer can show that a selection measure, or other employment procedure, that is having an adverse impact is job related and a business necessity, then the employer may be able to continue using the procedure

The terms "business necessity" and "job relatedness" are associated with: A. utility analysis B. comparable worth C. critical incidents D. adverse impact

A. Gottfredson's (1981; 1996; 2002) theory of circumscription and compromise addresses about how gender and prestige influence and limit career choice. The theory proposes four stages of cognitive development including: orientation to size and power; orientation to sex roles; influence of social class; introspection and perceptiveness and that the expression of occupational aspirations emerges as a process of elimination or is the outcome of the competing processes of circumscription and compromise. Circumscription refers to the progressive elimination of least preferred options or alternatives that occurs as children become increasingly aware of occupational differences in gender or sex-type, prestige, and then field of work. Compromise refers to the expansion of preferences in recognition of and accommodation to external constraints (e.g., level of effort required, accessibility, cost) encountered in implementing preferences. (See: Gottfredson, L. S. (1981). Circumscription and compromise: a developmental theory of occupational aspirations. Journal of Counseling Psychology Monograph, 28(No. 6, November), 545-579.) Gottfredson, L. S. (2002). Gottfredson's theory of circumscription, compromise, and self creation. In D. Brown (Ed.), Career Choice and Development (4th ed., pp. 85-148). San Francisco: Jossey-Bass.) Super's (b.) theory proposed a five stage model of career development wherein people achieve job satisfaction when they are able to express themselves and develop their self-concept through their work roles. Krumboltz's (c.) Social Learning Theory of Career Decision Making (SLTCDM) includes four types of influences on making career decisions: genetic characteristics and special abilities; environmental conditions and events; learning experiences; performance standards and values. Social learning influences can be positive or negative factors. Meichenbaum (d.) is associated with cognitive-behavioral therap

The terms "circumscription" and "compromise" are associated with: A. Gottfredson B. Super C. Krumboltz D. Meichenbaum

Social facilitation

The theory that states the presence of others will increase motivation; therefore, if the behavior is well-learned, increased motivation will lead to increased performance. If the behavior is not well-learned, however, the presence of others will lead to heightened arousal, which will interfere with the person's ability to concentrate on the task at hand.

C. Fiedler's Contingency Model states leaders have a dominant leadership style that is resistant to change and distinguishes between two types of leaders - high LPC leaders (person-oriented, more focused on maintaining good interpersonal relationships) and low LPC leaders (task-oriented, more focused on successful task performance). Changes in the structure of the situation can improve the chances of success as the leader's success is contingent on the situation, task to be completed, leader's style or personality, and the maturity of the group. Fielder proposes task-oriented (low-LPC) leaders are most effective when the leader has either low or high situational control and person-oriented (high-LPC leaders) are most effective when situational control is moderate. According to Fielder, situational control is determined by: leader-member relations, task-structure, and leader position power.

The three dimensions of situational control identified by Fiedler include all of the following except: A. leader-employee relations B. position or legitimate power C. role expectations D. task structure

B. Exposure is currently considered the best intervention for most (if not all) Anxiety Disorders. However, the best way of exposing clients to stimuli that elicit anxiety depends on the particular disorder. For Specific Phobia, brief in vivo exposure with response prevention is effective for many clients, and adding a cognitive component usually does not improve the effects of the intervention substantially

The treament-of-choice for Specific Phobia is ordinarily: A. systematic desensitization. B. in vivo exposure with response prevention. C. cognitive restructuring. D. habit reversal training.

D. Justice Perceptions are employee judgments about whether their work situation is fair and have been found to be related to: job satisfaction, performance, withdrawal and counterproductive behaviors and organizational commitment. Interactional justice focuses on the interpersonal treatment of individuals when procedures are implemented and consists of two components: interpersonal justice, which refers to the degree to which individuals affected by decisions are treated with dignity, respect and politeness by third parties or authorities involved in executing procedures or determining outcomes, and informational justice, which refers to the explanations provided to individuals conveying information about why procedures were used in a certain way or why outcomes were distributed in a particular manner.

The two aspects of interactional justice are: A. environmental and social justice B. interpersonal and environmental justice C. informational and comparative justice D. interpersonal and informational justice

C. In a job selection test, usefulness refers to the degree to which the test helps you hire successful candidates. Job selection tests are useful to the degree that the correlation between test scores and job performance is high, the base rate is moderate, and the selection ratio is low. The base rate refers to the percentage of people in the population who are able to perform the job successfully. To understand why you'd want a moderate base rate, think about the extreme cases: a base rate of 100% or 0%. A 100% base rate means that everyone can do the job successfully; a 0% base rate mean that no one can. In both cases, a job selection test is useless because you don't need it to distinguish between good and bad candidates. A base rate of 50% means that half of the population can do the job successfully and a selection test therefore becomes more useful in distinguishing between acceptable and unacceptable applicants. The selection ratio refers to the proportion of available jobs relative to applicants. For example, if 100 people apply for 10 openings, the selection ratio would be 10%. With a small selection ratio, you are only selecting the very top scorers on the test, and assuming the test has a reasonable correlation with job performance, these top scorers are more likely to succeed on the job. Note that the smaller the selection ratio, the higher the number of false negatives, which are people who fail the test but would have succeeded on the job. But this is more of a problem for candidates than it is for businesses that use the test; the latter will still end up with more successful candidates the lower the selection ratio.

The usefulness of a job selection test is increased to the degree that: A. the base rate is low and the selection ratio is high. B. the base rate is high and the selection ratio is low. C. the base rate is moderate and the selection ratio is low. D. the base rate is moderate and the selection ratio is high.

B. loss of appetite and insomnia The classic vegetative (physical) symptoms of depression include loss of appetite, weight loss, fatigue, insomnia, and loss of libido

The vegetative (physical) symptoms of depression include which of the following? A. indecisiveness and impaired concentration B. loss of appetite and insomnia C. increased appetite and weight gain D. hypersomnia and loss of libido

D. Knowing that Tiedeman and O'Hara based their model on Erikson's psychosocial theory of identity would have helped you select the correct answer. They proposed that a person develops a vocational identity through a process of differentiation (realizing that a career does not fit with one's personality) and integration (identifying with a career)

Tiedeman and O'Hara emphasize which of the following in the process of career development? A. innate drives B. interests C. aptitude D. identity

B. In systematic desensitization, anxiety-arousing stimuli are paired with stimuli that produce an incompatible response (often relaxation). In other words, it was designed to use counterconditioning in order to eliminate an anxiety response. Some research suggests, however, that it is actually just the exposure to anxiety-arousing stimuli, without aversive consequences, that explains the effectiveness of this technique.

There is evidence that the effectiveness of systematic desensitization for reducing anxiety is actually due to repeated exposure to the feared stimulus, which leads to extinction of the anxiety response. However, systematic desensitization was originally developed as an application of: A. negative reinforcement. B. counterconditioning. C. stimulus discrimination. D. avoidance conditioning.

D. Continuous schedules, or reinforcing every response, are associated with quick learning, satiation and extinction. The process of thinning, or switching from a continuous to an intermittent schedule, is used to increase the resistance to extinction once a behavior is established.

Thinning refers to the process of: A. switching from a fixed interval to a fixed ratio schedule B. switching from a variable interval to a variable ratio schedule C. switching from an intermittent to a continuous schedule D. switching from a continuous to an intermittent schedule

C. This question is referring to the notion of inoculation, which was derived from the medical usage of the term where it refers to exposing people to a weak form of a virus so that they are resistant to the virus in the future. In terms of resistance to persuasion, inoculation here refers to exposing a person to a weak argument against one's current position.

To best reduce the likelihood that a persuasive message will change a person's mind, the person should be provided with which of the following before hearing the persuasive message? A. weak support for his initial position B. strong support for his initial position C. a weak argument against his current position. D. a strong argument against the persuasive message

external locus of control

Those with __________ view outside forces, such as luck, powerful external entities, or societal injustices, as in control over what happens to them (Rotter) those with ________ of [responsibility] place credit or blame with others (DW Sue)

A. A mnemonic device is a method that one can use to enhance memory for specific information. Numerous effective mnemonic devices have been identified; this question describes, one, the method of loci that dates back to Ancient Greek times. It involves identifying a place one is familiar with and associating visual images of different locations within that place to items of information that one must remember. At the time of retrieval, one uses the images of the familiar location as cues for recall of the items. The other choices also describe mnemonic devices. The story method involves constructing a story about the items to be remembered. The peg method involves memorizing a list of words (the "pegs") that are easily associated with a number, and then associating information that must be remembered to the pegs. In order to facilitate memory of the pegs, a rhyming method (e.g., "one is a bun, two is a shoe") may be used to generate them. And the link method involves generating images of items that link individual items in a list in some way, and then remembering the links in order recall the whole chain

To remember a list of terms that she must memorize for an exam, a student visualizes all of the objects in her dorm room and then associates each object in the room with one item in the list. The mnemonic device that the student is using is known as the: A. method of loci. B. story method. C. peg method. D. link method.

B. In learning theory, habituation occurs when an individual no longer responds to punishment. More generally, the term is used when a person no longer responds to a stimulus that is repeatedly presented. Habituation is often used to study infant perception

To study perception in infants, you present an infant with a sight or sound until he/she stops looking at or turning toward it. The phenomenon you are using to study infant perception is called A. satiation. B. habituation. C. equilibration. D. adaptation.

C. Transactional leadership, in its simplest form, is leadership by contingent reinforcement. The leaders' rewards, promises and/or threats of disciplinary actions or punishments motivate the followers. In constructive transactions, the leader may participate in discussing what is to be done in exchange for implicit or explicit rewards and the allocation of desired resources, or they give out assignments, negotiate or contract with followers. The actions of the leader are contingent on whether the followers do what they have been "contracted" to do. Leaders that take corrective action with negative feedback or reprimand only after followers' mistakes have been called to their attention are engaging in passive management-by-exception. In corrective transactions (a.) or contingent rewarding (b.), leaders engage in active management-by-exception (d.) which means they monitor followers' performance and correct their mistakes if and when they occur.

Transactional leaders who wait for mistakes to be brought to their attention before taking corrective action are engaging in: A. corrective transactions B. contingent rewarding C. passive management-by-exception D. active management-by-exception

C. Based on the principle that a higher score must correlate with better performance on the job, top-down ranking (a.) occurs when an employer selects candidates in the strict highest/lowest order of their test scores. Decisions based on rank ordering when there is no correlation between a higher score and better performance on the job results in adverse impact. Proposed alternatives to reduce adverse impact are using a cut-off score (b.), reflecting the minimal qualifications for the job rather than an unreasonably stringent requirement, and banding. Banding (c.) is based on the assumption that different scores should be viewed as equivalent unless they are statistically significantly different (determined by the standard error of the difference (SED) between scores) and refers to a range of scores being considered as equivalent for selection purposes. Adverse impact is reduced due to lower-scoring applicants being included within the band and banding provides flexibility to select from candidates based on other factors such as ethnicity, gender, work experience, seniority and other job-related factors. Multiple hurdles (d.) approach requires applicants to score above a specific level or pass several measures such as a weighted application, drug test, interview, and skill tests.

Two applicants score 42 and 45 on an assessment test and the standard error of the difference is 3.5 points. The scores are treated as equivalent allowing the applicant with the score of 42 to be selected on the basis of some other job-related characteristic. This approach is an example of __________________ in selection decision making. A. top-down B. cut-off C. banding D. multiple hurdles-required to score above on specific level on several measures

B. The research suggests that about 15% of children diagnosed with ADHD will continue to meet the full diagnostic criteria for the disorder as adults, and up to 60% will not meet the full diagnostic criteria but will continue to have symptoms that have an adverse impact on their daily lives

Up to ___ percent of children with a diagnosis of ADHD will continue to have symptoms in adulthood that do not meet the full diagnostic criteria for the disorder but have adverse effects on their daily lives. A. 45 B. 60 C. 25 D. 10

A. Utilization review is concerned with conserving health care monies. It does this through having a utilization review committee assess the use of benefits and reduce or eliminate inappropriate or unnecessary use of health care resources. Answer B is a description of the concept of quality assurance and answer C is describing a medical team management approach to individual health care.

Utilization review, an important component of managed health care, refers to the idea that it is useful to: A. review benefits to eliminate or reduce unnecessary health care resources. B. determine the adequacy of health care standards by comparing them to predetermined standards. C. make decisions on patient care by a team of medical experts rather than an individual physician. D. allow a patient to choose from several insurance plans.

C. Expectancy theory is based on the premise that motivation is a cognitive process involving a combination of three variables: valence, instrumentality and expectancy. The theory is also referred to as Valence-Instrumentality-Expectancy Theory or VIE Theory. Expectancy (a.) refers to the belief that effort will lead to successful performance. Instrumentality (b.) refers to the beliefs that successful performance will result in certain outcomes; the value placed on the outcomes (d.) of performance is referred to as valence. The need to attribute cause describes Attribution Theory

We create expectations about future events as we are constantly predicting likely futures. This statement is associated with Expectancy Theory. Which of the following is not? A. The belief that if I complete certain actions then I will achieve the outcome. B. The belief that I am able to complete the necessary actions. C. The need to explain the world, to myself and others, ascribing cause to the events around me. D. The value of the perceived outcome or "What's in it for me?"

D. In the Prisoner's Dilemma game, it was found that people tend to compete straight away. They take a risk. This supports the pessimistic view of human nature: outside of our family and reference group we tend to be suspicious and hostile.

What happens if two individuals are provided a situation where, if they cooperate, they will both receive moderate rewards, but if they compete, only one will receive a large reward while the other will receive nothing? A. compete initially but later cooperate. B. cooperate initially but then compete. C. cooperate from the beginning D. compete from the beginning.

B. The item difficulty index ranges from 0 to 1, and it indicates the number of examinees who answered the item correctly. Items with a moderate difficulty level, typically 0.5, are preferred because it helps to maximize the test's reliability.

What value is preferred for the average item difficulty level in order to maximize the size of a test's reliability coefficient? A. 10.0 B. 0.5 C. 1.0 D. 0.0

C. TQM is an organizational philosophy that focuses on maximizing customer service and satisfaction. An important characteristic of TQM is its involvement of employees in all aspects of decision-making, and failures are often due to management's unwillingness to do this.

When Total Quality Management (TQM) fails, it is often because: A. there are too few managers. B. there is too much attention to customer demands. C. the employees are not given sufficient responsibility. D. the rewards are not distributed fairly.

B. The medication of choice for Bipolar I Disorder is usually lithium. However, when a person is nonresponsive to lithium or cannot tolerate its side effects, valproate, carbamazepine, or other anti-seizure medication is often prescribed

When a patient with Bipolar I Disorder is not responsive to lithium, __________ is often the medication of choice: A. an antidepressant B. an anti-seizure medication C. a beta-blocker D. an antihypertensive medication

A. A. One of Schacter's (1999) Seven Sins of Memory, misattribution is divided into source confusion, cryptomnesia and false recall/false recognition. This question is an example of source confusion, in which is a type of misattribution is in regard to the source of a memory. Cryptomnesia (d.) is a form of misattribution where a memory is mistaken for imagination, because there is no subjective experience of it being a memory. Schacter also proposed the "tip of the tongue" phenomenon, in which a person is able to recall parts of an item, or related information, but is frustratingly unable to recall the whole item. This is thought to be an instance of "blocking" (b.) where multiple similar memories are being recalled and interfere with each other.

When a person misremembers seeing an event personally when it was actually seen on television, this is an example of: A. source confusion B. blocking C. false recall D. cryptomnesia

D. According to the Specialty Guidelines for Forensic Psychologists (1991), "When forensic psychologists conduct an evaluation or engage in the treatment of a party to a legal proceeding, with foreknowledge that their professional services will be used in an adjudicative forum, they incur a special responsibility to provide the best documentation possible under the circumstances."

When a psychologist believes that her patient's records will be used in a legal proceeding, she should: A. rewrite them to meet the higher standards of a forensic setting B. avoid the use of any psychological terminology in favor of legal terminology C. maintain them in the same kind and quality as all of her patients' records D. maintain them in the kind and quality consistent with reasonable scrutiny in an adjudicative forum

A. Although group polarization and the risky shift also refer to poor decision-making, their antecedents have not been as well-defined as those associated with groupthink (i.e., only groupthink has been linked theoretically and empirically to an incomplete consideration of alternatives and consequences, high group cohesiveness, etc.).

When an impulsive group decision that reflects an incomplete consideration of alternatives and consequences is the result of high stress, high group cohesiveness, and a directive leader, this is an example of: A. groupthink. B. group polarization. C. the risky shift. D. the Rosenthal effect.

C. According to "Guidelines for Child Custody Evaluations in Divorce Proceedings," "The psychologist should be impartial regardless of whether he or she is retained by the court or by a party to the proceedings. If either the psychologist or the client cannot accept this neutral role, the psychologist should consider withdrawing from the case. If not permitted to withdraw, in such circumstances, the psychologist acknowledges past roles and other factors that could affect impartiality (American Psychologist, 49(7), 1994, 677-680).

When conducting a custody evaluation, a psychologist should: A. advocate for the party that retained him B. advocate for the party that retained him, unless retained by the court, in which case, he should remain impartial C. remain impartial regardless of whether he is retained by the court or either party in the proceedings D. acknowledge the fallacy of impartiality in an adversarial legal system

B. An item response curve provides one to three pieces of information about a test item - its difficulty (answer C); its ability to discriminate between high and low scorers (answer B); and the probability of answering the item correctly just by guessing (answer A).

When looking at an item characteristic curve (ICC), which of the following provides information about how well the item discriminates between high and low achievers? A. the Y-intercept B. the slope of the curve C. the position of the curve (left versus right) D. the position of the curve (top versus bottom)

A. When using the paired comparison technique, a rater compares each person with every other person. It's specific but time-consuming, as you can see. The order of merit comparisons has the rater rank order the people in terms of some criterion, such as overall job performance. You can see that it wouldn't take as long as paired comparisons, but wouldn't be as precise either

When several individuals are to be compared by a supervisor in a work setting, paired comparisons, as compared to order of merit comparisons, are: A. more precise but more difficult. B. less precise but easier. C. more precise and easier. D. less precise and more difficult.

reactance

When the local public library removes a book from its shelves because of the book's shocking sexual content, sales of the book in the local bookstores rise sharply. This is an example of: A. the rebound phenomenon. B. reactance. C. reaction formation. D. reactivity.

C. Use of a token economy involves administering secondary reinforcers such as a token each time the person engages in a desired behavior, or taking away a reinforcer when a person engages in an undesired behavior. The tokens can then be exchanged for primary reinforcers such as food or desired activities. Token economies are commonly used in institutional settings. A problem with them is that behaviors learned often fail to generalize to the real world, since tokens are not available in the real world every time we do something right

When undertaking token economies with seriously disturbed individuals in mental institutions, one of the major problems with the program's efficacy has to do with: A. generalization of behaviors. B. choice of reinforcers. C. exchange ratio. D. reinforcement value.

A. The rating scale described by the question has good inter-rater reliability, or consistency across raters. However, it may or may not have good validity; that is, it may or may not measure what it purports to measure. The question illustrates that high reliability is a necessary but not a sufficient condition for high validity.

When using a rating scale, several psychologists agree on the same diagnosis for one patient. This is a sign that the scale is: A. reliable. B. valid. C. reliable and valid. D. neither reliable nor valid.

A. Stimulus control for Insomnia consists of a list of instructions for actions that increase the association of the bed and bedroom with sleep. Going to bed only when sleepy and using the bed only for sleeping and sexual activity are two of the instructions. Sleep restriction therapy includes instructing the person to go to bed no earlier than the prescribed bedtime and get up no later than the prescribed rise time. Sleep hygiene education involves providing information about sleep conditions and lifestyle habits (e.g., eating a heavy meal or drinking coffee right before bedtime) that contribute to sleep problems. Cognitive behavior therapy for Insomnia focuses on identifying and replacing maladaptive beliefs that contribute to symptoms with less worrisome and more realistic beliefs.

When using stimulus control as a treatment for Insomnia, the client would be instructed to: A. go to bed only when sleepy and use the bed only for sleeping and sexual activity. B. go to bed no earlier than the prescribed bedtime and get up no later than the prescribed rise time. C. avoid coffee or other stimulants, alcohol, and heavy meals several hours before going to bed. D. replace maladaptive thoughts about sleep and lack of sleep with more realistic and less worrisome thoughts.

B. When using the DSM-5, "other specified disorder" and "unspecified disorder" are used when a person's symptoms do not meet the criteria for a specific disorder. "Other specified disorder" is coded when the clinician wants to indicate why a client's symptoms do not meet the criteria for a specific diagnosis - e.g., other specified Depressive Disorder, recurrent brief depression. "Unspecified disorder" is coded when the clinician does not want to indicate why the client's symptoms do not meet the criteria for a specific diagnosis - e.g., unspecified Depressive Disorder

When using the DSM-5, __________ is coded when a clinician does not want to specify why a client's symptoms do not meet the criteria for a specific disorder. A. provisional B. unspecified disorder C. disorder not otherwise specified D. other specified disorder

C. Intellectual Disability requires deficits in cognitive functioning, deficits in adaptive functioning, and an onset during the early developmental period. Level of adaptive functioning in conceptual, social, and practical domains is used to determine severity of the disorder (mild, moderate, severe, or profound).

When using the DSM-5, level of severity of __________ is based on the person's functioning in three domains - conceptual, social, and practical. A. Major or Mild Neurocognitive Disorder B. Autism Spectrum Disorder C. Intellectual Disability D. Schizophrenia

A. It's often important for forensic psychologists to acknowledge the limits of their conclusions. This is because forensic psychologists are often asked for opinions on matters (e.g., probability of future violence) that cannot be made with absolute certainty

When working in a forensic capacity, psychologists must often be careful to: A. acknowledge the limits of their data or conclusions. B. take the time to study the facts of the case so they can arrive at their own conclusion regarding the guilt or innocence of the defendant. C. be present every day of the trial in case the judge requires the psychologist's opinion at some point. D. present their conclusions very forcefully and with confidence, so that the public's confidence in the field of psychology is reinforced.

referent power

When you look up to a person, are influenced by that person, identify with that person and hold them in high regard, that person has: A. Expert power B. Coercive power C. Referent power D. Reward power

B. Organ defined organizational citizenship behavior as discretionary, voluntary behaviors that are not part of an employee's specified role requirements nor formally rewarded by the organization and in aggregate, contribute to organizational effectiveness by enhancing the "social and psychological context that supports task performance" (Organ 1997: 91). Smith, Organ, and Near (1983) initially identified two distinct OCB dimensions: altruism (b.), defined as behaviors directed toward a specific person such as helping co-workers with work-related tasks and generalized compliance (d.), defined as behaviors representative of what a "good worker" should do, such as arriving on time and not wasting time while at work. Organ (1988) later proposed a five-factor model of OCB which included altruism as previously defined; (2) conscientiousness, previously referred to as generalized compliance by Smith et al. (1983) and representing behaviors that go above and beyond minimal expectations of good workers in areas such as attendance, timeliness, and conservation of resources; (3) sportsmanship, referring to behaviors such as tolerating trivial or minor inconveniences without complaining or initiating a grievance; (4) courtesy, involving anticipatory acts that help someone else prevent a problem such as consulting with others when making decisions that may affect them or providing relevant information in advance; and (5) civic virtue, referring to constructive involvement or participation in the overall organization and may include attending meetings regarding the organization. A number of other OCB dimensions have since been proposed and examined, with estimates ranging from one to seven, however most of the OCB literature relies either on the initial two factors or the five dimensions of Organ's (1988) taxonomy (LePine et al, 2002). Contextual performance (a.) is work behavior that affects organizational performance less directly than the task-related behavior typically construed as "performance" and is often used synonymously with OCB in recent studies

Which aspect is not associated with organizational citizenship behavior? A. contextual performance B. responsibility C. altruism D. generalized compliance

C. Glick and Fiske's (1996, 2001) Ambivalent Sexism Theory (AST) describes two complementary, cross-culturally prevalent ideologies called hostile and benevolent sexism, both of which predict gender inequality.. Both hostile and benevolent attitudes about each gender encompass three domains of female-male relations instantiating power differences and interdependence: patriarchy (or men's structural power), gender differentiation (division of labor between the genders, gender roles, and stereotypes), and heterosexuality. Hostile sexism is defined as an adversarial view of gender relations in which women are viewed negatively, as being competitive and seeking to control men. Benevolent sexism is subjectively positive, rewarding women for conforming to a patriarchal status quo and characterizing them as morally pure, to be protected, and adored (p. 110). Based on the results from administration of the Ambivalent Sexism Inventory to over 15,000 individuals in more than 20 countries, Glick and Fiske state that hostile and benevolent sexism are present around the world and both promote gender inequality (See: Glick, P., & Fiske, S. T. (2001a). Ambivalent sexism. In M. P. Zanna (Ed.), Advances in experimental social psychology (Vol. 33, pp. 115-188). San Diego, CA: Academic Press; Glick, P., Fiske, S. T., Mladinic, A., Saiz, J. L., Abrams, D., Masser, B., et al. (2000). Beyond prejudice as simple antipathy: Hostile and benevolent sexism across cultures. Journal of Personality and Social Psychology, 79, 763-775; and Glick, P., & Fiske, S. T. (1996). The ambivalent sexism inventory: Differentiating hostile and benevolent sexism. Journal of Personality and Social Psychology, 70, 491-512.). Fiske et al's Stereotype Content Model (a.) asserts that the primary definition of prejudice as antipathy or hate is too simplistic. The model suggests that prejudice directed at social groups is subject to the same structural factors that affect female-male relations: relative status of groups and the nature of their interdependence (competitive or cooperative). These two factors would determine the content of stereotypic beliefs, the quality of emotions, and the kind of behaviors in response to out-groups. (See: Fiske, S. T., Cuddy, A. J., Glick, P., & Xu, J. (2002). A model of (often mixed) stereotype content: Competence and warmth respectively follow from perceived status and competition. Journal of Personality and Social Psychology, 82, 878-902.). Social role theory (b.) proposes that men and women behave differently in social situations and act in accordance with their social roles, often segregated along gender lines, due to society's expectations put upon them. In other words, men and women confirm gender stereotypes largely because the different roles that they perform place different social demands upon them.

Which of Glick and Fiske's (1996, 2001) theories start from the premise that the relations between the genders are characterized by the coexistence of power differences and intimate interdependence? A. stereotype content model B. social role theory C. ambivalent sexism theory D. sexual prejudice

C. The Big Five personality traits: Openness, Conscientiousness, Extroversion, Agreeableness, and Neuroticism ("OCEAN") have been matched with several of the personality disorders. Antisocial Personality Disorder was found to be associated with low scores in Agreeableness and Conscientiousness and generally higher scores in Neuroticism and Extroversion. Note that "neuroticism" is sometimes referred to as its opposite: "emotional stability

Which of the "Big Five" personality traits is most associated with Antisocial Personality Disorder? A. low on neuroticism, low on agreeableness, and low on conscientiousness B. low on neuroticism, low on openness, low on extroversion C. high on neuroticism, low on agreeableness, and low on conscientiousness D. high on neuroticism, low on openness, low on agreeableness

D. All of the Big Five traits are relatively stable throughout adulthood. However, several studies have concluded that extraversion is the most stable trait, while neuroticism is the least stable over time. In a meta-analysis of 152 longitudinal studies, researchers obtained the following stability coefficients: Extraversion (.55), followed by Agreeableness (.52), Openness (.51), Conscientiousness (.49), and Neuroticism (.46)

Which of the Big Five personality traits is least stable over time? A. Conscientiousness B. Extraversion C. Agreeableness D. Neuroticism

B. A DSM-5 diagnosis of Caffeine Intoxication requires the development of at least five characteristic symptoms following recent caffeine consumption (e.g., restlessness, nervousness, insomnia, diuresis, gastrointestinal disturbance, and flushed face). Headache, irritability and flu-like symptoms are characteristic of Caffeine Withdrawal. Dizziness, depressed reflexes, tremor, and euphoria are symptoms of Inhalant Intoxication. Autonomic hyperactivity, hand tremor, and psychomotor agitation are symptoms of Sedative, Hypnotic, or Anxiolytic Withdrawal

Which of the following are symptoms of Caffeine Intoxication? A. headache, irritability, and flu-like symptoms B. restlessness, insomnia, diuresis, and flushed face C. dizziness, depressed reflexes, tremor, and euphoria D. autonomic hyperactivity, hand tremor, and psychomotor agitation

B. Operant behavioral interventions rely on consequences to increase or decrease the frequency or strength of a behavior, and reinforcement and punishment are the methods used to deliver those consequences. Reinforcement increases the strength or probability of a response, punishment decreases the strength of a response. Reinforcement and punishment can be positive or negative; positive means a stimulus is applied following a response; negative means a stimulus is removed following a response. Thus, negative punishment involves removal of a stimulus following a response in order to decrease that response. Response cost involves removal of a prespecified reward every time a person performs an undesired behavior. For example, a child's allowance may be reduced or skipped if the child yells at her parents. Thus, of the choices listed, it is the only example of negative punishment.

Which of the following behavioral techniques is the best example of negative punishment? A. differential reinforcement for other behaviors (DRO) B. response cost C. reducing the amount of time a child is grounded due to "good behavior" D. a spanking

C. Choice C best describes the distinction between shaping and chaining. Shaping involves reinforcing successive approximations of a single behavior as the person approaches that behavior. For instance, an autistic individual learning to speak might be reinforced first for moving his mouth, then for uttering nonsense sounds, and then for saying a particular word. The successive approximations being reinforced are all components of one behavior. Chaining, on the other hand, involves linking a group of simple behaviors to form a more complex response chain. For instance, a child learning to put on his shirt might first learn to open the drawer, then put his shirt on over his head, and then button the shirt. Each of these behaviors is a separate behavior, but can be linked as a set to constitute the more complex behavior of putting on the shirt

Which of the following best describes the distinction between shaping and chaining? A. shaping involves reinforcement for successive approximations of a single behavior; chaining involves providing a chain of reinforcement for one behavior B. shaping involves providing successive approximations of a reinforcer for a given behavior over a period of time; chaining involves reinforcing multiple behaviors with the same reinforcers C. shaping involves reinforcing component parts of one simple behavior; chaining involves many simple behaviors that are linked to form a more complex behavior D. there is no distinction between these two terms; they are essentially synonymous

A. Pure speed tests and pure power tests are opposite ends of a continuum. A speed test is one with a strict time limit and easy items that most or all examinees are expected to answer correctly. Speed tests measure examinees' response speed. A power test is one with no or a generous time limit but with items ranging from easy to very difficult (usually ordered from least to most difficult). Power tests measure level of content mastered.

Which of the following descriptive words for tests are most opposite in nature? A. speed and power B. subjective and aptitude C. norm-referenced and standardized D. maximal and ipsative

A. Panic Disorder involves recurrent unexpected panic attacks, which are characterized by an abrupt surge of intense fear accompanied by at least four characteristic symptoms. Symptoms may include derealization or depersonalization, which are types of perceptual disturbances

Which of the following disorders is most associated with unusual perceptual experiences? A. Panic Disorder B. Adjustment Disorder C. Illness Anxiety Disorder D. Persistent Depressive Disorder

C. Of the reported cases of child abuse, 50% are White; 25% are African American; 15% are Hispanic; 2% are Native American; and 1% are Asian/Pacific Islanders. However, the proportions of child abuse victims who are Native American or African American are two times greater than the proportions of these children in the general population

Which of the following ethnic groups has the highest proportion of reported incidents of child abuse? A. Asian B. Hispanic C. Native American D. White

B. This question is more about an awareness of your limitations than it is about a dual relationship. The first step, whether you decide to take the case or not, should be to refer her to her family physician to rule out medical concerns.

You are a small-town psychologist in a rural area and a client comes in and states that she is depressed and is having headaches. She has babysat with your child several times over the last year. She would like you to give her biofeedback sessions. You should first: A. Complete the biofeedback treatment and not ask her to babysit in the future. B. Refer her to her family physician C. Refer her to a psychologist in another town for therapy; it will be OK for you to complete the biofeedback sessions D. Refer her to a biofeedback specialist in another town

D. Donald Kirkpatrick's (1959, 1998) model for evaluating training and learning programs includes four levels: reaction, learning, behavior and result. Each level of evaluation is used to assess the effectiveness of training programs and the effects of the training on the employees. Level 1 (b.) evaluations focus on the individual's response to the training or intervention. Level 2 (a.) measures what has been learned, often using pre/post-tests or end-of-course tests. Level 3 evaluations assess the impact of the intervention on the individual's behavior or performance in the workplace and level 4 (c.) measure the impact of training on the organization's results. (See: Kirkpatrick, D.L. (1998). Evaluating Training Programs: The Four Levels (3rd ed.). San Francisco, California: Berrett-Koehler.) Return on investment (ROI), is a "level 5" evaluation, proposed by Jack Phillips, used to calculate the return on investment of an intervention or training. Phillips' Five Level ROI is an expansion of Kirkpatrick's model and encourages ROI evaluations for each of the first four levels

Which of the following is NOT one of the levels of Kirkpatrick's training evaluation model? A. learning B. reaction C. results D. return on investment

C. In the three-stage model of memory, sensory memory is the first stage. It preserves a large amount of sensory information for a very brief period of time (only a second or two). Information from all the senses can be stored in sensory memory. Visual stimuli are stored as "iconic" memories; auditory stimuli are stored as "echoic" memories; and kinesthetic stimuli are stored as "haptic" memories. Contrary to Choice C, sensory memories are believed to be represented in the form of the original stimuli; that is, they are not stored in an altered form.

Which of the following is NOT true regarding sensory memory? A. It has virtually an unlimited capacity. B. It does not retain information for more than 2 seconds. C. It stores altered forms of the original stimulus. D. It stores iconic, echoic, and haptic memories.

B. Habit reversal training is a multicomponent behavioral treatment that was originally developed as a treatment for tics and nervous habits but has also been found useful for treating stuttering and other problematic repetitive behaviors. It consists of awareness training, competing response training, social support, and generalization training

Which of the following is considered to be an effective treatment for tics, trichotillomania, stuttering, and other problematic repetitive behaviors? A. self-instructional training B. habit reversal training C. covert conditioning D. overcorrection

B. In previous versions of the DSM, Obsessive-Compulsive Disorder was included with the Anxiety Disorders. In the DSM-5, it is included in a separate category (Obsessive-Compulsive and Other Disorders) with other disorders that involve obsessive preoccupation and repetitive behaviors including Body Dysmorphic Disorder, Hoarding Disorder, and Excoriation (Skin-Picking) Disorder

Which of the following is included in the DSM-5 in the same category as Obsessive-Compulsive Disorder? A. Acute Stress Disorder B. Body Dysmorphic Disorder C. Generalized Anxiety Disorder D. Illness Anxiety Disorder

Brown and Nicassio (1987) distinguish between two coping strategies for chronic pain -- passive and active. Passive strategies are associated with worse pain and adjustment among chronic pain patients. Passive strategies give responsibility for pain management to an outside source and allow other areas of one's life to be adversely affected by pain. Examples of passive coping strategies are focusing on where the pain is and how much it hurts, restricting or cancelling social activities, and thoughts such as " There's nothing I can do to lessen this pain," or "I wish my doctor would prescribe me better pain medication." In contrast, active coping strategies entail the patient taking responsibility for pain management including attempts to control the pain or to function in spite of it. Examples include engaging in physical therapy or exercise, staying busy/active, relaxation techniques, clearing distracting thoughts and attention from the pain. Cognitive therapy and cognitive behavior therapy have been shown to be effective in reducing the experience of pain and improving positive behavior expression, appraisal and coping in individuals with chronic pain. Evidence also supports a relationship between positive health outcomes and the use of religious coping to manage pain

Which of the following is more likely to exacerbate chronic pain rather than reduce it? A. passive coping B. religious coping C. active coping D. cognitive therapy

D. According to APA's Publication Manual (2001), authors should avoid perpetuating demeaning attitudes in their writing. It is recommended that people be put before their disabilities and not labeled as their disabilities. Thus, "borderlines" (A) would more accurately and sensitively be referred to as "people diagnosed with borderline personality disorder." Similarly "disabled persons" (B) are better described as "people with (or who have) a disability." Thus, choice D is appropriate. The phrase, "individuals confined to a wheelchair" (C) does put the people first; however, it is overly negative and suggests continued helplessness, which should be avoided. Instead, the manual recommends using emotionally neutral expressions, for example, "individuals who use a wheelchair."

Which of the following is most appropriate for referring to a group of research participants? A. "borderlines" B. "disabled persons" C. "individuals confined to a wheelchair" D. "people who have cerebral palsy"

A. Divergent thinking is a creative elaboration of ideas prompted by a stimulus. It enables one to "diverge" from traditional approaches, or what is commonly referred to as "thinking out of the box." Brainstorming, in which people are encouraged to generate all the ideas they can without worrying about evaluation or censure, encourages divergent thinking. The other choices listed all stimulate convergent thinking, which is based on an analysis of information usually leading to a single solution.

Which of the following is most likely to stimulate divergent thinking? A. brainstorming B. a review of the facts and evidence from a variety of sources C. analysis and integration of remembered information D. comparing and contrasting two different models

B. For the DSM-5 diagnosis of ADHD, several symptoms of inattention or hyperactivity-impulsivity must have been present prior to age 12.

Your new client is 14 years old and, based on your initial impression, you believe that his symptoms meet the criteria for a DSM-5 diagnosis of Attention-Deficit/Hyperactivity Disorder. To confirm this diagnosis, he must have had several symptoms of the disorder prior to _____ years of age. A. 10 B. 12 C. 7 D. 5

D. The objective of risk or harm reduction strategies is to keep individuals as safe as possible until ready to discontinue drug use. Harm reduction, based on principles of public health, offers a pragmatic yet compassionate set of strategies designed to reduce the harmful consequences of addictive behavior for both drug users and the communities in which they live. "Four basic assumptions central to harm reduction are: (1.) it is a public health alternative to the moral/criminal and disease models of drug use and addiction; (2.) it recognizes abstinence as an ideal outcome but accepts alternatives that reduce harm; (3.) it has emerged primarily as a "bottom-up" approach based on addict advocacy, rather than a "top-down" policy established by addiction professionals; and (4.) it promotes low threshold access to services as an alternative to traditional high threshold approaches."

Which of the following is not an example of a risk reduction strategy? A. condom distribution B. methadone maintenance C. the designated driver D. educating about drugs' effects on the body

C. Factor analysis has shown that job commitment has three forms. Affective commitment (a.) refers to the employee's psychological attitudes toward the organization; continuance commitment (b.) refers to the costs of leaving the company and normative commitment (d.) is the employee's perceived obligation to stay with the company. Continuance commitment is least related to job satisfaction and other job-related attitudes and more to practical considerations

Which of the following is not one of the three forms of job commitment? affective commitment continuance commitment instrumental commitment normative commitment

C. Flextime allows for flexibility in scheduling work hours, as long as the required number of hours are worked and the employee is present for the pre-defined "core" hours" (e.g., 10 a.m. to 2 p.m.). Research indicates flextime is associated with improved employee absenteeism, productivity, job satisfaction, satisfaction with work schedule, and less worker stress. The greatest impact was on absenteeism, followed by productivity, satisfaction with schedule, and job satisfaction. Self-rated performance has not been found to be significantly related to flextime.

Which of the following is not significantly associated with flextime? A. productivity B. absenteeism C. self-rated performance D. job satisfaction

D. Work samples provide standardized, job-like conditions to measure work behavior. They are used as a selection technique, as an indicator of individuals likely to benefit from training, and as a means of reducing turnover through a job preview. Research indicates that work samples are generally good predictors of job performance; however, work samples of motor skills have more validity than work samples of verbal skills. Additionally, work samples are less likely to discriminate against individuals from different groups and are acceptable to applicants

Which of the following is not true about work samples as a selection technique? A. less likely to discriminate against members of minority groups B. good predictor of job performance C. work samples of motor skills have more validity than work samples of verbal skills D. work samples of verbal skills have more validity than work samples of motor skills

B. According to Hollander (1985) in order to successfully challenge the majority opinions of a group, a person must first conform to the group in order to establish credibility as a competent insider. By becoming accepted members of the group, we accumulate idiosyncracy credits which are like brownie points

Which of the following is related to minority influence? A. Ambiguity B. Idiosyncracy credits C. Conformity D. Psychological reactance

B In a study conducted by Elizabeth Loftus and her colleagues (who has been critical of the possibility of recovering repressed sexual abuse memories), it was found that 19% of sexual abuse victims had forgotten but later recalled sexual abuse. Although this is a lower rate than reported in other studies (which the authors criticize as methodologically flawed), the authors did conclude that, at least for some women, repression and recovery of these memories are possible. No factors distinguishing these women from victims who don't repress memories of sexual abuse were found.

Which of the following is the best conclusion regarding repressed memories of sexual abuse? A. Based on what we know about the process of memory, the idea that it's possible to forget but later recall childhood sexual abuse is not credible. B. Some people may forget about sexual abuse and subsequently remember it later in life. However, as of yet, no factors that distinguish between these people and other victims of sexual abuse are known. C. Some people, particularly less intelligent victims, may forget about sexual abuse and subsequently remember it later in life. D. Due to the traumatic nature of sexual abuse, memories of it are repressed in the majority of cases.

C. Transformational leaders are change-oriented. They encourage and inspire subordinate acceptance and support for organizational change that is in the best interests of the organization. In contrast, transactional leaders motivate through appealing to the followers' self-interests and focus more on stability than change.

Which of the following is true of transformational leaders? A. the goals of the organization are redefined to reflect the self-interest of followers B. believe that subordinates are persuaded when the organization's goals are compatible with their self-interests C. motivate by encouraging subordinates to transcend self-interest for the greater good of the organization D. assent to the self-interest of their followers

C. Compliance with medical treatment regimens, such as those designed to manage diabetes, tends to be lower for adolescents as compared to children or adults. There are many reasons for this, including adolescents' greater desire to be similar to their peers and independent from the restrictions of their parents.

Which of the following is true regarding compliance with medical treatment regimens? A. Compliance is better among adolescent boys than adolescent girls. B. Compliance is better among adolescents than children. C. Compliance is better among children than adolescents. D. Compliance is about equal for adolescents and children.

B. The diagnosis of Disruptive Mood Dysregulation Disorder requires an onset of symptoms prior to 10 years of age. The characteristics listed in the other answers are consistent with this diagnosis.

Your new client is Billy, age 14. Your initial impression is that his symptoms meet the criteria for a DSM-5 diagnosis of Disruptive Mood Dysregulation Disorder. Which of the following would rule out this diagnosis? A. Billy has temper outbursts three or four times a week. B. Billy's symptoms began when he was 12 years of age. C. Billy has never had a symptom-free period that lasted more than three months since the onset of his symptoms. D. Billy's has been persistently irritable between temper outbursts most of the day, nearly every day.

C. You probably remember that the alternate forms coefficient is considered by many to be the best reliability coefficient to use when practical (if you don't, commit this factoid to memory now). Everything else being equal, it is also likely to have a lower magnitude than the other types of reliability coefficients. The reason for this is similar to the reason why it is considered the best one to use. To obtain an alternate forms coefficient, one must administer two forms of the same test to a group of examinees, and correlate scores on the two forms. The two forms of the test are administered at different times and (because they are different forms) contain different items or content. In other words, there are two sources of error (or factors that could lower the coefficient) for the alternate forms coefficient: the time interval and different content (in technical terms, these sources of error are referred to respectively as "time sampling" and "content sampling"). The alternate forms coefficient is considered the best reliability coefficient by many because, for it to be high, the test must demonstrate consistency across both a time interval and different content.

Which of the following methods of establishing a test's reliability is, all other things being equal, likely to be lowest? A. split-half B. Cronbach's alpha C. alternate forms D. test-retest

C. You may have been able to answer this one through the process of elimination if you knew that the models described by Fiedler, Hersey and Blanchard, and House don't include a decision-tree, which is provided by the Vroom and Yetton model

Which of the following models of leadership provides a "decision tree" to help a leader determine whether an autocratic, consultative, or consensual decision-making approach is best given the nature of the work situation? A. Fiedler's contingency model B. Hersey and Blanchard's situational model C. Vroom and Yetton's normative model D. House's path-goal model

D. In forward conditioning, which is used in classical conditioning to establish a conditioned response, the CS is presented prior to the US (c.). In contrast, backward conditioning entails presenting the US prior to the CS. Changing from a continuous to an intermittent schedule (a.) is referred to as "thinning" and not the procedures associated with backward conditioning.

Which of the following most accurately reflects the process of backward conditioning? A. Establish a response on a continuous schedule then use an intermittent schedule of reinforcement. B. Repeatedly present the CS and the US simultaneously. C. Repeatedly present the CS before the US. D. Repeatedly present the US before the CS.

B. Based on the logic that decision making will be improved by using a decision-support framework that explicitly considers the costs and benefits of human resources (HR) decisions, a utility analysis summarizes and identifies key variables that describe the consequences of HR programs. Utility analysis has most frequently been studied as applied to selection procedures although other cost/benefit analyses have been applied to HR interventions such as recruitment, training, performance feedback, promotion, turnover/layoff management and more recently, new policy implementation. Studies have purported to identify, with a high degree of precision, the financial payback to be realized through investments designed to improve employee productivity. Advances in utility analysis application and related research have been limited due to an inability to develop an agreed-upon method to estimate the dollar value of job performance and its standard deviation (SDy). The challenge arises in uniformly evaluating human inputs, actions, and/or the outcomes of those actions as well as the uncertainty of task characteristics, relationships, and levels of task independence across positions and organizations. Utilization Review (a.) is a cost containment procedure involving an evaluation of patients' use of services to identify any unnecessary or inappropriate use of health care resources. Concurrent review (c.) describes the utilization review conducted during the course of treatment. Quality management (d.) is another name for quality assurance

Which of the following procedures summarizes and identifies key variables that describe the consequences of HR programs to improve decision making? A. utilization review B. utility analysis C. concurrent review D. quality management

Internalized racism

Which of the following refers to an acceptance by members of stigmatized races of negative messages about their own abilities and intrinsic worth? A. personally mediated racism B. internalized racism C. subtle racism D. institutional racism

C. The Premack Principle refers to using a frequently occurring behavior to reinforce an infrequently occurring one. Choice C is an example -- if the child eats his spinach (presumably, something he does not do very often on his or her own), he will be allowed to play (presumably, something he does quite often on his own).

Which of the following statements best exemplifies the Premack Principle? A. "If you're a good boy, you'll get candy." B. "If you don't get home before midnight, you'll be grounded." C. "Eat your spinach before you go out to play." D. "I have no idea what those socks are doing on the chandelier."

A. Polls in different countries at different times have indicated that both males and females are more likely to prefer a male boss to a female boss. For example, one recent study in the U.S. found that 45% and 50% of males and females respectively prefer a male boss, as compared to 19% of men and 26% of women who prefer a female boss; the rest of the respondents expressed no preference. The preference for male bosses over female bosses has been on the decline over the years, but contrary to choice 4, it is not non-existent.

Which of the following statements regarding employee preference for male and females boss' is most supported by survey research? A. A greater proportion of both male and female employees prefer a male boss to a female boss. B. A greater proportion of both male and female employees prefer a female boss to a male boss. C. Males prefer a male boss to a female boss, and females prefer a female boss to a male boss. D. In the past, there was a greater preference of a male bass over a female boss in both genders, but that difference is now non-existent.

D. In most cases, informed consent to research must be obtained from participants. However, there are some exceptions, such as research involving anonymous questionnaires or naturalistic observations. Ethical standard 8.05 discusses the circumstances under which research does not require informed consent.

Which of the following statements regarding informed consent procedures in research is least true? A. Psychologists should inform research participants, in language that is reasonably understandable, about the nature of the research. B. Research participants should be informed of significant factors that may affect their willingness to participate in the research. C. Informed consent to research must be appropriately documented. D. Informed consent to research must be obtained in all cases.

C. Ordinarily, you need to obtain informed consent from a client before releasing information you've obtained from him or her. However, this is not always the case; for instance, when you reasonably suspect child abuse, you are legally obligated to file a report with the appropriate state agency -- with or without the client's consent. As for consulting situations (choice B), informed consent from the client is not necessary as long as you adequately disguise the client's identity.

Which of the following statements regarding the need to obtain informed consent before releasing client information is most true? A. A psychologist should always obtain informed consent from the client before releasing information. B. Before consulting with a colleague about a case, a psychologist must always obtain informed consent from the client. C. A psychologist must obtain informed consent from a client some of the time before releasing information. D. If a psychologist wants to release information about a case, he or she may do so without obtaining informed consent.

C. Multiple regression is the preferred technique for combining test scores in this situation as it is a compensatory technique since a low score on one test can be offset (compensated for) by high scores on other tests. Multiple baseline (a.) is a research design, not a method for combining test scores. Multiple hurdle (b.) and multiple cutoff (d.) are noncompensatory techniques

Which of the following techniques would be most useful for combining test scores when poor performance on one test can be offset by excellent performance on another: A. multiple baseline B. multiple hurdle C. multiple regression D. multiple cutoff

D. Tiedeman and O'Hara based their career development model on Erikson's psychosocial theory of ego identity development. They proposed that a person develops a vocational identity through a process of many differentiations and reintegrations throughout one's lifespan. Differentiation comes from the realization that a particular career does not "fit" with one's personality. Integration involves identification with a career.

Which of the following theorists related career development to ego identity development? A. Anna Freud B. Super C. Holland D. Tiedeman and O'Hara

D. Personality tests have not been shown to have very good validity in predicting performance on almost all types of jobs. Reported validity coefficients between scores on these tests and measures of performance are typically less than .15. Reported validity coefficients for biographical data or job interviews are typically in the moderate range; structured interviews as opposed to the informal interviews that are most commonly used, often have higher validity. Assessment centers, which involve structured, group activities designed to assess how well prospective executives function in simulated situations requiring leadership, have been found to be relatively highly correlated with measures of job performance

Which of the following types of assessment has the lowest validity in terms for predicting performance in an industrial setting? A. assessment center B. biographical data C. personal interview D. personality inventory

D. Management development is a growing field in organizational psychology. A number of techniques are used to train (develop) managers including job rotation, coaching, mentoring, and even Outward Bound, which fosters trust and cooperation. Vestibule training involves the use of procedures or an environment similar to the actual job and is used for such jobs as bank tellers, clerks, and typists

Which of the following would be least useful as a method for management development? A. Outward Bound programs B. job rotation C. coaching or mentoring D. vestibule training

D. Item characteristic curves (ICCs), which are associated with item response theory, are graphs that depict individual test items in terms of the percentage of individuals in different ability groups who answered the item correctly. For example, an ICC for an individual test item might show that 80% of people in the highest ability group, 40% of people in the middle ability group, and 5% of people in the lowest ability group answered the item correctly. Although costly to derive, ICCs provide much information about individual test items, including their difficulty, discriminability, and probability that the item will be guessed correctly

Which of the following would be used to determine the probability that examinees of different ability levels are able to answer a particular test item correctly? A. criterion-related validity coefficient B. item discrimination index C. item difficulty index D. item characteristic curve

C. Internal consistency is one of several types of reliability. As its name implies, it is concerned with the consistency within a test, that is, the correlations among the different test items. Split-half reliability is one of the measures of internal consistency and involves splitting a test in two and correlating the two halves with each other. Other measures of internal (inter-item) consistency are the Kuder-Richardson Formula 20 (for dichotomously scored items) and Cronbach's coefficient alpha (for multiple-scored items). Test-retest reliability ("B") is not concerned with internal consistency, but rather, the stability of a test over time, and uses the correlations of scores between different administrations of the same test. Alternative forms reliability ("D") is concerned with the equivalence of different versions of a test. And the kappa coefficient is used as a measure of inter-rater reliability, that is, the amount of agreement between two raters.

Which of the following would be used to measure the internal consistency of a test? A. kappa coefficient B. test-retest reliability C. split-half reliability D. alternate forms reliability

C. To answer this you need to know two concepts in forgetting: one related to neuropsychological functioning, and the other related to cognitive psychology. If amnesia starts from a time of trauma and extends forward into new experiences (e.g., the person can't remember anything occurring after a head trauma), it's anterograde. If the amnesia extends backwards into experiences before the trauma, it's retrograde. So keep this in mind: anterograde for new experiences, retrograde for past experiences. Now let's move to cognitive psychology and compare memory inhibitory processes. Retroactive inhibition occurs when new learning interferes with previous learning. Proactive inhibition occurs when previous learning interferes with new learning. Now, if we make a comparison between the two sets of terms, we will combine proactive inhibition with anterograde amnesia. This is because, in proactive inhibition, the forgetting is for information occurring after a particular event, and in anterograde amnesia, the person cannot retain any information that occurs after a trauma. Retroactive inhibition and retrograde amnesia are analogous for similar reasons, except that the word "before" rather than "after" applies

Which of these pairs is most similar? posttraumatic amnesia and retroactive inhibition. retrograde amnesia and proactive inhibition. anterograde amnesia and proactive inhibition. anterograde amnesia and retroactive inhibition.

B. This question is difficult because the language of the response choices is convoluted and imprecise. We don't write questions like this because we're sadistic; it's just that you'll sometimes see this type of language on the exam as well, and we want to prepare you. What you need to do on questions like this is bring to mind what you know about the issue being asked about, and to choose the answer that best applies. Here, you should bring to mind what you know about the relationship between reliability and validity: For a test to have high validity, it must be reliable; however, for a test to have high reliability, it does not necessarily have to be valid. With this in mind, you should see that "high validity assumes high reliability" is the best answer. This means that a precondition of high validity is high reliability. The second best choice states that low reliability assumes low validity. This is a true statement if you interpret the word "assume" to mean "implies" or "predicts." But if you interpret the word "assume" to mean "depends on" or "is preconditioned by," the statement is not correct. So B is the best answer

Which statement is most correct? A. High reliability assumes high validity. B. High validity assumes high reliability. C. Low validity assumes low reliability. D. Low reliability assumes low validity.

C. A test's reliability sets an upper limit on its criterion-related validity. Specifically, a test's validity coefficient can never be higher than the square root of its reliability coefficient. In practice, a validity coefficient will never be that high, but, theoretically, that's the upper limit

Which statement is most true about validity? A. Validity is never higher than the reliability coefficient. B. Validity is never higher than the square of the reliability coefficient. C. Validity is never higher than the square root of the reliability coefficient. D. Validity is never higher than 1 minus the reliability coefficient.

B. Schmuck & Schmuck (2000) define group cohesion as "the relation of individual group members to the group as a whole." Whether or not high levels of group cohesiveness manifests in improved productivity depends in part on how the group is managed. Management supportiveness is one of several variables that moderates the relationship between group cohesiveness and group productivity, with high levels of support being associated with higher levels of productivity and management hostility or indifference being associated with lower productivity

While both positive and negative effects are associated with high levels of group cohesiveness, with respect to productivity, high group cohesiveness is: A. associated with higher productivity when management is indifferent to the group. B. associated with higher productivity when management is supportive of the group. C. consistently associated with lower group productivity. D. consistently associated with higher group productivity.

A. The psychologist's ethical obligation is to serve the best interests of the child, but legally, he/she cannot proceed with treatment if the parents do not give consent to do so. While some states have laws that allow psychologists to provide services without parental consent, no information is given in this question regarding the relevant law or whether the child's problem(s) would be covered by this type of law. The Ethics Code Standard 1.02 states that "If psychologists? ethical responsibilities conflict with law, psychologists make known their commitment to the Ethics Code and take steps to resolve the conflict in a responsible manner." Therefore, the psychologist would not just terminate without attempting to advocate for the patient (c.); nor continue to treat the child without the parents consent (d.), as violating the law is not consistent with resolving the conflict in a responsible manner

With parental consent, a psychologist has been providing therapy to a 12-year-old boy for a couple of months. Unexpectedly, both parents suddenly withdraw their consent. The psychologist believes that continuing therapy is in the best interests of the boy. The psychologist's best course of action would be to: A. seek judicial intervention B. obtain a signed consent from the boy and continue treatment C. stop the treatment because both parents have withdrawn their consent but provide them with referrals to other therapists D. continue to see the boy without the parents' consent

C. High group cohesiveness has positive and negative consequences. In terms of productivity, the relationship between group cohesiveness and group productivity is affected by several moderator variables. One of which is management supportiveness. High management support is associated with high levels of productivity for cohesive groups, while management hostility is associated with low productivity for cohesive groups.

With respect to productivity, cohesive groups are associated with: A. low levels of productivity for the group B. high levels of productivity for the group C. higher levels of productivity for the group only when management is supportive D. higher levels of productivity for the group only when management is hostile or indifferent

B. Dawis and Lofquist's (1964) model of career development, Theory of Work Adjustment (TWA), centers on the connection between a worker and his/her job and work environment. A key element of the theory is that an individual is best suited for positions that have matching demands and rewards to his/her individual characteristics. According to the TWA, satisfaction and satisfactoriness are factors related to job tenure. Satisfaction, in this theory, refers to a worker's satisfaction with the job. This is established by the degree to which the worker's needs are satisfied by the provided rewards of the job. Satisfactoriness, on the other hand, refers to the organization or employer's satisfaction with the worker. For example, how well the worker's skills fulfill the requirements of the job and is able to perform the job

Within the context of Theory of Work Adjustment (TWA), job tenure is related to an employee's: A. organizational commitment. B. satisfaction and satisfactoriness. C. career concept. D. motivation and ability.

D. A process consultant, rather than merely correcting problems for his or her client, focuses on an organization's underlying processes, with the goal of making changes in processes so that individuals within the organization can solve their own problems. Thus, choice D is the best answer, as it is the only one which offers a way in which the nurses and administrators can work together to solve the problem of dissatisfaction with work conditions. By comparison, choices A and B would have the consultant diagnose and "fix" the problem; these interventions would be consistent with the medical rather than the process model of consultation.

You are an organizational process consultant hired by a hospital. Several nurses tell you they feel they're being overworked and underpaid. In this case, your most likely intervention would be to: A. explain the nurses' concerns to the hospital administrators. B. conduct a job analysis to determine how much the nurses deserve to be paid and present your results to the hospital administrators. C. act with the best interests of the hospital's patients in mind. D. help the nurses organize regular meetings with the physicians and administrators to discuss their concerns.

B. The situation described in this question is referred to as differential validity, which can be a cause of adverse impact. (Unfairness is another cause of adverse impact but is different from differential validity.)

You are concerned that the validity coefficient of a new selection test might be significantly lower for females than for males. Specifically, your concern is that use of the test might lead to: A. unfairness. B. adverse impact. C. statistical regression. D. criterion contamination.

B. This question refers to a framework that identifies three general strategies to bring about change in an organization: empirical-rational, power-coercive, and normative-reeducative. Empirical-rational strategies are based on the assumption that people are basically rational and, if they have all relevant information about a situation, will act in accord with their self-interest. Thus, a proposed change will be adopted if the proposer can rationally justify the change and show that it will benefit the members of the organization. The consultant in this question is explaining how the proposed changes will advance the self-interest of the employees; thus, he is adopting an empirical-rational approach

You are hired as a consultant by a large corporation. You evaluate the company and come up with a list of interventions that you believe would increase the company's profitability. To encourage employees to accept these changes, you assemble them in a large meeting hall and give a speech explaining how your proposed changes will benefit the company and, by extension, the employees. Which of the following strategies for increasing acceptance to change are you using? A. normative-reeducative B. empirical-rational C. power-coercive D. advocacy consultation

D. This question is about multiple relationships, or relationships where a psychologist sees a client professionally but is also involved in some other kind of relationship with him or her, such as a business or personal relationship. The point of this particular question is that not all multiple relationships are unethical. The ethical standard applicable to multiple relationships, standard 3.05, states that psychologists refrain "from entering into a multiple relationship if [it] ...could reasonably be expected to impair the psychologist's objectivity, competence, or effectiveness...or otherwise risk exploitation or harm." It also states "Multiple relationships that would not reasonably be expected to cause impairment or risk exploitation or harm are not unethical." Though you would need more information to be certain, it is most reasonable to assume that a professional relationship with the second cousin of your child's barber falls into the latter category of multiple relationships that would not likely cause harm or risk. If the question stated that your client was (for example) your husband's boss, your obligations would be different, because in that situation, you could expect that your objectivity would be impaired. In that situation, you would not take the case to begin with if you knew in advance that the client was your husband's boss. If you found this out in the course of therapy, you would have to, according to the same ethical standard, "take reasonable steps to resolve [the relationship] with due regard for the best interests of the affected person and maximal compliance with the Ethics Code."

You are seeing a client whom you discover, in your third session with him, is the second cousin of your child's barber. In this situation, your ethical obligation is to: A. terminate therapy as soon as possible because multiple relationships are prohibited. B. immediately refer the client to another therapist. C. continue seeing the client for a few more sessions and refer the client out when appropriate. D. continue to see the client.

A. This issue is addressed in Standard 5.02(b) of the Ethics Code, which states that "Psychologists do not compensate employees of press, radio, television, or other communication media in return for publicity in a news item."

You are starting a stop-smoking group and tell a newspaper reporter who smokes that he can participate in the group for free if he'll write an article about it for the local newspaper. This arrangement is: A. unethical. B. ethical as long as you don't tell the reporter what to write about. C. ethical as long as you check to make sure that information about the group is accurate. D. ethical as long as the confidentiality of the group participants is not violated.

C. Variables that affect the validity of a test are moderator variables. When a moderator variable is present a test is said to have differential validity--meaning there would be a different validity coefficient for the New Zealanders group than for the others.

You are testing a cross-section of minority clients including New Zealanders, Hispanics, African-Americans and Asians. The New Zealander's group turns out to serve as a moderator variable. This means the test has: A. Cross validation B. Shrinkage C. Differential validity D. Criterion contamination.

D. Overcorrection involves two phases -- restitution and positive practice -- which both require the individual to perform certain corrective behaviors. If the individual doesn't comply, he/she may be physically guided through the desired behaviors

You are using overcorrection to modify a child's misbehavior. If the child refuses to comply with your requests during the procedure, you would most likely use which of the following? A. participant modeling B. withdrawal of attention C. verbal reprimands D. physical guidance

B. The best answer is that you should bill according to your office policies which, presumably, was explained and agreed to by the client at the beginning of treatment (Standard 6.04: Fees and Financial Arrangements). If charging the client for the missed session was not agreed upon in advance, this would be unethical (which response "D" suggests). Responses "A" and "D" are correct in stating that you cannot bill the insurance company, since you should only bill insurance for services that you actually provide (Standard 6.06: Accuracy in Reports to Payors and Funding Sources). While your financial agreement may or may not require the client to cover the co-payment for missed sessions (response "C"), Current Procedural Terminology (CPT) codes do not provide a code for no-shows, and billing for services not rendered is unethical

You belong to a managed-care panel and a client with a 12-session limit was referred to you. Without calling to cancel, she does not show up for her fourth session. How would you bill for the missed session? A. bill the insurance company B. bill the client according to office policies C. bill the client for the co-payment and the insurance company for the no-show D. you cannot bill either the client or insurance company

C. According to the ethical standards, a referring psychologist can receive payment from the new psychologist, as long as the payment is "based on the services (clinical, consultative, administrative, or other) provided and is not based on the referral itself." In this situation, it appears that the new psychologist is paying for legitimate consultative services, rather than providing you with a "kickback." Therefore, the arrangement appears to be ethical

You conduct a psychological assessment on a patient and then refer the patient to another psychologist. The new psychologist doesn't understand your assessment and offers to pay you for consultation concerning the assessment. The proposed arrangement is: A. unethical, because it represents a payment for a referral. B. unethical, because it represents a dual relationship. C. ethical, as long as the payment is not for the referral. D. ethical, because the current version of the APA's ethical standards permits payment for referrals.

A. Because this evaluation was court-ordered you were not actually required to obtain informed consent. However Standard IV.E.1 of the Speciality Guidelines for Forensic Psychologists states that "if the client appears unwilling to proceed after receiving a thorough notification of the purposes, methods, and intended uses of the forensic evaluation, the psychologist should take steps to place the client in contact with his or her attorney for the purpose of legal advice on the issue of participation." Remember, you always want to take the most careful and conservative approach if possible. While you may be tempted to choose answer C and add the client's comments to your report, this would contradict Standard V.C. of the Forensic Guidelines which states that, "In situations where the right of the client to confidentiality is limited, the forensic psychologist makes every effort to maintain confidentiality with regard to any information that does not bear directly upon the legal purpose of the evaluation." Finally, as a thorough evaluator you should have read the case file closely; however, you would not proceed with an evaluation differently just because your client is an attorney

You have been court-ordered to evaluate a prisoner who is being tried for murder. You explain the purpose of the evaluation and complete it. As you are packing up your testing materials, the prisoner smirks, and says, "I agreed to this but I didn't sign anything. You are out of luck, doctor." A. You should go ahead and complete the report, but leave out the prisoner's comments. B. The prisoner is correct -- you should refer for a new assessment. C. You should add this verbal comment to the report, noting the prisoner's passive-aggressive tendencies. D. You should have read the case file because you would have learned that the accused is also an attorney; this would affect how you would treat this prisoner.

C. None of the answers are quite right, but response C is the best choice. It suggests you first try to explore the clinical transference. However, once investigated, the resolution probably won't include a new sweater. Choice A is incorrect because while it is usually unethical to accept gifts, it is not illegal to do so. Choice B is a simple solution, but doesn't demonstrate any clinical exploration of the situation.

You have been seeing a female client in therapy for over a year and on your birthday she arrives with a beautifully gift-wrapped sweater and birthday card. You should: A. explain that it is unethical and illegal to accept anything from a client. B. keep the card, but don't accept the sweater. C. explore the meaning of the gift. D. agree to keep the gift if the client will exchange it for a different color -you have already received quite a few green sweaters.

A. external locus of control, internal locus of responsibility.

You see an African-American client who tells you that he can't get anywhere in life due to racism, but then tells you "I get what I deserve." Based on this information, the client has a world view of: A. external locus of control, internal locus of responsibility. B. external locus of control, external locus of responsibility. C. internal locus of control, internal locus of responsibility. D. internal locus of control, external locus of responsibility.

C. The Ethics Code (Standard 1.08) explicitly states that a person cannot be denied tenure solely because he or she has made or is the subject of an ethics complaint. Therefore, it would be unethical for you to vote against tenure in this case if your only reason was that the professor has been accused of sexual harassment. If, however, the allegations had been proven, it would be ethical to vote against tenure.

You work for a university on a panel set up to make personnel decisions. The panel is considering granting tenure to a professor who has an unproven charge of sexual harassment pending. You vote against granting the professor tenure. Your actions are: A. ethical because the Ethics Code states that psychologists should do whatever they can to prevent sexual harassment. B. ethical because, as a member of this panel, you have the right to vote however you please, regardless of your reasons. C. unethical if the reason for your vote was the allegation of sexual harassment. D. unethical regardless of the reasons for your vote.

A. The nature and duration of Lily's symptoms and the fact that they're due to not getting a "dream job" are most suggestive of a diagnosis of Adjustment Disorder with depressed mood. Her symptoms seem to be out of proportion to their cause and have affected her functioning (e.g., ability to look for another job or use her time more productively). No information is provided in the question indicating that meets the diagnostic criteria for Major Depressive Disorder, Acute Stress Disorder, or Persistent Depressive Disorder

Your new client, Lily, says that, since learning she did not get the "dream job" she applied for two weeks ago, she has felt hopeless about finding a good job. She tells you that she has not been her "usual happy self" and that she's been staying at home and "wasting time" on the Internet and cleaning out her closets and kitchen cabinets. The most likely diagnosis for Lily is: A. Adjustment Disorder with depressed mood. B. Major Depressive Disorder. C. Acute Stress Disorder. D. Persistent Depressive Disorder.

C. There are two major types of hypertension: Primary hypertension is also known as essential hypertension. It's the most common type and is diagnosed when the cause is unknown. Secondary hypertension is secondary to a known disorder or condition. Stage 1 and Stage 2 refer to the severity of hypertension, with Stage 2 indicating a greater elevation in blood pressure than Stage 1

____________ hypertension has no currently identifiable cause. A. Secondary B. Stage 1 C. Primary D. Stage 2

C. Secondary prevention techniques are presented to individuals who have been identified as being at high risk by some type of screening procedure whereas primary preventions (b.) are offered to groups of individuals who have been identified as being at high-risk. Tertiary preventions (d.) are for people who have already developed a disorder and are aimed at reducing relapse and other negative consequences.

______________ preventions are offered to individuals identified as being at high risk by a screening test or some type of screening procedure. A. community B. primary C. secondary D. tertiary

cognitive dissonance

a theorized drive individuals have to reduce discomfort caused by holding to conflicting cognitions; this construct has been used to explain the results of studies in which individuals change their beliefs to be more consistent with how they have behaved in the study

The Barnum Effect

also known as the Forer effect, is defined as the tendency for people to accept very vague or general feedback as uniquely applicable to themselves without realizing that the same description could be applied to just about anyone, such as horoscopes, as accurate.

The tend-and-befriend response

hypothesized to be typical for females; stressors prompt females to protect their offspring and join social groups to reduce vulnerability.

referent power (def)

is based on a person's attraction to or desire to be like the holder of power

Expert power

is based on the belief that the power holder has special knowledge or expertise

Institutional racism

is defined as, "the structures, policies, practices, and norms resulting in differential access to the goods, services, and opportunities of society by race... often evident as inaction in the face of need" and manifests in restriction or denial of material conditions and access to power to members of minority groups

Elaboration Likelihood Model

persuasive messages can be communicated via a central route or a peripheral route. The central route is used when listeners think carefully about the contents of the message. When listeners do not think carefully about an argument's content, they rely more on superficial (peripheral) cues. Peripheral cues include the messengers' perceived attractiveness or expertise, and the receivers' mood.

Self verification theory

proposes that people seek confirmation of their self-concept

fundamental attribution bias or error

refers to the tendency to overestimate dispositional (personality) factors and underestimate situational factors in explaining a person's behavior (is the tendency to explain others' behavior in terms of internal disposition, (e.g., personality traits, abilities, motives, etc.) as opposed to external situational factors)

Functional fixedness (def)

refers to the tendency to think of objects only in terms of their usual functions

Coercive power

results from the holder's ability to punish others.

reward power

results from the holder's ability to reward others

Subjective validation

the process of validating words, initials, statements or signs as accurate because one is able to find them personally meaningful and significant

Self-serving bias

the tendency to attribute one's successes to internal factors and one's failures to external factors. (the tendency to take credit for our successes and to blame situational factors for our failures)

actor-observer effect

the tendency to overestimate situational factors and underestimate dispositional factors regarding one's own behaviors and to underestimate situational factors and overestimate dispositional factors in others. (the tendency to attribute one's own behavior to situational factors and the behavior of others to dispositional factors)

Confirmatory bias (or confirmation bias)

the tendency to seek, interpret, and create information that verifies our existing beliefs (the tendency to seek and interpret information that confirms one preconceptions and avoid information that does not confirm them)


Kaugnay na mga set ng pag-aaral

Drawing Conclusions in A Black Hole Is NOT a Hole

View Set

Chapter 13: Screening for Cancer

View Set

Critical Thinking & Analysis Final Exam Practice (study.com)

View Set

Ch. 8 Interpersonal Communication

View Set

Unit 4 quiz review- infection, inflammation & immunity

View Set

Quiz: Chapter 3 - Computers in Business Assessment

View Set

Strength and Conditioning Final study guide

View Set